Will's study set

¡Supera tus tareas y exámenes ahora con Quizwiz!

ANSWER: A The findings in this patient are consistent with a lung abscess caused by anaerobic organisms, which is usually related to aspiration. Most patients have a history of compromised consciousness, such as a seizure disorder or drug and alcohol abuse, and many have dental or gingival disease. Routine bacteriologic studies of expectorated sputum are hampered by mouth contamination, but are useful for detecting mycobacteria and other potential etiologic agents. An open lung biopsy, immediate bronchoscopy, and transtracheal aspiration are useful when the patient has not responded to initial therapy. Since this is a typical clinical picture for anaerobic lung abscess, the treatment of choice would be large doses of intravenous clindamycin.

A 28-year-old white male alcoholic presents with a 4-week history of fever, malaise, cough, sputum production, and weight loss. Twelve hours ago his cough increased and he noted frank blood in his sputum, along with a foul taste. Sputum is obtained for routine culture. A chest radiograph is shown in Figure 5. Which one of the following diagnostic procedures should be performed prior to the initiation of therapy? A) A culture of expectorated sputum B) An open lung biopsy C) Immediate bronchoscopy D) Transtracheal aspiration

ANSWER: D Scleritis is an inflammatory disorder affecting the sclera, often associated with a connective tissue disorder such as rheumatoid arthritis, systemic lupus erythematosus, Wegener's granulomatosis, polyarteritis nodosa, or relapsing polychondritis. In the anterior form of scleritis, inflammation results in local or diffuse erythema and thickening of the sclera. Patients present with either diffuse or focal ocular erythema, tenderness, and pain. When the inflammation is focal, a tender nodule may be present. The initial treatment of scleritis is an oral NSAID to help reduce ocular inflammation. Topical or systemic corticosteroids may be used when NSAIDs fail or are contraindicated (SOR B).

A 37-year-old previously healthy male presents with a 1-week history of a painful swollen area on his left eye, associated with redness (see Figure 3). He complains of dull aching in the eye that radiates to his ipsilateral temple. He denies any ocular discharge or vision changes. His visual acuity is normal. Which one of the following is the most appropriate first step in managing this patient's condition? A) A topical mydriatic agent B) A topical cycloplegic agent C) A topical antibiotic D) An oral NSAID E) An oral antibiotic

Right ventricular infarction is most frequently an extension of an inferior myocardial infarction (MI). Right-sided precordial leads may need to be evaluated to document this on the EKG. Hypotension during the acute event, especially after nitrate administration, is characteristic. Frequently, 1-2 liters of normal saline must be administered. Acute pericarditis most frequently develops 2-4 days after the infarction. Free wall rupture, septal rupture, and papillary muscle rupture typically do not occur until 1-5 days after the acute MI. Septal rupture is more common with anterior MI.

A 58-year-old African-American male presents to the emergency department with a 6-hour history of substernal pain. He has a previous history of hypertension. His blood pressure drops to 60 mm Hg systolic after three sublingual nitroglycerin tablets. His EKG is shown in Figure 6. In addition to nitrate side effects, which one of the following should be the first diagnostic consideration? A) Right ventricular infarction B) Pericarditis with tamponade C) Papillary muscle rupture D) Ventricular free wall rupture E) Ventricular septal rupture

A 45-year-old female has an alkaline phosphatase level that is twice the normal level on a chemistry panel ordered for evaluation of pruritus. Other tests of liver function are within normal limits, including bilirubin and ALT (SGPT) levels, and repeat testing 2 months later shows no change. A -glutamyltransferase level is also significantly elevated, as is an antimitochondrial antibody titer. Hepatic ultrasonography is unremarkable. Which one of the following diagnoses is most likely? A) Primary biliary cirrhosis B) Paget's disease of the bone C) Sarcoidosis D) Choledocholithiasis E) Drug-induced cholestasis

ANSWER: A A middle-aged woman with pruritus and elevated levels of alkaline phosphatase, -glutamyltransferase (GGT), and antimitochondrial antibody titers is likely to have primary biliary cirrhosis. Levels of 5'-nucleotidase or GGT are usually elevated in parallel with those of alkaline phosphatase in patients with liver disease, but not in patients with bone disorders. Infiltrative liver disease, as seen with sarcoidosis, drug-induced cholestasis, and choledocholithiasis, is not associated with elevated antimitochondrial antibody levels. Patients with choledocholithiasis will also usually have dilated hepatic ducts on ultrasonography. This patient should next have a liver biopsy to confirm her diagnosis.

A 64-year-old male with a previous history of hypertension and atrial fibrillation presents with an acute onset of ataxia, headache, mild confusion, and restlessness. His only current medications are lisinopril (Prinivil, Zestril) and warfarin (Coumadin). On examination his blood pressure is 160/100 mm Hg, pulse rate 86 beats/min, respirations 12/min, and temperature 36.7°C (98.1°F). A CBC, serum electrolyte levels, and cardiac enzyme levels are normal. His INR is 1.1. Noncontrast CT shows a cerebellar hemorrhage with a hematoma volume of 50 mL. Which one of the following should be performed urgently? A) Neurosurgical consultation for posterior cerebellar hematoma decompression B) A reduction in blood pressure to 140/90 mm Hg C) Administration of vitamin K, 10 mg intravenously D) Administration of mannitol (Osmitrol), 0.5-1.0 mg/kg intravenously E) Induction of hypothermia to achieve a body temperature of 34.4°C (94.0°F)

ANSWER: A Aggressive neurosurgical intervention is not indicated to evacuate clots in patients with intracerebral hemorrhage except in those with a cerebellar hemorrhage, which is always an indication for neurosurgical consultation. Guidelines have been developed by the American Heart Association for lowering blood pressure in patients with a systolic blood pressure >180 mm Hg, or a mean arterial pressure >130 mm Hg. The use of various forms of osmotherapy, including mannitol, to prevent the development of cerebral edema has not been shown to improve outcomes. The data regarding hypothermia induction is unclear. Patients with an INR >1.5 should receive therapy to replace vitamin K-dependent factors and have their warfarin withheld.

A 67-year-old female sees you for a routine follow-up visit for hypertension. Her chemistry profile and other laboratory studies are normal except for a serum calcium level of 10.9 mg/dL (N 8.4-10.4). This result is confirmed on repeat testing and is elevated when adjusted for her albumin level. Her current medications are lovastatin (Mevacor), 20 mg daily for hypercholesterolemia, and lisinopril (Prinivil, Zestril), 10 mg daily for hypertension. Her medical history is otherwise negative. Her parathyroid hormone level is 74 pg/mL (N 15-75), her serum creatinine level is 1.1 mg/dL (N 0.6-1.5), and her 25-hydroxyvitamin D level is 26 ng/mL (N 14-60). Which one of the following is the most likely diagnosis? A) Primary hyperparathyroidism B) Occult malignancy C) Sarcoidosis D) Paget's disease of bone E) Hypervitaminosis D

ANSWER: A All of the diagnoses listed may cause elevations of serum calcium, but malignancy, sarcoidosis, Paget's disease, and hypervitaminosis D are all associated with suppressed levels of parathyroid hormone. While the parathyroid levels in this patient are within the normal range, they are inappropriately high for the level of serum calcium and suggest hyperparathyroidism, the most common cause of hypercalcemia in this age group.

A 54-year-old male presents to your office with a chief complaint of vomiting and diarrhea, along with stomach cramps. He has not noticed blood in his stool or vomit. His symptoms began in the middle of the night, approximately 4 hours after he ate at a local delicatessen. He has not been out of the country and has not eaten any exotic foods or foods that are not part of his normal diet. His vital signs include a temperature of 37.0°C (98.6°F), a pulse rate of 90 beats/min, and a blood pressure of 130/80 mm Hg. Which one of the following organisms is implicated in this patient's presumed case of food poisoning? A) Staphylococcus aureus B) Clostridium botulinum C) Campylobacter jejuni D) Enterohemorrhagic Escherichia coli

ANSWER: A Among the causes of food poisoning, Staphylococcus aureus is associated with the shortest incubation period (1-6 hours). Most cases are related to contamination of food by infected human carriers. Leaving food to cool slowly at room temperature allows organisms that produce enterotoxins to multiply. Common food sources include ham, poultry, potato or egg salad, cream, and pastries. Campylobacter jejuni infections can be subclinical or symptomatic. Symptoms usually occur within 2-4 days of exposure to the organism in food or water. A prodrome of fever, headache, and myalgias occurs 24-48 hours before the diarrheal symptoms begin. Vomiting is usually not a symptom of the infection. Clostridium botulinum is usually associated with canned foods. Enterohemorrhagic Escherichia coli produces a Shiga-like toxin, which kills intestinal epithelial cells. While the symptoms are like those of food poisoning, they are much more severe and are usually associated with bloody diarrhea (SOR C).

You are discussing job-related exposure to human immunodeficiency virus (HIV) with your certified nursing assistants. Exposure to which one of the following from an HIV-positive patient would require consideration of post-exposure prophylaxis? A) Breast milk B) Saliva C) Sweat D) Urine E) Feces

ANSWER: A Breast milk is considered potentially infectious in patients with HIV infection, along with vaginal secretions, semen, and blood. Contact with saliva, sweat, urine, or feces does not require postexposure prophylaxis.

In symptomatic young children with Campylobacter enterocolitis that is refractory to conservative management, the preferred treatment is A) erythromycin B) ciprofloxacin (Cipro) C) ampicillin D) trimethoprim (Primsol) E) metronidazole (Flagyl)

ANSWER: A Campylobacter enterocolitis in children is generally a mild, self-limiting disease. However, in patients who are sick enough to require hospitalization or who remain symptomatic by the time a bacteriologic diagnosis has been made, antibiotic therapy is indicated. The preferred drug is oral erythromycin, which clinical trials indicate may produce clinical improvement. Ciprofloxacin may be an effective alternative to erythromycin in the treatment of Campylobacter, but it is contraindicated in young children. There is no evidence that ampicillin, trimethoprim, or metronidazole is effective for this disease.

You are treating a 68-year-old male for COPD, hypertension, systolic heart failure, and coronary artery disease. You are considering adding a -blocker but you are concerned that it could affect his COPD. Which one of the following options would be most appropriate for this patient? A) Metoprolol tartrate (Lopressor), 12.5 mg twice daily B) Nadolol (Corgard), 20 mg daily C) Sotalol (Betapace), 40 mg twice daily D) Timolol, 5 mg daily E) Avoiding -blocker use

ANSWER: A Cardioselective -blockers, such as metoprolol, should not be withheld from patients with COPD. Metoprolol could be started at a low dosage in this patient. Nadolol, timolol, and sotalol are not cardioselective.

A 14-year-old female is concerned because she is unable to gain weight. A review of systems reveals intermittent diarrhea and chronic dermatitis previously diagnosed as eczema. Her past and family histories are unremarkable. A laboratory workup is negative, including a complete metabolic profile, a TSH level, and a sweat chloride test. A stool sample is negative for WBCs, ova, and parasites. Which one of the following is true regarding this patient? A) She should be tested for IgA anti-tissue transglutaminase B) She should be placed on a lactose-free diet C) She should be referred to an eating disorders specialist D) She should have a colonic mucosal biopsy

ANSWER: A Celiac sprue (gluten-sensitive enteropathy) classically presents as a malabsorption syndrome associated with dermatitis herpetiformis. This dermatitis usually appears as excoriated papules, as it is extremely pruritic. The rash may be misdiagnosed as atypical psoriasis or nonspecific dermatitis. With the development and use of better diagnostic tests, it now appears that this disorder has been underdiagnosed. Symptoms include fatigue, weight loss, diarrhea, abdominal pain, anemia, bone pain, aphthous ulcers, stomatitis, infertility, impotence, alopecia areata, dental enamel defects, seizures, ataxia, and dermatitis. Serologic tests are now available to aid in confirming the diagnosis of celiac sprue, including IgA antigliadin antibody, IgG antigliadin antibody, IgA antiendomysial antibody, and IgA antitransglutaminase. Cystic fibrosis, Crohn's disease, and anorexia nervosa can cause weight loss but not dermatitis. Sprue affects the small intestine; a biopsy of the colon would be inappropriate given this presentation.

A 52-year-old male presents with major depressive disorder. He reports a decrease in his libido and is concerned about the impact of medication on his sex life. Which one of the following antidepressants is least likely to cause sexual dysfunction? A) Bupropion (Wellbutrin) B) Duloxetine (Cymbalta) C) Fluoxetine (Prozac) D) Mirtazapine (Remeron) E) Paroxetine (Paxil)

ANSWER: A Paroxetine has been shown to cause the highest rate of sexual dysfunction among the SSRIs and other antidepressants. The fewest sexual side effects occur with bupropion.h

A 72-year-old male is hospitalized for pneumonia. During his hospitalization he develops diarrhea due to Clostridium difficile infection. Appropriate antibiotics are prescribed. What other measures should be taken to prevent the spread of infection in the hospital? A) Contact precautions with gown and gloves plus handwashing with soap and water B) Contact and respiratory precautions with gown, gloves, and face mask C) Use of hand sanitizers before and after patient contact D) Placing the patient in a reverse airflow room

ANSWER: A Clostridium difficile infection is a common cause of diarrhea in hospitalized patients, and recent antibiotic use is a risk factor for infection. The bacteria can be spread in a hospital setting by contact, and contact precautions with gown and gloves are indicated in addition to hand washing with soap and water to ensure removal of spores (SOR A). Hand sanitizer is inadequate, as it does not kill the spores. Respiratory precautions are not necessary.

The fluid of choice for resuscitation after a significant burn injury is A) lactated Ringer's solution B) hypertonic saline C) packed RBCs D) whole blood E) 5% albumin

ANSWER: A Crystalloids are the essential component of fluid resuscitation in patients with severe burn injuries, with lactated Ringer's solution being the most commonly used. Substantial early loss of blood is unusual, and transfusions are not often required. The use of colloids in these patients has not been shown to be helpful and may be harmful. Hypertonic saline solution may be useful in selected patients but requires careful monitoring and may be detrimental.

Which one of the following combination hormonal contraceptives is most effective in obese women? A) The etonogestrel/ethinyl estradiol vaginal ring (NuvaRing) B) The norelgestromin/ethinyl estradiol transdermal patch (Ortho Evra) C) Oral norethindrone/ethinyl estradiol (Aranelle, Brevicon) D) Oral levonorgestrel/ethinyl estradiol (Aviane, Seasonale) E) Oral drospirenone/ethinyl estradiol (Ocella, Yaz)

ANSWER: A Depot medroxyprogesterone acetate and the combination contraceptive vaginal ring are the most effective hormonal contraceptives for obese women because they do not appear to be affected by body weight. Women using the combination contraceptive patch who weight 90 kg may experience decreased contraceptive efficacy. Obese women using oral contraceptives may also have an increased risk of pregnancy.

A 58-year-old female with diabetes mellitus complains of 2 years of right shoulder pain, which is worse with activity. There has been no trauma. She tells you one of her friends had a similar problem and was treated successfully with "some sort of shock wave treatments." Which one of the following diagnoses is most likely to be successfully treated with extracorporeal shock wave therapy? A) Calcific tendinitis B) Gout C) Partial rotator cuff tear D) Frozen shoulder E) Hooked acromion

ANSWER: A Extracorporeal shock wave therapy is effective for calcific tendinitis of the rotator cuff. Side effects include bruising and pain. Needling and irrigation, physical therapy, and cortisone injections are sometimes used in patients with acute symptoms. Endoscopic and open surgical treatments are alternatives to extracorporeal shock wave therapy in refractory cases. Extracorporeal shock wave therapy does not have an established role in gout, rotator cuff tear, frozen shoulder, or hooked acromion.

You make a diagnosis of polymyalgia rheumatica in a 72-year-old female. This is a new diagnosis for this patient and she has not received any treatment for this condition up to this point. Which one of the following prednisone regimens would be the best initial treatment? A) 15 mg daily with a slow taper B) 15 mg daily with a rapid taper C) 30 mg daily with a slow taper D) 30 mg daily with a rapid taper E) 60 mg daily with a rapid taper

ANSWER: A For the initial treatment of polymyalgia rheumatica, current evidence suggests using prednisone, 15 mg daily, or its equivalent, with slow tapering. Relapses are more common with an initial dosage of 10 mg daily, and slow tapering is associated with fewer relapses. Few patients require a dosage greater than 15 mg/day, which increases the risk for adverse effects.

A 37-year-old male returns for follow-up after an episode of nephrolithiasis. He passed a 3-mm calcium oxalate stone and requests information about preventing further stones. You would advise that he A) drink up to 2 L of water/day B) increase his consumption of meats and grains C) increase the level of fructose in his diet D) restrict foods high in oxalate, such as spinach and rhubarb

ANSWER: A General recommendations regarding prevention of recurrent nephrolithiasis include increasing fluid intake up to 2 L of water daily (SOR B); greater volumes may lead to electrolyte disturbances and are not recommended. More specific dietary recommendations depend on the stone type. If the stone is not recovered, the type may be inferred from a 24-hour urine collection for calcium, phosphorus, magnesium, uric acid, and oxalate. Approximately 60% of all stones in adults are calcium oxalate. Uric acid stones account for up to 17% of stones and, like cystine stones, form in acidic urine. Alkalinization of the urine to a pH of 6.5-7.0 may reduce stone formation in patients with these types of stones. This includes a diet with plenty of fruits and vegetables, and limiting acid-producing foods such as meat, grains, dairy products, and legumes. Drinking mineral water, which is relatively alkaline with a pH of 7.0-7.5, is also recommended. Restriction of dietary oxalates has not been shown to be effective in reducing stone formation in most patients. Acidification of the urine to a pH 7.0 is recommended for patients with the less common calcium phosphate and struvite stones. This can be accomplished by consumption of at least 16 oz of cranberry juice per day, or by taking betaine, 650 mg three times daily.

Which one of the following is the most common cause of death for African-American males in the United States? A) Heart disease B) Stroke C) Cancer D) Accidental injuries E) Homicide

ANSWER: A Heart disease is the leading cause of death in the U.S., and this holds true for both men and women. Among men the only ethnicity for which heart disease is not the most common cause of death is Asian/Pacific Islander.

Which one of the following patients is eligible for the Medicare hospice benefit? A) A patient with end-stage COPD with a life expectancy of 6 months B) A patient with amyotrophic lateral sclerosis with a life expectancy of 9 months C) A patient on hemodialysis with a life expectancy of 12 months D) A patient with stage IV breast cancer with a life expectancy of 18 months

ANSWER: A Patients with a life expectancy of 6 months or less are eligible for the Medicare hospice benefit. This benefit allows patients to receive hospice care in either the home or hospital setting. In addition to patients with terminal cancer, patients with end-stage cardiac, pulmonary, and chronic debilitating diseases are eligible. Approximately two-thirds of patients enrolled in hospice die from non-cancer-related diagnoses, and approximately 60% of Medicare patients are not enrolled in hospice at the time of their death.

In a child, which one of the following findings on cardiac auscultation is most likely to be associated with structural heart disease? A) Increased murmur intensity with standing B) An early systolic murmur C) A murmur limited to a small area D) An S2 with a variable split duration E) A musical or low pitch

ANSWER: A Heart murmurs are common in children and adolescents. Often the murmur is innocent, but it may also be the only finding in an asymptomatic child with structural heart disease. Physical findings that should lead one to consider evaluation for structural heart disease include increased intensity with standing, a holosystolic murmur, a grade of 3 or higher, a harsh quality, an abnormal S2, maximal intensity at the upper left sternal border, a diastolic murmur, or a systolic click. Characteristics that are more likely to be associated with innocent murmurs include a systolic murmur, a soft sound, a short duration, a musical or low pitch, intensity that varies with phases of respiration, increased loudness in the supine position, and increased loudness with exercise, anxiety, or fear. If the diagnosis of an innocent murmur cannot be made from physical findings, an echocardiogram is the most appropriate study. A chest radiograph and EKG rarely assist in the diagnosis of heart murmurs in children (SOR B) and should not routinely be ordered.

A 12-year-old male presents to the office with a 2-day history of fever, myalgias, and rhinorrhea. He is otherwise healthy, and you suspect influenza. Which one of the following is the most appropriate next step in the management of this patient? A) Symptomatic treatment only B) Diagnostic testing to confirm influenza infection C) Oseltamivir (Tamiflu) D) Amantadine (Symmetrel) E) Antibiotics to prevent bacterial coinfection

ANSWER: A Influenza should be diagnosed on the basis of clinical signs and symptoms rather than diagnostic testing. Antiviral treatment is not recommended in otherwise healthy adults and children. Symptomatic treatment should be initiated with over-the-counter antipyretics and anti-inflammatory medications, and aspirin should be avoided due to the risk of Reye's syndrome. Antibiotics are indicated only when a bacterial coinfection is diagnosed and not for prophylaxis.

A 71-year-old white male with COPD and lung cancer is discharged from the hospital. In addition to a medical diagnosis, which one of the following criteria is used to determine whether Medicare will pay for his home oxygen therapy? A) Oxygen saturation B) PCO2 C) FEV1 D) The patient's finances

ANSWER: A Medicare eligibility for home oxygen therapy is based on oxygen saturation. To qualify for continuous long-term oxygen therapy the patient must have a PaO2 less than or equal to 55 mm Hg or an SaO2 less than or equal to 88 mm Hg.

A 22-year-old female in her second trimester of pregnancy presents with a 48-hour history of a sore throat. She has also had coryza and a nonproductive cough. A physical examination reveals a temperature of 37.3°C (99.2°F) and a blood pressure of 110/70 mm Hg. A HEENT examination reveals tonsillar and pharyngeal erythema with no exudate. There is no adenopathy. Her chest is clear. Which one of the following would be most appropriate for this patient? A) Reassurance and symptomatic treatment only B) A routine throat culture C) A rapid antigen detection test for Streptococcus D) Azithromycin (Zithromax) for 5 days E) Penicillin V for 10 days

ANSWER: A Most episodes of pharyngitis are caused by viral rather than bacterial infections. The use of clinical decision rules for diagnosing group A -hemolytic streptococcal pharyngitis improves quality of care while reducing unwarranted treatment and overall cost (SOR A). The original Centor score used four signs and symptoms to estimate the probability of acute streptococcal pharyngitis in adults with a sore throat, and was later modified by adding age as a fifth criterion. One point each is assigned for (1) absence of cough, (2) swollen, tender anterior cervical nodes, (3) temperature >38.0°C (100.4°F), and (4) a tonsillar exudate and swelling. One point is added for patients between the ages of 3 and 14 years, and a point is subtracted for patients over the age of 45. The cumulative score determines the likelihood of streptococcal pharyngitis and the need for antibiotics, and guides testing strategies. Patients with a score of zero or 1 are at very low risk for streptococcal pharyngitis and do not require testing or antibiotic therapy. Patients with a score of 2-3 should be tested using a rapid antigen test or throat culture, and a positive result warrants antibiotic therapy. Patients with a score of 4 or higher are at high risk for streptococcal pharyngitis, and empiric treatment may be considered. This patient's score is zero, and no testing or treatment is warranted.

Which one of the following is true concerning people in the United States who do not have health insurance? A) Most uninsured people are members of a family with at least one working adult B) Most uninsured people who are employed full-time work for large companies C) Most uninsured people who work part-time and have incomes below the poverty line are eligible for Medicaid D) On average, uninsured people have as much access to routine health care as those with insurance

ANSWER: A Most uninsured people in the United States are members of a family with at least one working adult. Most uninsured people who are employed work for small companies or work part-time. Most uninsured people who work part-time with incomes below the poverty line are not eligible for Medicaid. On average, uninsured people have less access to care and have poorer health outcomes.

A 35-year-old white female schoolteacher presents with anxiety, fatigue, and insomnia. The symptoms began after a heart murmur was discovered on a routine physical examination. An echocardiogram revealed mild mitral valve prolapse. A student at her school recently died suddenly on a school field trip because of undiagnosed idiopathic hypertrophic cardiomyopathy and the patient is now afraid she will die in a similar manner. She is anxious, sleepless, and fearful of physical activity. You perform a physical examination and EKG, with normal results. Which one of the following would be most appropriate at this point? A) Reassurance regarding the benign course of her condition B) A stress test C) Clonazepam (Klonopin) D) Referral to a cardiologist E) Referral for group psychotherapy

ANSWER: A Much of the psychological distress caused by the diagnosis of mitral valve prolapse is related to a lack of information and a fear of heart disease, which may be reinforced by the death of a friend or relative. A clear explanation of mitral valve prolapse, along with printed material, is a powerful aid in relieving the patient's emotional distress. The American Heart Association publishes a helpful booklet about this condition which can be given to these patients. It is important to avoid reinforcing illness behavior with unnecessary testing, medications, or referrals to specialists.

The diagnosis of Osgood-Schlatter disease (osteochondritis of the tibial tubercle apophysis) is best made on the basis of findings from A) the history and examination B) evaluation by an orthopedic specialist C) radiographs D) ultrasonography E) MRI

ANSWER: A Osgood-Schlatter disease is an inflammatory condition that is a common cause of knee pain in children and adolescents. The diagnosis is usually based on clinical findings, although radiographs may be necessary to rule out fractures or other problems if findings are not typical. MRI, ultrasonography, and orthopedic referral are not usually needed. The problem is typically self-limited and responds to activity modification, over-the-counter analgesics, stretching, and physical therapy.

A 23-year-old female becomes pregnant while using a copper T 380A intrauterine device (ParaGard) for contraception. Ultrasonography indicates an estimated gestational age of 8 weeks and confirms the location of the intrauterine device (IUD) within the uterus. A speculum examination shows the string coming through the cervix. Which one of the following is the best management strategy? A) Remove the IUD now B) Remove the IUD during the second trimester C) Remove the IUD after 37 weeks gestation D) Remove the IUD when the patient goes into labor E) Leave the IUD in place until delivery

ANSWER: A Pregnancy with an intrauterine device in place is rare but does occur. Removal of an in situ intrauterine device in early pregnancy reduces the risks of spontaneous abortion, preterm labor, and sepsis, so gentle removal should be accomplished as soon as the pregnancy becomes known.

Effective communication with patients from other countries requires knowledge of communication styles within various cultures. Which one of the following is consistently appropriate for all patients from non-English-speaking countries? A) Discouraging the use of family members as interpreters B) Expecting patients to make their own decisions regarding care C) Discussing test results with the patient only D) Maintaining eye contact with the patient

ANSWER: A Providing quality health care to individuals from diverse sociocultural backgrounds requires effective communication. Low health literacy in almost half of the U.S. population makes communication more difficult. When a language barrier exists it is better to have a professional interpreter than a family member, and children should be used as interpreters only in cases of emergency when no other source is available. The typical approach to medical care in the United States assumes that patients want to make their own decisions based on guidance from their health care providers. However, there are cultures in which patient autonomy is not the norm. There may be a specific authority figure in the family that is regarded as the decision maker. Effective communication also involves knowledge of communication styles within various cultures. Nonverbal communication can be through touch, eye contact, and personal space. For example, there are cultures in which direct eye contact is avoided, but in other cultures it is considered a sign of respect.

Which one of the following findings on examination of the head, oral cavity, and neck is associated with diabetes mellitus? A) Parotid enlargement B) Tooth erosion C) Diffuse melanin pigmentation D) Cobblestone oral mucosa E) Painful oral ulcers

ANSWER: A Sialadenosis, bilateral noninflammatory enlargement of the parotid gland, is associated with diabetes mellitus. Periodontal bleeding and inflammation, candidiasis, and delayed wound healing also are associated with diabetes mellitus. Tooth erosion can be an oral manifestation of gastroesophageal reflux disease or bulimia. Cobblestone oral mucosa is seen in Crohn's disease. Diffuse melanin pigmentation is an oral finding of Addison's disease. Painful oral ulcers occur in several conditions, including Behçet syndrome, aphthous ulcers, pemphigus, and pemphigoid.

A 63-year-old male presents with increasing shortness of breath over the past year. He smokes a pack of cigarettes a day, and has done so since he was 18 years old. Your evaluation leads to a diagnosis of COPD. Which one of the following interventions has been shown to slow the decline in lung function in this situation? A) Smoking cessation B) Regular use of an inhaled short-acting 2-agonist C) Regular use of an inhaled long-acting 2-agonist D) Regular use of an inhaled long-acting anticholinergic agent E) Regular use of oral corticosteroids

ANSWER: A Smoking cessation slows the decline of lung function in COPD. Long-acting 2-agonists, anticholinergic agents, and inhaled corticosteroids are useful for improving the symptoms of COPD. They improve exercise tolerance and quality of life, and can reduce the frequency of exacerbations. However, they do not slow the progression of COPD. Oral corticosteroids, along with antibiotics, are useful in treating acute exacerbations of COPD, but long-term treatment is not recommended.

A 73-year-old female presents with signs of an acute ischemic stroke, which began 2 hours earlier. She has a National Institutes of Health Stroke Scale score of 14. Noncontrast head CT shows no sign of hemorrhage. Which one of the following treatments is recommended and FDA approved for patients with this problem who have no contraindications? A) Intravenous tissue plasminogen activator (tPA) B) Warfarin (Coumadin) C) Glycoprotein IIb/IIIa receptor antagonists D) Aspirin and clopidogrel (Plavix) E) Heparin

ANSWER: A Studies have shown that the use of intravenous tissue plasminogen activator offers sustained patient benefit at 6 and 12 months if given within 3 hours of symptom onset (SOR B). All other listed interventions have not been shown to be efficacious.

Which one of the following is the leading cause of death among adolescents age 12-19 in the United States? A) Accidents B) Suicide C) Homicide D) Cancer E) Heart disease

ANSWER: A Teenage mortality is an important public health issue because the majority of deaths among teenagers are caused by external causes of injury such as accidents, homicide, and suicide. The leading causes of death for the teenage population remained constant throughout the period 1999-2006: accidents (48% of deaths), homicide (13%), suicide (11%), cancer (6%), and heart disease (3%). Motor vehicle accidents accounted for 73% of all deaths from unintentional injury.

During a routine office visit, a 65-year-old female asks if she should be screened for carotid artery stenosis. The patient has a history of controlled hypertension and hypercholesterolemia, and a family history of stroke. Physical examination of the carotid artery is normal and the patient is asymptomatic. Which one of the following is consistent with U.S. Preventive Services Task Force and American Heart Association recommendations regarding carotid artery ultrasonography for this patient? A) She does not need screening ultrasonography at this time B) She should have one-time screening ultrasonography now C) She should have routine screening ultrasonography now and every 5 years D) She should have routine screening ultrasonography now and every 10 years

ANSWER: A The U.S. Preventive Services Task Force and the American Heart Association/American Stroke Association recommend not performing carotid artery screening with ultrasonography or other screening tests in patients without neurologic symptoms because the harms outweigh the benefits. In the general population, screening tests for carotid artery stenosis would result in more false-positive results than true-positive results. This would lead to surgical procedures that are not indicated or to confirmatory angiography. As a result of these procedures, some patients would suffer serious harms such as death, stroke, or myocardial infarction, which outweigh the potential benefit surgical treatment may have in preventing stroke.

A 56-year-old male sees you for a health maintenance visit. He inquires about the options for colon cancer screening. He has not had any screening tests performed in the past and has no personal or family history of colon cancer. You tell him that there are several alternatives, but according to the U.S. Preventive Services Task Force, recommendations regarding the optimal screening intervals vary by test. He opts for fecal occult blood testing. You recommend he repeat this test at which one of the following intervals? A) Yearly B) Every 5 years C) Every 7 years D) Every 10 years E) Never, if the results are negative

ANSWER: A The U.S. Preventive Services Task Force recommends that all adults be screened for colon cancer beginning at age 50 and continue regular screening until age 75 (SOR A). They recommend against continued routine screening in previously screened adults 75-85 years of age and against any screening in adults over 85 (SOR A). Most organizations do not recommend a particular screening method, but instead list screening options, including fecal occult blood testing, flexible sigmoidoscopy, and colonoscopy. The recommended interval for fecal occult blood testing is every year. There is new evidence based on randomized, controlled trials that participation and detection rates for advanced adenomas and cancer are higher for immunochemical fecal testing than for stool guaiac testing (SOR A). As long as results are normal, screening colonoscopy is recommended at 10-year intervals and screening sigmoidoscopy at 5-year intervals.

For adolescents with scoliosis, observation is always indicated for a curve below a threshold of A) 20° B) 30° C) 40° D) 50°

ANSWER: A The diagnosis of idiopathic scoliosis is based on a coronal plane curvature >10°. It is a diagnosis of exclusion after congenital, neuromuscular, and myopathic diseases and conditions have been ruled out. Adolescent scoliosis is most common, and occurs in about 2%-3% of adolescents. More marked curvature (>30°) occurs in about 0.3% of adolescents, as measured on posterior-anterior and lateral radiographs using the Cobb method. For mild degrees of curvature there is an even distribution between girls and boys, but girls have a tenfold greater risk for more severe curvature. 40 Screening for scoliosis in the asymptomatic adolescent is controversial; the U.S. Preventive Services Task Force recommends against routine screening in its most recent update in 2004 (D recommendation). However, if idiopathic scoliosis is discovered incidentally or when the adolescent or parent expresses concern about scoliosis, options for further evaluation and treatment include observation for curvatures of less than 20° and consideration for bracing and/or surgery for more severe curvatures. The risk of progression depends on the amount of growth remaining, the magnitude of the curve, and the patient's gender.

A 32-year-old gravida 2 para 1 with long-standing untreated hypertension presents at 8 weeks gestation for prenatal care. Her physical examination is normal except for a blood pressure of 156/114 mm Hg. Which one of the following would be most appropriate as initial treatment? A) Labetalol (Trandate) B) Lisinopril (Prinivil, Zestril) C) Losartan (Cozaar) D) Metoprolol (Lopressor, Toprol-XL) E) Nifedipine, immediate release (Procardia)

ANSWER: A The drug most often recommended as first-line therapy for hypertension in pregnancy is labetalol. Reports of an association of metoprolol with fetal growth restriction have given rise to the recommendation to avoid its use in pregnancy. Both ACE inhibitors and angiotensin-receptor blockers are contraindicated in pregnancy because of the risk of birth defects and fetal or neonatal renal failure. Immediate-release nifedipine is not recommended due to the risk of hypotension.

A 1-year-old female has head lice. She has three siblings who have been treated unsuccessfully for this problem with permethrin (Nix). Which one of the following would be the best alternative treatment for this child? A) Benzyl alcohol lotion (Ulesfia) B) Malathion 0.5% lotion (Ovide) C) Permethrin cream rinse D) Pyrethrin shampoo (Pronto) E) Spinosad (Natroba)

ANSWER: A The frequency of head lice infestations has increased in recent years, and resistance to permethrin is now common. Permethrin is unlikely to be effective in this child since her siblings' infestations have failed to respond to it. Of the other choices, only benzyl alcohol lotion is approved for use in children under 2 years of age.

A pregnant 32-year-old gravida 2 para 1 develops an acute deep-vein thrombosis in the left lower extremity during the third trimester. The patient had a cesarean delivery with her first pregnancy and wants to breastfeed. Which one of the following is the treatment of choice? A) Low molecular weight heparin B) Unfractionated heparin C) Warfarin (Coumadin) D) A vena cava filter

ANSWER: A The preferred anticoagulant for venous thrombosis during pregnancy is low molecular weight heparin. Unfractionated heparin requires more monitoring and may increase the risk of heparin-induced thrombocytopenia. Warfarin should not be used during pregnancy but may be used in women who are breastfeeding (SOR B).

The primary indication for joint replacement surgery in patients with osteoarthritis is A) intractable pain B) joint laxity C) limited range of motion D) recurrent subluxation

ANSWER: A The primary indication for joint replacement surgery in patients with osteoarthritis is intractable pain, which is almost always relieved by the surgery. Joint replacement may also be appropriate for patients with significant limitations of joint function or with altered limb alignment. Range of motion, joint laxity, and recurrent subluxation relate to musculotendinous function, and are not reliably improved by joint replacement.

A 35-year-old female immigrant from a rural village in Southeast Asia visits your clinic shortly after arriving in the United States. She presents with a 1-week history of low-grade fever and a nonproductive cough, and has crackles but no signs of consolidation or pleural effusion on examination. You order a chest radiograph and see several oval infiltrates, 1-2 cm in size. Which one of the following is the most likely cause of these symptoms? A) Ascaris lumbricoides B) Enterobius vermicularis (pinworm) C) Taenia saginata D) Taenia solium E) Diphyllobothrium latum

ANSWER: A This patient has the classic pulmonary manifestations of Ascaris infection, which develop during the transpulmonary passage of Ascaris larvae (SOR C). The larvae produce a syndrome of transient eosinophilic pulmonary infiltrates, commonly referred to as Löffler syndrome. Ascaris infection is the most common worldwide cause of this syndrome. Symptoms develop when larvae are within the lungs, approximately 9-12 days after ingestion of Ascaris eggs. Patients may develop the following symptoms and signs: • an irritating, nonproductive cough and burning substernal discomfort • dyspnea and blood-tinged sputum • urticaria during the first few days of the illness (15% of patients) • fever, which infrequently exceeds 38.3°C (101.0°F) • crackles and wheezing, with no signs of consolidation • hepatomegaly The acute symptoms generally subside within 5-10 days, depending upon the severity of the illness. The chest radiograph may show round or oval infiltrates ranging in size from several millimeters to several centimeters in both lung fields; these lesions are more likely to be present when blood eosinophilia exceeds 10%. The infiltrates are migratory and may become confluent in perihilar areas, and usually clear completely after several weeks. Taenia does not infiltrate the lungs, but forms cysts in the muscles. Diphyllobothrium latum, the fish tapeworm, does not cause pulmonary problems. Enterobius (pinworm) does not migrate from the gastrointestinal tract into other organs.

A 25-year-old male graduate student develops an acute headache, fever, and rash while visiting his parents during fall break. When he comes to the emergency department he has a widespread petechial rash and a stiff neck, and his blood pressure is 78/40 mm Hg. He is treated with appropriate empiric antibiotics, and the spinal fluid from a tap reveals a large number of polynuclear leukocytes and gram-negative diplococci. What is the most appropriate treatment at this point? A) Ceftriaxone (Rocephin) B) Rifampin (Rifadin) C) Ciprofloxacin (Cipro) D) Amoxicillin E) Doxycycline

ANSWER: A This patient likely has meningitis due to Neisseria meningitidis. Ceftriaxone is recommended as first-line therapy and should not be delayed once the diagnosis is suspected (SOR B). Ciprofloxacin and rifampin are not recommended as first-line therapy for infected individuals, but are recommended as prophylaxis for close contacts (SOR B). Doxycycline and amoxicillin are not proven to be effective for treatment or prophylaxis.

A 52-year-old African-American female has a chest radiograph after a PPD test is equivocal. She is a schoolteacher, and a child in her classroom has been confirmed as a TB contact. The radiograph shows large bilateral hilar nodes. She has recently been diagnosed with psoriatic arthritis on the basis of a scaly skin rash and arthralgias. A physical examination also reveals nodular skin lesions on her shins, and scattered, slightly enlarged lymph nodes. Which one of the following would be the most appropriate next step for confirming the diagnosis? A) A lymph node biopsy B) An antinuclear antibody test C) CT angiography of the chest D) Pulmonary function tests E) An echocardiogram

ANSWER: A This patient almost certainly has sarcoidosis. The diagnosis is supported by a compatible clinical and radiographic presentation, and histologic evidence of noncaseating granulomas on a biopsy (without organisms or particles). In patients who present with Löfgren syndrome (erythema nodosum, hilar adenopathy, and polyarthralgias), a probable diagnosis of sarcoidosis can be made without a biopsy. In all other cases a biopsy should be performed on the most accessible organ, such as the skin or peripheral lymph nodes. While the thorax is the most common site of disease, skin involvement occurs in at least 30% of patients and is often missed. This patient was diagnosed with psoriasis, which may have been another manifestation of sarcoidosis. Cutaneous sarcoidosis presents as single foci or crops, and is often attributed to other causes, perhaps because of its highly variable manifestations such as macular-papular, nodular, psoriatic-like, and hypomelanotic lesions. Careful skin examination is warranted because biopsy of a sarcoidal lesion has a high diagnostic yield.

A 73-year-old white female sees you for a routine visit. The patient is on multiple medications, and on examination her blood pressure is 140/80 mm Hg and her heart rate is 75 beats/min. She also has a systolic heart murmur, osteoarthritic changes of the knees, and a trace of peripheral edema. Her free T4 level is elevated, and her TSH level is <0.01 microU/mL (N 0.5-5.0). Which one of the following medications the patient is taking is most likely to cause abnormal thyroid hormone levels? A) Amiodarone (Cordarone) B) Digoxin C) Enalapril (Vasotec) D) Furosemide (Lasix) E) Lithium

ANSWER: A This patient has asymptomatic hyperthyroidism, which is more common in the elderly. Elevated T4 and markedly suppressed TSH are diagnostic. Common causes include Graves' disease, toxic adenoma, multinodular goiter, thyroiditis, and use of iodine-containing medications such as amiodarone. Amiodarone-associated hyperthyroidism may be related to either iodine excess or a toxic effect on the gland, causing thyroiditis (level of evidence 3). Lithium is associated with hypothyroidism.

A 50-year-old female presents with a 3-week history of a moderately pruritic rash, characterized by flat-topped violaceous papules 3-4 mm in size. The lesions are located primarily on the volar wrists and forearms, lower legs, and dorsa of both feet. Ten days after the rash first appeared she went to the emergency department and was treated for "possible scabies," but the treatment has made little or no difference. Which one of the following treatments is indicated at this time? A) Clobetasol (Cormax, Temovate) 0.05% ointment B) Permethrin 5% cream C) Tacrolimus (Protopic) 0.1% ointment D) Triamcinolone 0.1% cream

ANSWER: A This patient has classic lichen planus, with pruritic, symmetrically distributed papular lesions. The violaceous flat-topped papules, usually 3-6 mm in size, are distinct and so characteristic in appearance that a biopsy is usually not necessary to make the diagnosis. First-line treatment is with high-potency topical corticosteroids such as clobetasol, as mid-potency topical agents such as triamcinolone are ineffective. Topical calcineurin inhibitors, including tacrolimus, can be used in cases not responding to topical corticosteroids. While scabies can masquerade as a variety of other dermatoses, retreatment with a scabicide is not indicated in this patient.

A 35-year-old white female comes to your office with a 3-month history of the gradual onset of pain and tenderness in her wrists and hands. She also complains of 1 hour of morning stiffness. She denies rash, fever, or skin changes. On physical examination she has symmetric swelling of the proximal interphalangeal joints and metacarpophalangeal joints. Motion of these joints is painful. She has no rash or mouth ulcers. Radiographs of the hands and wrists are negative, and a chest film is unremarkable. A CBC is normal, but the erythrocyte sedimentation rate is elevated at 40 mm/hr. Latex fixation for rheumatoid factor is negative, and an antinuclear antibody (ANA) test is negative. The most likely diagnosis in this patient is A) rheumatoid arthritis B) systemic lupus erythematosus C) sarcoidosis D) Lyme disease E) calcium pyrophosphate deposition disease

ANSWER: A This patient has rheumatoid arthritis (RA) by symptoms and physical findings. A positive latex fixation test for rheumatoid factor is not necessary for the diagnosis. A negative rheumatoid factor does not exclude RA, and a positive rheumatoid factor is not specific. Rheumatoid factor is found in the serum of approximately 85% of adult patients with RA; in subjects without RA, the incidence of positive rheumatoid factor is 1%-5% and increases with age. The ANA test is positive in at least 95% of patients with systemic lupus erythematosus, but in only about 35% of patients with RA. Elevation of the erythrocyte sedimentation rate is seen in many patients with RA, and the degree of elevation roughly parallels disease activity. At a mean of 6 months after the onset of Lyme disease, 60% of patients in the United States have brief attacks of asymmetric, oligoarticular arthritis, primarily in the large joints and especially in the knee.

A 54-year-old female presents with a new onset of headaches. She rates her pain as severe and reports that the headaches frequently awaken her in the early morning. The patient has a history of hypertension and stage 4 chronic kidney disease. Her glomerular filtration rate is 24 mL/min. You suspect the patient has a brain tumor and order gadolinium-enhanced MRI of the head. Which one of the following is the patient at high risk for developing with the use of gadolinium contrast? A) Nephrogenic systemic fibrosis B) Anaphylaxis to contrast dye C) Dermatomyositis D) Focal seizures E) Hypertensive crisis

ANSWER: A This patient has stage 4 chronic kidney disease with a glomerular filtration rate (GFR) <30 mL/min. This puts her at high risk for developing nephrogenic systemic fibrosis. The FDA recommends against using gadolinium-based contrast agents in patients with acute or chronic kidney disease and a GFR <30 mL/min. While anaphylaxis is possible, this patient would not be at a high risk for this. Dermatomyositis, focal seizures, and hypertensive crisis are not associated with gadolinium-based contrast agents.

A 6-year-old male is brought to your office with a 1-day history of bloody diarrhea. The cause is determined to be enterohemorrhagic Escherichia coli, which is producing Shiga toxin. Which one of the following is the most appropriate treatment? A) Supportive treatment only B) Ciprofloxacin (Cipro) C) Clindamycin (Cleocin) D) Doxycycline E) Trimethoprim/sulfamethoxazole (Bactrim, Septra)

ANSWER: A Treatment of enterohemorrhagic Escherichia coli infection consists of supportive measures only. Antibiotics are contraindicated because they can trigger the release of Shiga toxins, which may lead to hemolytic-uremic syndrome in children.

A 50-year-old female presents for evaluation of dyspnea that tends to occur with exercise. She has a 40-pack-year history of smoking and has been diagnosed with exercise-induced asthma. She denies any other medical problems. You perform spirometry and find that the expiratory loop is normal and that she has a flattened inspiratory loop. What is the most likely diagnosis? A) Vocal cord dysfunction B) COPD C) Asthma D) Restrictive lung disease

ANSWER: A Vocal cord dysfunction is a disorder in which the vocal cords move toward midline during inspiration or expiration, leading to varying degrees of obstruction. It is commonly misdiagnosed as exercise-induced asthma. There are a number of precipitating factors, including exercise, psychological conditions, irritants, rhinosinusitis, and gastroesophageal reflux disease. Spirometry generally will show a normal expiratory loop with a flattened inspiratory loop. In asthma and COPD the FEV1/FVC ratio is decreased, resulting in a concave shape in the expiratory portion of the flow-volume curve. The inspiratory loops are generally normal. Patients with restrictive lung disease have a normal FEV1/FVC ratio with a reduced FVC.

A 67-year-old female who is a recently retired college professor takes warfarin (Coumadin) because of chronic atrial fibrillation. She asks you about the possibility for self-management of her anticoagulation using a portable monitor at home. Which one of the following is true regarding self-management of anticoagulation therapy compared to standard monitoring? A) It decreases the number of thromboembolic events B) It increases the number of bleeding events C) It increases all-cause mortality D) It increases the patient's level of anxiety E) When cost, complication rates, and mortality rates are considered, it is inferior to standard monitoring

ANSWER: A When self-management and standard management of anticoagulation therapy are compared, self-management improves the rate of minor hemorrhage, with no difference in the rate of major hemorrhage. Self-monitoring also improves the rate of thromboembolism. Both self-monitoring and self-management improve the rate of all-cause mortality. When studied, patients who self-managed their anticoagulation therapy perceived greater self-efficacy compared to patients receiving standard care, and self-management did not increase their levels of anxiety. When all factors are considered, self-monitoring and self-management have outcomes superior to those of standard monitoring and management.

A 22-year-old college student comes to your office to discuss her several-year history of abdominal pain and constipation. It has gotten worse since she returned to school this fall. She describes crampy pain and bloating that eases after defecation. Her bowel movements are firm and difficult to pass, and occur about every 3 days on average. Her symptoms have not included vomiting, weight loss, blood in the stool, or melena. Her menses are regular and she is an otherwise healthy young woman. Her family history is negative for any gastrointestinal or genitourinary diseases. On examination you find her abdomen to be soft and without masses, with no tenderness to palpation. Which one of the following would be most appropriate at this time? A) A therapeutic trial of increased soluble fiber intake B) A therapeutic trial of lubiprostone (Amitiza) C) Abdominal ultrasonography D) Abdominal CT E) Colonoscopy

ANSWER: A (Note: An article on this subject appeared in AFP after the exam was sent to the printer, and came to our attention after the exam was scored. B is now considered a better answer, and the critique and references have been updated to reflect this.) This patient's symptoms are consistent with irritable bowel syndrome (IBS). Her history, physical examination, and laboratory evaluation did not show any evidence of peptic ulcer disease, celiac disease, thyroid disease, or inflammatory bowel disease. Red flags include unintentional and unexplained weight loss, rectal bleeding, a family history of bowel or ovarian cancer, and a change in bowel habits to looser and/or more frequent stools persisting for more than 6 weeks in a person over 60. The patient does not have any of these findings and therefore does not require any additional testing to confirm the diagnosis of IBS. Patients should be given information that explains the importance of self-help in effectively managing their IBS. This should include information on general lifestyle, physical activity, and dietary intake. A Cochrane review showed that soluble fiber such as psyllium is not effective for IBS. Lubiprostone is effective for constipation-predominant IBS.

One of your patients is planning to fly from Nebraska to Israel and asks about measures to prevent or reduce jet lag. Which one of the following is supported by the best evidence? A) Melatonin should be started on the morning of departure and taken every morning for 1 week B) Melatonin should be taken nightly for 2-5 nights, beginning on the first night at the destination C) Melatonin will help only on the return flight D) A dose of caffeine equivalent to 3 cups of coffee should be taken every morning, beginning on the morning of departure

ANSWER: B A Cochrane review found that melatonin was effective for reducing jet lag, especially when crossing 5 or more time zones in an easterly direction. The drug can also be effective when crossing 2-4 time zones. The most effective dosage seems to be 0.5-5 mg taken at bedtime starting on the day of arrival, with higher doses being more effective. Taking it before departure does not help, and taking it earlier in the day could make jet lag worse. There does not seem to be any benefit from taking melatonin prior to departure, and melatonin is not recommended when flying westward.

The husband of a 25-year-old white female consults you about his wife. Eighteen months ago, her last pregnancy was complicated by placental abruption, hemorrhagic shock, and the birth of a stillborn infant. She did not lactate and has not menstruated since the delivery. Since that time she has become increasingly fatigued and apathetic and has noticed a marked decrease in her libido. Which one of the following is the most likely diagnosis? A) Prolonged grief reaction B) Postpartum pituitary necrosis C) Postpartum depression D) Iron deficiency anemia

ANSWER: B A prolonged grief reaction, postpartum depression, and iron deficiency anemia could all cause fatigue, apathy, and decreased libido, but none of these conditions is characterized by failure to lactate and amenorrhea. This patient most likely has postpartum pituitary necrosis (Sheehan's syndrome), a complication of childbirth in which hemorrhagic shock leads to pituitary necrosis. The syndrome is caused by the lack of hormonal influence from the anterior pituitary gland on other endocrine glands, resulting in failure to lactate, breast atrophy, mental apathy, low blood pressure, absence or deficiency of sweating, loss of secondary hair characteristics and libido, and loss of ovarian function, resulting in amenorrhea.

Effective therapy for myocarditis-induced dilated cardiomyopathy includes A) ibuprofen B) lisinopril (Prinivil, Zestril) C) methotrexate D) oseltamivir (Tamiflu) E) prednisone

ANSWER: B ACE inhibitors improve the quality of life and the prognosis for patients with myocarditis-induced dilated cardiomyopathy, just as they do for other patients with heart failure. Neither antiviral therapy nor immunosuppression has been shown to improve this type of cardiomyopathy when tested in controlled trials. NSAIDs actually increase mortality by worsening sodium retention.

A 5-month-old child begins to cough and wheeze. He has no previous history of respiratory problems, although he has had upper respiratory symptoms for the past 2 days. On physical examination you note tachypnea and mild intercostal retractions and wheezes. The most likely diagnosis is A) asthma B) bronchiolitis C) croup D) pertussis E) pneumonia

ANSWER: B Acute bronchiolitis is a viral illness most frequently caused by the respiratory syncytial virus. Its peak incidence occurs at approximately 6 months of age. The illness frequently causes a few days of mild upper respiratory symptoms, followed by increased coughing and wheezing. Examination often reveals tachypnea and use of the accessory muscles of respiration, such as intercostal retractions. Acute asthma is uncommon in the first year of life, and is difficult to diagnose without recurrent episodes or prior respiratory problems. Croup usually presents with stridor, and pertussis and pneumonia do not usually present with wheezing.

A 37-year-old male returns for his first follow-up visit after being diagnosed with major depression 4 weeks earlier. The patient is taking citalopram (Celexa), 20 mg/day. He is tolerating the medication well and his energy level and sleep are improved, but he still complains of anhedonia. He has no other health problems and takes no other medications. The most reasonable management at this point is to A) add aripiprazole (Abilify) B) increase the dosage of citalopram C) add bupropion (Wellbutrin) D) add levothyroxine (Synthroid)

ANSWER: B All of the treatment options listed may improve the patient's depression, but it is unnecessary to add a second agent until the initial drug is at the maximum recommended dosage. Citalopram can be increased to a dosage of 40 mg/day.

A 55-year-old female is concerned about variations in her heartbeat. She describes "fluttering," "flip-flopping," and sometimes "pounding" sensations in her chest, with occasional delays between beats. Her symptoms are episodic, and have been occurring for several months. They have not been present for the past week. The patient's family history is negative for thyroid disease, but she recalls some "heart trouble" in several family members that was accompanied by fainting spells, and at least one relative died suddenly. She takes no medications, has a negative psychiatric review of systems, and has a normal physical examination. Which one of the following would be most appropriate at this point? A) Reassurance that her symptoms are associated with a benign condition B) A standard 12-lead EKG C) Echocardiography D) Intermittent event (loop) cardiac monitoring E) Admission to a hospital-based cardiac monitoring unit

ANSWER: B All patients who present with palpitations should be evaluated for a cardiac cause, since this is the etiology in 43% of cases. A standard 12-lead EKG is the initial test of choice and, along with a history and physical examination, can determine the cause in 40% of cases. A normal resting EKG does not exclude a cardiac arrhythmia. Therefore, if the EKG is normal, palpitations of suspected arrhythmic etiology may require further investigation with ambulatory EKG monitoring. Echocardiography is helpful in evaluating patients for structural heart disease and should be performed when the initial history, physical examination, and EKG are unrevealing, or in patients with a history of cardiac disease or more complex signs and symptoms. This patient's family history, along with the fact that she takes no medications, suggests the possibility of familial long QT syndrome, which often can be diagnosed from a resting EKG. Diagnosing long QT syndrome is important, since it is associated with an increased risk of sudden cardiac death. Based on the patient's clinical presentation and evaluation, hospital admission is not warranted prior to obtaining a standard 12-lead EKG.

Which one of the following is most appropriate for the treatment of gonorrhea? A) Azithromycin (Zithromax) B) Azithromycin plus ceftriaxone (Rocephin) C) Cefixime (Suprax) D) Ciprofloxacin (Cipro)

ANSWER: B Because of increased resistance to fluoroquinolones, ciprofloxacin is no longer recommended for the treatment of gonorrhea. In addition, there appears to be emerging resistance to cephalosporins, as evidenced by an increase in the minimum inhibitory concentrations of cephalosporins between 2000 and 2010. Unfortunately, no other well-studied and effective alternative antibiotic treatment regimens are currently available. It also appears that gonococcal resistance to cefixime might develop before resistance to ceftriaxone. As a result, in 2011 the Centers for Disease Control and Prevention recommended dual treatment with ceftriaxone, 250 mg intramuscularly, and azithromycin, 1 g orally, as the most effective treatment for uncomplicated gonorrhea.

You prescribe enalapril (Vasotec) for a 68-year-old male with heart failure. At a follow-up visit 6 weeks later the patient's serum creatinine level is 2.5 mg/dL (N 0.6-1.5) and his serum potassium level is 5.7 mEq/L (N 3.4-4.8). His baseline values were normal. Which one of the following is a side effect of ACE inhibitors that is the most likely cause of these changes in renal function? A) Toxicity to the proximal renal tubules B) Impaired autoregulation of glomerular blood flow C) Microangiopathic arteriolar thrombosis D) Rhabdomyolysis E) Interstitial nephritis

ANSWER: B Blood flow to the kidney is autoregulated so as to sustain pressure within the glomerulus. This is influenced by angiotensin II-related vasoconstriction. ACE inhibitors can impair the kidney's autoregulatory function, resulting in a decreased glomerular filtration rate and possibly acute renal injury. This is usually reversible if it is recognized and the offending agent stopped. NSAIDs can exert a similar effect, but they can also cause glomerulonephritis and interstitial nephritis. Statins, haloperidol, and drugs of abuse (cocaine, heroin) can cause rhabdomyolysis with the release of myoglobin, which causes acute renal injury. Thrombotic microangiopathy is a rare mechanism of injury to the kidney, and may be caused by clopidogrel, quinine, or certain chemotherapeutic agents.

A 14-year-old male with Tanner stage 1 pubic hair has prepubertal-size testes. His height is at the 3rd percentile. The physical examination is otherwise unremarkable. Which one of the following additional findings would be most consistent with constitutional delay of growth and puberty? A) Impairment of the sense of smell B) Delayed bone age C) Elevated LH and FSH D) Elevated thyrotropin E) Elevated prolactin

ANSWER: B Constitutional delay of growth and puberty (CDGP) tends to be inherited. Bone age is delayed, but growth potential is often normal. LH and FSH are elevated in hypergonadotropic hypogonadism, but this is not characteristic of CDGP. Thyrotropin is most often elevated in hypothyroidism, which can cause a secondary delay in growth and puberty. Anosmia is characteristic of Kallmann syndrome, but not CDGP. Puberty is also delayed in this form of hypogonadotropic hypogonadism. Prolactin is elevated in some pituitary tumors and by dopamine-blocking agents (SOR C).

A 34-year-old white female who works as an engineer for a major corporation complains of fatigue, low energy, and a depressed mood. She states that she has felt this way for most of her life. She feels depressed most of the time but denies any recent stresses or significant losses in her life. She reports that she is doing well at work and that she recently received a promotion. She has no interests other than her job and states that she has no happy thoughts and that her self-esteem is very low. She denies suicidal thoughts but states that she does not care if she dies. She has had no sleep disturbance, change in appetite, or difficulty concentrating. She is taking no medications and denies substance abuse. Results of a recent medical evaluation required by her employer were all normal, including a physical examination, EKG, multiple chemical profile, CBC, urinalysis, and TSH level. Which one of the following is the most likely diagnosis? A) Major depression B) Dysthymic disorder C) Bipolar disorder D) Cyclothymia E) Adjustment disorder with depressed mood

ANSWER: B Dysthymic disorder is characterized by depressed mood for at least 2 years in addition to at least two of the following: change in appetite, alteration in sleep, low energy, low self-esteem, poor concentration, or feelings of hopelessness. There must be no history of a manic or hypomanic episode, substance abuse, a chronic psychotic disorder, or an organic cause. Symptoms of major depression are similar to those of dysthymia and can occasionally be difficult to distinguish from dysthymia. This patient's lifelong history of a depressed mood not triggered by any particular depressing event, and the predominance of patient complaints as opposed to objective signs, indicate that major depression is not the diagnosis in this case. Bipolar disorder is characterized by major depression with periods of mania. Cyclothymia is characterized by dysthymia with periods of hypomania. Adjustment disorder with depressed mood is characterized by impaired social or occupational functioning or abnormal symptoms within 3 months of a stressor.

A 78-year-old female has chronic symptomatic orthostatic hypotension, likely related to diabetic autonomic dysfunction, which has failed to respond to nonpharmacologic treatment. Her current medications include metformin (Glucophage), 1000 mg twice daily; atorvastatin (Lipitor), 40 mg daily; aspirin, 81 mg daily; and insulin glargine (Lantus), 24 units at bedtime. Which one of the following would be the most effective therapy for her orthostatic hypotension? A) Clonidine (Catapres) B) Midodrine C) Pseudoephedrine D) Terbutaline E) Theophylline

ANSWER: B Effective treatments for chronic orthostatic hypotension include fludrocortisone, midodrine, and physostigmine (SOR B). Clonidine, pseudoephedrine, terbutaline, and theophylline are not appropriate therapies.

In patients with systolic heart failure, which one of the following -blockers is best for reducing mortality and hospitalization rates? A) Atenolol (Tenormin) B) Carvedilol (Coreg) C) Labetalol (Trandate) D) Nebivolol (Bystolic)

ANSWER: B Evidence shows that -blockers reduce mortality and hospitalization rates for patients with systolic heart failure (SOR A). They should be started at a low dosage and increased to target dosages (SOR A). -Blockers should be considered even in patients with COPD and asthma, given their benefits. The benefit of -blockers is proportional to the degree of reduction in heart rate (SOR A). Of the listed -blockers, carvedilol has been shown to reduce the rates of death and hospitalization in heart failure patients. Other beta-blockers that have been established through randomized, controlled trials to benefit heart failure patients are bisoprolol and metoprolol succinate. The effect of nebivolol on mortality has not been adequately studied.

Which one of the following is true concerning the use of hemoglobin A1c levels to diagnose diabetes mellitus? A) A level >6.0% is diagnostic of diabetes mellitus B) Results can be misleading in patients with sickle cell disease C) The test is equally sensitive in African-Americans and whites D) The test is useful to diagnose diabetes during pregnancy

ANSWER: B Hemoglobin A1c (HbA1c) levels of 6.0%-6.5% indicate an increased risk for diabetes mellitus, and levels >6.5% can be used to diagnose diabetes. Hemoglobinopathies and conditions causing hemolysis can cause HbA1c measurements to be falsely low. The opposite effect is seen in African-Americans, who have higher HbA1c levels than whites along the continuum of glycemia. Other tests should be used in patients with conditions that affect HbA1c, including pregnancy.

A 52-year-old female sees you for the first time to establish care for her stable COPD. Since losing her insurance 4 months ago she has been off all medications except for a short-acting bronchodilator. She stopped smoking 2 years ago. She has a frequent, chronic cough and is dyspneic when climbing stairs. Pulmonary function testing reveals an FEV1 of 55%. Her O2 saturation is 90% on room air. In addition to the short-acting inhaled bronchodilator, recommended maintenance monotherapy for this patient would be either an inhaled long-acting anticholinergic agent or an inhaled A) corticosteroid B) long-acting beta-agonist C) mast-cell stabilizer D) antihistamine

ANSWER: B In 2011, the American College of Physicians published new guidelines on COPD management. For patients with COPD who are symptomatic and have an FEV1 less than 60% of predicted, the recommendation is monotherapy with either a long-acting inhaled anticholinergic (tiotropium) or a long-acting inhaled -agonist such as salmeterol or formoterol. This is in addition to rescue therapy with a short-acting inhaled bronchodilator such as albuterol. Long-acting inhaled anticholinergics and long-acting inhaled -agonists reduce exacerbations and improve quality of life. The evidence is inconclusive with regard to their effect on mortality, hospitalizations, and dyspnea. Inhaled corticosteroids have been found to be better than placebo for decreasing COPD exacerbations, but their side-effect profile keeps them from being preferred as monotherapy. Neither inhaled mast-cell stabilizers nor inhaled antihistamines are recommended as first-line agents for the treatment of COPD.

A 16-year-old male asthmatic with no other medical problems presents with a severe attack of respiratory distress and a peak expiratory flow rate <40%. After 1 hour of acute treatment his respiratory distress has resolved, but he complains of blurred vision. Which one of the following therapeutic agents would be the most likely cause? A) Nebulized albuterol (Proventil, Ventolin) B) Nebulized ipratropium bromide C) Nebulized levalbuterol (Xopenex) D) Subcutaneous terbutaline E) Intravenous methylprednisolone

ANSWER: B Ipratropium bromide is an anticholinergic agent. When nebulized it can sometimes cause inadvertent ocular effects. Blurred vision and pupil inequality may occur. Ipratropium has been shown to decrease the rate of hospital admissions in severe asthmatic attacks. Corticosteroids may elevate glucose levels in diabetic patients. The onset would be more gradual, however. Adrenergic agents used for acute asthma do not commonly produce adverse ocular effects (level of evidence 3).

A 40-year-old female with chronic asthma presents for a 6-month follow-up visit. For the past year she has done very well on fluticasone/salmeterol (Advair), 250/50 g, except for when she had to use an albuterol (Proventil, Ventolin) inhaler for 2 days because of cold symptoms 5 months ago. The most reasonable change to this patient's medication regimen would be to A) add montelukast (Singulair) B) replace fluticasone/salmeterol with fluticasone (Flovent) C) replace fluticasone/salmeterol with budesonide/formoterol (Symbicort) D) replace fluticasone/salmeterol with tiotropium (Spiriva)

ANSWER: B It is recommended that asthmatics, once stabilized, be taken off long-acting -agonists and maintained on an inhaled corticosteroid such as fluticasone. It is not recommended to change from a combination inhaled corticosteroid/long-acting -agonist to a long-acting anticholinergic agent. Montelukast can be used for maintenance, but inhaled corticosteroids are preferable.

A 3-year-old male is brought to your office by his father for evaluation of 5 days of knee pain and fever up to 101.6°F. There was no known trauma preceding these symptoms. The pain and fever respond well to oral acetaminophen but continue to recur 4 hours after each dose. On examination the child appears well and is afebrile. He had a dose of acetaminophen about 2 hours ago. There are no signs of upper respiratory infection. Examination of the knee reveals no redness, warmth, or swelling, and you see no other skin changes. He has full range of motion of both the knee and hip without pain. You note tenderness to firm palpation of the proximal tibia. He is able to bear weight and walk but refuses to jump due to anticipation of pain in his knee. Plain films of the knee are normal. The next step in the evaluation of this patient should include which one of the following? A) Close monitoring at home B) A CBC, a C-reactive protein level, and an erythrocyte sedimentation rate C) Ultrasonography of the hip D) Knee joint aspiration E) MRI of the knee

ANSWER: B Joint pain in the presence of fever with no apparent source indicates a possible infection, malignancy, or rheumatologic condition and requires further workup. Laboratory evaluation, including a CBC, a C-reactive protein level, and an erythrocyte sedimentation rate can help assess for these conditions, even though none of the tests is sufficiently sensitive to rule out these diseases, and they are not specific to a single disease entity. Knee joint aspiration would be indicated to rule out septic arthritis in the presence of a joint effusion. If the hip were painful or had decreased range of motion, then ultrasonography could help identify a hip joint effusion, which would need to be aspirated. MRI may be needed in this patient, but it would likely require sedation and thus is more invasive. Starting with laboratory work is a good first step toward identifying the source of his pain and fever.

A previously healthy 27-year-old female presents with dysuria and urinary urgency and frequency. She also complains of right flank pain, fevers and chills, and nausea without vomiting. She has a decreased appetite, but has been able to drink liquids. On examination she has a temperature of 38.4°C (101.2°F), a heart rate of 102 beats/min, and a blood pressure of 126/82 mm Hg. She has mild suprapubic tenderness and right costovertebral angle tenderness. A urinalysis shows microscopic pyuria, hematuria, and a positive leukocyte esterase test. Additional laboratory studies are notable for leukocytosis with a left shift, but are otherwise normal, including a negative pregnancy test. The patient does not have allergies to any antibiotics. Which one of the following would be most appropriate for this patient? A) Outpatient management with oral amoxicillin B) Outpatient management with oral ciprofloxacin (Cipro) C) Outpatient management with oral nitrofurantoin (Macrodantin) D) Inpatient management with intravenous ceftriaxone (Rocephin) E) Inpatient management with intravenous levofloxacin (Levaquin)

ANSWER: B Most cases of uncomplicated acute pyelonephritis, including the one described here, can be managed in the outpatient setting. Findings that might prompt consideration of inpatient management include comorbid conditions (e.g., renal dysfunction, urologic disorders, diabetes mellitus, advanced liver or cardiac disease), hemodynamic instability, male sex, metabolic derangements, pregnancy, severe pain, a toxic appearance, an inability to take liquids by mouth, or a temperature >39.4°C (103.0°F). Nitrofurantoin for 5 days is an appropriate treatment for an uncomplicated urinary tract infection, but not for pyelonephritis. Amoxicillin is generally not considered first-line treatment for pyelonephritis because of a high prevalence of resistance to oral -lactam antibiotics, and it should only be chosen if susceptibility results for the urinary isolate are known and indicate likely activity. Fluoroquinolones, such as ciprofloxacin, are the preferred empiric antibiotic treatment for outpatient treatment of pyelonephritis, as long as the local prevalence of resistance to community-acquired Escherichia coli is less than 10%.

A 52-year-old male comes to your office with a 2-month history of hoarseness that began with the onset of a head cold. His other symptoms resolved but the hoarseness has continued. He has smoked for 32 years and drinks 4 beers per day, and has had gastroesophageal reflux disease (GERD) for several years. Which one of the following would be most appropriate at this point? A) Voice rest B) Laryngoscopy C) Upper endoscopy D) Inhaled corticosteroids E) High doses of a proton pump inhibitor

ANSWER: B Patients with hoarseness lasting longer than 2 weeks with risk factors for dysplasia or carcinoma, such as smoking, heavy alcohol use, or long-standing gastroesophageal reflux disease, should be evaluated with laryngoscopy. Inhaled corticosteroids can contribute to hoarseness.

You have recently diagnosed rheumatoid arthritis in a 49-year-old female. She has started methotrexate (Rheumatrex) for disease-modifying therapy. You counsel her that she is at increased risk for various diseases related to her arthritis, but that the leading cause of death in patients with rheumatoid arthritis is A) infection B) cardiovascular disease C) lymphoma D) lung cancer

ANSWER: B Patients with rheumatoid arthritis (RA) are at increased risk for various extra-articular manifestations of the inflammatory disease, as well as side effects of the medications used to manage it. The leading cause of death in RA patients is cardiovascular, related to accelerated atherosclerosis (SOR C). Patients with RA should be screened for cardiovascular risk factors and managed appropriately to lower their risk. Patients with RA are also at increased risk for other problems that are not leading causes of mortality. Their risk for infection is increased, which can be related to either the RA itself or to the use of immunosuppressive agents. Patients with RA also have a twofold increase in their risk for lymphoma. This is independent of whether or not they are on immunosuppressive agents. Their risk for lung cancer related to interstitial lung disease is also increased, and smoking increases this risk further.

An overweight 42-year-old female complains of foot pain. Which one of the following would be most suggestive of plantar fasciitis? A) A sudden onset of ecchymosis and plantar heel pain B) Sharp, stabbing pain with palpation of the medial plantar calcaneal area C) Posterior medial ankle pain D) Burning pain in the medial plantar region

ANSWER: B Plantar fasciitis affects more than 1 million people in the United States each year. Risk factors include excessive pronation, running, obesity, and prolonged standing. Patients often have pain when they get out of bed and take their first steps in the morning, or after prolonged sitting. Palpation usually causes pain in the medial plantar calcaneal region. The pain is described as sharp and stabbing. 59 A sudden onset of ecchymosis and heel pain is more consistent with a diagnosis of plantar fascia rupture. Pain in the region of the posterior medial ankle is more consistent with posterior tibial tendinitis. Burning pain in the medial plantar region is more consistent with medial calcaneal and abductor digiti quinti nerve entrapment.

Examination of a patient with COPD and a complaint of increased shortness of breath reveals dullness to percussion in the left lower lung field, with decreased fremitus and decreased breath sounds. This is most compatible with A) pulmonary consolidation B) pleural effusion C) emphysematous bleb formation D) pneumothorax

ANSWER: B Pleural fluid is associated with a dull-to-flat percussion note, decreased-to-absent tactile fremitus, and decreased-to-absent breath sounds. A consolidation would be indicated by bronchial breath sounds and increased fremitus. Emphysematous blebs and pneumothorax are hyperresonant to percussion.

Which one of the following reflects the percentage of patients with a disease who have a positive test for the disease in question? A) Likelihood ratio B) Sensitivity C) Specificity D) Positive predictive value E) Negative predictive value

ANSWER: B Sensitivity is defined as the percentage of patients with a disease who have a positive test for the disease in question. Specificity is the percentage of patients without the disease who have a negative test. The positive predictive value is the percentage of patients with a positive or abnormal test who have the disease in question. The negative predictive value is the percentage of patients with a negative or normal test who do not have the disease in question. Likelihood ratios correspond to the clinical impression of how well a test rules in or rules out a given disease.

You are considering adding sitagliptin (Januvia) to the regimen of a patient with type 2 diabetes mellitus. Which one of the following best describes the mechanism of action of this drug? A) It increases glucagon levels B) It slows inactivation of incretin hormones C) It reduces the absorption of glucose in the gastrointestinal tract D) It reduces insulin resistance in skeletal muscle E) It reduces insulin resistance in the liver

ANSWER: B Sitagliptin is a DPP-4 inhibitor. These agents slow the inactivation of incretin hormones, prolonging their action and thereby increasing insulin release and decreasing glucagon. Sitagliptin decreases hemoglobin A1c levels by 0.7%, but there is no data on patient-oriented outcomes or long-term safety with this medication.

When stratifying patient risk to determine whether an implantable cardioverter-defibrillator is indicated for the primary prevention of sudden cardiac death, which one of the following is associated with the greatest risk? A) Atrial fibrillation B) Heart failure, with an ejection fraction less than or equal to 35% C) Uncontrolled hypertension D) Complete heart block

ANSWER: B Sudden cardiac death affects 500,000 people in the United States each year, causing more deaths than lung cancer, breast cancer, and stroke combined. The most common final pathway is ventricular tachycardia degenerating into ventricular fibrillation. The best predictor of sudden cardiac death is an ejection fraction less than or equal to 35%. Thus, it is critical for family physicians to evaluate the ejection fraction of patients with heart disease.

An 82-year-old female is hospitalized for pneumonia and sepsis. She has advance directives in place. Should it become necessary, the patient's decision-making capacity is determined by A) the spouse or next of kin B) the attending physician C) a consulting psychiatrist D) the hospital ethics committee E) a judge, at the request of hospital social services or the physician

ANSWER: B The attending physician is responsible for determining capacity and incapacity for decision making. The extent, cause, and probable duration of any incapacity should be documented in the clinical record.

Which one of the following can cause a person's normal hemoglobin level to be lower than the reference range? A) Hemochromatosis B) African-American ethnicity C) Cigarette smoking D) Living at a high altitude

ANSWER: B The range of normal hemoglobin values for healthy individuals varies with age, sex, pregnancy, smoking, altitude, and ethnicity to an extent that an adjustment derived from population-based studies is appropriate in each of these situations. A healthy individual should have their lifetime highest hemoglobin concentrations at full-term birth, exclusive of any later changes from altitude or smoking. Hemoglobin levels fall during the first 2 months of life and thereafter gradually increase until stabilizing at approximately 6 months of age. Studies of ethnic groups in the U.S. demonstrate no significant differences in normal hemoglobin values among East Asians, Hispanics, Japanese, Native Americans, and non-Hispanic whites; hemoglobin values of African-Americans tend to be 1 g/dL lower compared to the other U.S. ethnic groups studied. Living at higher altitudes has a direct effect on hemoglobin levels, and a hemoglobin reference range adjustment of +1 g/dL at 1000 meters up to +5.5 g/dL at 5000 meters is appropriate. Similarly, smoking increases hemoglobin levels by 3 g/dL for a 1 pack/day smoker to as high as +7 g/dL for individuals smoking more than 2 packs/day. The plasma volume expansion that occurs during pregnancy results in a 1.0-1.5 g/dL reduction in normal hemoglobin levels.

A 70-year-old female presents with recurrent episodes of cough, voluminous sputum, and dyspnea. She is a nonsmoker and has never smoked, except for a few cigarettes in her teens. Her past, family, and occupational histories do not suggest a cause for pulmonary or liver disease. Her examination is within normal limits except for the lung examination, which reveals crackles at both lung bases on auscultation. A chest radiograph shows nonspecific markings at both bases. The most appropriate next step in her workup would be A) a PPD skin test B) high-resolution CT C) an a1-antitrypsin level D) referral for bronchoscopy

ANSWER: B The symptoms of this patient fit the criteria for bronchiectasis, and the gold standard for diagnosis is high-resolution chest CT. The chest film does not suggest pulmonary tuberculosis, so a PPD would not be appropriate initially (although tuberculosis can be a cause of bronchiectasis). Bronchoscopy may eventually be necessary, but not at this point in the investigation. This patient's age, the absence of findings of emphysema, and the lack of a family history of emphysema or liver disease make the diagnosis of a1-antitrypsin deficiency unlikely.

A 65-year-old Asian male with a long history of cigarette smoking presents with weakness, lethargy, and mental confusion. A physical examination is normal. There are no signs of dehydration, edema, or pigmentary changes. Laboratory Findings: Serum sodium... 122 mEq/L (N 135-145) Urine osmolality... 280 mOsm/kg H2O (N 50-1400) Plasma osmolality...260 mOsm/kg H2O (N 285-295) Urine sodium... 25 mEq/L BUN...4 mg/dL (N 8-25) Serum potassium...4.1 mEq/L (N 3.5-5.0) The most likely diagnosis is A) sodium depletion B) syndrome of inappropriate secretion of antidiuretic hormone (SIADH) C) primary polydipsia D) adrenal insufficiency

ANSWER: B The syndrome of inappropriate secretion of antidiuretic hormone should be suspected in any patient who has hyponatremia and excretes urine that is hypertonic relative to plasma. A urine sodium concentration >20 mEq/L combined with a low BUN level provides further support for the diagnosis. Additional findings may include weakness, lethargy, mental confusion, and weight gain. Sodium depletion usually causes clinical features of dehydration, an elevated BUN level, and a urine sodium concentration <20 mEq/L. Primary polydipsia almost invariably results in dilute urine with low osmolality when compared to serum. Renal failure is unlikely with a BUN level of 4 mg/dL. Adrenal insufficiency is also unlikely, as most patients will have skin pigmentation, weight loss, and hypotension. A normal serum potassium level is also inconsistent with the diagnosis of adrenal insufficiency.

In an adult who has a critical illness but no history of cardiac disease, the threshold for transfusion of red blood cells should be a hemoglobin level of A) 6 g/dL B) 7 g/dL C) 8 g/dL D) 9 g/dL E) 10 g/dL

ANSWER: B The threshold for transfusion of red blood cells should be a hemoglobin level of 7 g/dL in adults and most children.

An otherwise healthy 53-year-old male presents with acute shortness of breath and pleuritic chest pain. His O2 saturation on room air is 85%, and he is hemodynamically stable. The patient underwent an appendectomy 2 weeks ago and his postoperative course was complicated by an abscess, which required a week-long hospital stay. Which one of the following should be the initial test to further evaluate this patient? A) D-dimer B) CT angiography of the chest C) Pulmonary angiography D) Venous ultrasonography

ANSWER: B This patient has a high probability of pulmonary embolism, given his clinical presentation and recent hospitalization with bed rest. Multidetector CT is the best initial test to confirm pulmonary embolism in this situation. D-dimer testing is of limited value in patients with a high probability of pulmonary embolism. If positive for deep-vein thrombosis, venous ultrasonography of the lower limbs can eliminate the need for CT or lung scans, but this occurs in only about 10% of patients. Pulmonary angiography is currently reserved for the rare case in which catheter-based treatment is indicated.

A 62-year-old female presents to your office because of painless rectal bleeding. Over the past several months she has occasionally noted blood on the toilet tissue and in her stool after bowel movements. She also reports periodic anal itching and discharge, and protrusion of rectal tissue during bowel movements that resolves spontaneously. She had a normal colonoscopy at age 50. An abdominal examination is normal and a digital rectal examination is not painful and no mass is palpated. However, her stool is positive for occult blood. Anoscopy demonstrates dilated purplish-blue veins above the dentate line. Which one of the following has the best evidence for reducing symptoms in this situation? A) Sitz baths B) Fiber supplementation C) Topical 1% hydrocortisone D) Topical diltiazem (Cardizem) E) Topical lidocaine cream (LidaMantle)

ANSWER: B This patient has grade 2 internal hemorrhoids. These protrude with defecation but reduce spontaneously. Sitz baths are commonly recommended, but a review of studies found no benefit from sitz baths for various anorectal disorders, including hemorrhoids. A meta-analysis of seven randomized trials of patients with symptomatic hemorrhoids showed that fiber supplementation with psyllium, sterculia, or unprocessed bran decreased bleeding, pain, prolapse, and itching. No randomized, controlled trials support the use of corticosteroid creams for treating hemorrhoidal disease. Topical diltiazem and topical lidocaine have been shown to provide pain relief postoperatively following excision of external hemorrhoids.

A 62-year-old female presents with painful lesions at both corners of her mouth characterized by redness, scaling, and deep cracks. The cracks sometimes bleed when she opens her mouth. She has treated them with bacitracin/neomycin/polymyxin B ointment (Neosporin) but says it has not helped. Which one of the following would be most appropriate at this point? A) A biopsy of the lesions B) An anticandidal medication C) Bacitracin D) Vitamin B12

ANSWER: B This patient has perlèche, or angular cheilitis. Most cases are secondary to moisture from patients licking their lips, promoting a monilial or staphylococcal infection. Other causes include contact and irritant dermatitis. Underlying HIV infection, celiac disease, or vitamin B12 and iron deficiencies have also been reported. Treatment may include appropriate topical creams such as mupirocin or antifungal agents, or low-potency nonfluorinated corticosteroid creams for irritant or contact causes.

Your laboratory reports a borderline low vitamin B12 level in an anemic patient. Which one of the following tests can confirm vitamin B12 deficiency? A) LDH B) Methylmalonic acid C) Mean corpuscular volume D) Serum ferritin E) Homocysteine

ANSWER: B Vitamin B12 and folate deficiencies typically cause macrocytic anemias. When the serum vitamin B12 level is borderline low, an elevated methylmalonic acid level can be used to confirm a vitamin B12 deficiency. An elevated homocysteine level plays a similar role for folate deficiency anemia. Hemolysis can be associated with an elevated LDH level, and serum ferritin is useful for diagnosing iron deficiency anemia.

An 84-year-old female presents for follow-up of multiple chronic medical problems. She is usually accompanied by her daughter, who lives nearby, but today is brought in by her son and daughter-in-law, who live out of town. They are supportive, but are insistent that the patient see a specialist for a problem that she has previously decided not to pursue further. The patient wants to avoid conflict but does not want to see any other physicians. Which one of the following is the most appropriate way to deal with this situation? A) Speak with the son and daughter-in-law privately B) Maintain neutrality and avoid triangulation C) Call the daughter to discuss the situation D) Try to talk your patient into seeing the specialist E) Schedule the appointment to appease the family, then cancel it later

ANSWER: B When family dynamics lead to conflict during an office visit, it is best for the physician to attempt to remain neutral by avoiding triangulation, which occurs when the two sides in conflict each attempt to align with a third party. Priority should be given to the patient's right to privacy and confidentiality, and the physician should ask permission from the patient to discuss his or her health issues with other people. Physicians should always remember who they are primarily responsible to.

A 19-year-old sexually active female comes to your office for a routine checkup. She is generally healthy with no chronic conditions and does not smoke. For this patient, screening for which one of the following is supported by the best evidence? A) Hypercholesterolemia B) Cervical cancer C) Chlamydia infection D) HPV infection E) Intimate partner violence

ANSWER: C According to the U.S. Preventive Services Task Force (USPSTF), there is good evidence that screening for Chlamydia infection in women who are at increased risk can reduce the incidence of pelvic inflammatory disease, while the harms are minimal. The evidence regarding screening for cervical cancer with Papanicolaou testing or human papillomavirus (HPV) testing, however, shows that the harms outweigh any possible benefits. Harms include overdiagnosis and overtreatment, including invasive cervical procedures that can affect future pregnancy outcomes. In addition, there is adequate evidence that screening women younger than 21 years of age (regardless of sexual history) does not reduce the incidence of cervical cancer or mortality compared with beginning screening at age 21. The USPSTF concludes that the evidence is insufficient to recommend for or against routine screening for lipid disorders or intimate partner violence in women this age.

A 39-year-old male with a history of alcoholism presents to your office with complaints of abdominal pain, vomiting, and nausea following a recent binge. He has eaten little since the onset of his symptoms 3 days ago. Laboratory findings suggest alcoholic ketoacidosis. His serum bicarbonate level is 16.3 mEq/L (N 22.0-26.0). In addition to thiamine, what other treatment should be provided for this patient? A) Bicarbonate and insulin B) Glucagon (GlucaGen) and hydrocortisone C) Normal saline and glucose D) N-acetylcysteine and pyridoxine (vitamin B6)

ANSWER: C Alcoholic ketoacidosis generally occurs in a patient who has been drinking heavily without eating. Blood glucose levels are usually low or normal, and volume depletion associated with nausea, vomiting, and abdominal pain is the norm. Patients typically have high osmolal and anion gaps. Treatment of alcoholic ketoacidosis includes vigorous volume repletion with normal saline, along with administration of thiamine and glucose. Only in the rare presence of marked acidemia (pH <7.10) is the administration of bicarbonate thought to be necessary. Though insulin levels may be low, hyperglycemia is seldom found. N-acetylcysteine and pyridoxine are not used for the treatment of alcoholic ketoacidosis. Levels of glucagon and hydrocortisone are typically elevated in patients with alcoholic ketoacidosis.

You are writing a prescription for amoxicillin for a 6-year-old female with acute otitis media. Her mother has had an anaphylactic reaction to penicillin in the past and is concerned that she may have passed this trait down to her daughter. You reassure her that this is not usually the case but warn her about potential signs of an allergic reaction. Which one of the following is the most concerning early symptom of a dangerous drug reaction? A) Tachycardia and elevated blood pressure B) Small, bright, erythematous macules diffusely over the trunk C) Pruritus around the mouth and on the palms of the hands and soles of the feet D) Eczematous patches in the antecubital and popliteal fossae E) Diarrhea with blood on the tissue paper

ANSWER: C Allergic reactions to medications have four primary mechanisms, referred to as Gell and Coombs classifications. The most frequent forms are type I reactions, which are immediate and mediated through IgE, and type IV reactions, which are delayed and mediated through T-cell hypersensitization. Severe type I reactions are often referred to as anaphylaxis and are the most likely to be life threatening with very little warning. Recognition of the early signs of anaphylaxis is the first step in preventing such catastrophes. 50 Anaphylactic reactions result from a massive release of histamine and start with pruritus around the mouth, on the scalp, and on the palms and soles; flushing of the face and neck, with rhinitis and conjunctivitis; angioedema of the oral mucosa, especially of the pharynx and larynx; severe urticaria; dyspnea and bronchospasm (especially in known asthmatics); and hypotension. A delay in lifesaving therapy during this phase will result in full shock, hypotension, and death. Type IV reactions usually result in benign, diffuse erythematous macules on the trunk and proximal extremities, often referred to as a drug rash. These reactions infrequently become more severe and rarely are life threatening. In severe cases the lesions become painful and palpable, and may involve blistering, mucositis, and ecchymosis.

A 40-year-old male presents with right eye pain and redness. There is no history of trauma or injury. Which one of the following should be done initially? A) Irrigation B) Funduscopic examination C) Visual acuity testing D) Fluorescein staining E) Application of a local anesthetic

ANSWER: C Almost all patients with ocular problems should have visual acuity testing before anything else is done (level of evidence 3, SOR A). If this is difficult, a local anesthetic may be applied. The main exception to this rule is a chemical burn of the eye, which should be irrigated for 30 minutes before further evaluation or treatment is undertaken..

A previously healthy 29-year-old pediatric nurse has a 3-day history of malaise, arthralgias, and a nonpruritic rash. The rash is a faint, maculopapular, irregular, reticulate exanthem that covers her thighs and the inner aspects of her upper arms. Symmetric synovitis is present in several distal and proximal interphalangeal joints and in her metacarpophalangeal joints. Small effusions, warmth, and tenderness are noted in her left wrist and right elbow. No other joints are affected. The most likely cause of this problem is A) varicella-zoster virus B) measles (rubeola) virus C) parvovirus B19 D) adenovirus E) human immunodeficiency virus (HIV)

ANSWER: C Also known as erythema infectiosum or fifth disease, parvovirus B19 infection is a fairly common cause of an exanthematous rash and arthritis in younger women. This infection should be particularly suspected in health-care workers who have frequent contact with children. The specific characteristics of the rash, the pattern of joint involvement, and the place of employment in an otherwise healthy person all offer clues suggesting parvovirus B19 as the infecting agent. Measles virus, adenovirus, and HIV rarely cause arthritis, although HIV infection can cause a musculoskeletal syndrome later in the disease. Varicella-zoster virus may cause large-joint arthritis, but the rash is distinctively vesicular and pruritic.

You see a 75-year-old male for his Medicare annual wellness visit. Which one of the following satisfies the Medicare requirement for vision screening? A) Questioning the patient about vision changes B) Use of the Amsler grid to detect age-related macular degeneration C) Use of the Snellen eye chart to evaluate visual acuity D) Use of an ophthalmoscope to detect cataracts E) Use of tonometry to detect glaucoma

ANSWER: C Although Medicare does not pay for an "annual physical," it does provide for annual preventive screening services, including a complete health history and an array of screening measures for depression, fall risk, cognitive problems, and other challenges. The physical examination conducted as part of the annual wellness visit includes measurement of blood pressure and weight, a vision check, and hearing evaluation, as well as additional elements depending on the individual's health risks. While questioning the patient or caregiver regarding perceived hearing difficulties may suffice when screening for hearing loss, screening for vision loss requires use of a standard screening tool. Documentation of visual acuity by use of the Snellen chart is an accepted means of screening for visual acuity in the primary care setting (SOR A). Vision screening will not pick up age-related macular degeneration or cataracts, however.

During hospital rounds you are called to attend the resuscitation of one of your patients, a chronic alcoholic with known esophageal varices secondary to hepatic cirrhosis, who just experienced sudden massive hematemesis that resulted in aspiration and respiratory arrest. Endotracheal intubation and suctioning appear to improve her respiratory crisis momentarily, but her pulse quickly becomes too weak to palpate. After 20 minutes of resuscitative effort, cardiac monitoring fails to detect any cardiac electrical activity, no spontaneous respiratory activity is noted, and the process is halted. When completing the death certificate for this patient, the diagnosis most appropriately listed as the immediate cause of death is A) cardiac arrest B) respiratory arrest C) upper gastrointestinal hemorrhage D) esophageal varices E) cirrhosis of the liver

ANSWER: C Although the registration of death is a state function and the details may vary based on the laws and regulations of each state, recorded data is contractually shared with the National Vital Statistics System. To ensure consistency of reporting, the National Center for Health Certificates coordinates collection of the data points by providing a standard form which most state certificates are modeled from. The standard format includes a section titled "Cause of Death," which is subdivided into two parts. In part 1, the immediate cause of death is to be recorded on the top line (labeled "a"). This is defined as the final disease, injury, or complication directly causing the death, and the directions clearly state that terminal events such as cardiac arrest, respiratory arrest, or ventricular fibrillation are not to be entered without showing the etiology. Additional lines are provided to list conditions leading to the cause of death, including a final line for entering the disease or injury that initiated the process leading to death. In this case, the proximate cause of death was the upper gastrointestinal hemorrhage. The source of the bleeding was most likely from esophageal varices resulting from hepatic cirrhosis, so those conditions should be entered respectively in the next two lines. The appropriate entry for the final line in part 1 would be chronic alcoholism. Space is provided in part 2 to include significant conditions contributing to death, such as other chronic illness and tobacco use.

An 82-year-old white male consults you following several syncopal episodes that are clearly orthostatic in nature. During the course of your evaluation you find that he has a large tongue, mild cardiomegaly, and findings suggestive of bilateral carpal tunnel syndrome. The most likely diagnosis at this time is A) pernicious anemia B) cervical spondylosis C) amyloidosis D) cardiomyopathy E) polymyalgia rheumatica

ANSWER: C Amyloidosis is defined as the extracellular deposition of the fibrous protein amyloid at one or more sites. It may remain undiagnosed for years. Features that should alert the clinician to the diagnosis of primary amyloidosis include unexplained proteinuria, peripheral neuropathy, enlargement of the tongue, cardiomegaly, intestinal malabsorption, bilateral carpal tunnel syndrome, or orthostatic hypotension. Amyloidosis occurs both as a primary idiopathic disorder and in association with other diseases such as multiple myeloma.

Treatment with which one of the following antihypertensive medications may mimic the effects of primary hyperparathyroidism? A) Amlodipine (Norvasc) B) Doxazosin (Cardura) C) Hydrochlorothiazide D) Lisinopril (Prinivil, Zestril) E) Metoprolol (Lopressor, Toprol-XL)

ANSWER: C An elevated level of parathyroid hormone (or a level that is in an unexpected "normal" range) in a patient with an elevated calcium level generally indicates a diagnosis of primary hyperparathyroidism. However, these laboratory findings may also occur with lithium or thiazide use, tertiary hyperparathyroidism associated with end-stage renal failure, or familial hypocalciuric hypercalcemia, and a medical and family history should be obtained to assess these possibilities. The other medications listed do not cause hypercalcemia.

In counseling a 35-year-old female about smoking cessation, you find that her greatest concern is that she will gain weight. Among the following therapies, which one is most strongly associated with weight gain after smoking cessation? A) Bupropion (Wellbutrin) B) Clonidine (Catapres) C) Varenicline (Chantix) D) Nicotine gum

ANSWER: C At every office visit, family physicians should encourage smokers to quit (SOR A). Patients who are ready to quit may be helped by various pharmacologic treatments. Sustained-release bupropion and nicotine replacement, especially gum and lozenges, may delay the weight gain often associated with smoking cessation. Varenicline has a variety of side effects, including an increased risk for cardiovascular events and a multitude of neuropsychiatric symptoms. It is also the agent most commonly associated with post-smoking cessation weight gain. Clonidine, considered a second-line, off-label alternative for smoking cessation, is not associated with weight gain.

According to the most recent American Diabetes Association guidelines, which one of the following groups of patients with diabetes mellitus should take aspirin for primary prevention of cardiovascular events? A) All patients B) All patients over the age of 55 C) Only those whose risk for cardiovascular disease events is >10% D) Only those whose risk for cardiovascular disease events is >20%

ANSWER: C At one point, the American Diabetes Association (ADA) recommended aspirin for all patients with diabetes mellitus. They have since revised their guidelines and advise that aspirin not be used for primary prevention of cardiovascular events unless a patient's cardiovascular risk is >10% over 10 years. The reason the ADA revised their guidelines on use of low-dose aspirin is because of the results from two studies: the Prevention of Progression of Arterial Disease and Diabetes (POPADAD) study and the Japanese Primary Prevention of Atherosclerosis with Aspirin for Diabetes (JPAD) study. The POPADAD study compared aspirin versus placebo in patients with diabetes and found that death rates from coronary heart disease and stroke were similar for the two groups, as were rates of nonfatal myocardial infarction and nonfatal stroke. The JPAD study also compared aspirin vs. placebo in diabetic patients, with similar rates of sudden death, nonfatal myocardial infarction, nonfatal stroke, unstable angina, TIA, and peripheral vascular disease. Rates of fatal myocardial infarction and fatal stroke were lower in the aspirin group.

An 80-year-old male comes to your office for evaluation of neck stiffness with a sensation of grinding when turning his head. He has had this for several years, and has now developed dull aching in his arms and numbness in his fingers. He also has noted stiffness in his legs. Examination reveals that flexion of the neck results in a sensation that the patient says feels like an electric shock going down his back. You note some wasting of the intrinsic musculature of the hands, as well as hyperreflexia. Which one of the following should you do now? A) Order EMG of both upper extremities B) Order plain films of the cervical spine C) Order MRI of the cervical spine D) Refer for cervical corticosteroid injections E) Reevaluate the patient after 4 weeks of cervical bracing

ANSWER: C Cervical spondylotic myelopathy (CSM) is the most common cause of spinal cord dysfunction in the elderly. Degenerative changes in the cervical spine, such as osteophyte formation, stiffened and hypertrophied ligamentum flava, and spinal stenosis, can result in spinal cord compression. Symptoms usually develop insidiously and may include neck stiffness, pain in the arm(s), tingling or numbness in the hands, and weakness of the hands or legs. Flexion of the neck may produce a shock-like sensation down the back, known as Lhermitte's sign. Sensory abnormalities may vary. Hyperreflexia is a characteristic physical finding. The gait may be stiff or spastic, and atrophy of the intrinsic muscles of the hands is common. CSM can be differentiated from amyotrophic lateral sclerosis (ALS) by the fasciculations and leg atrophy seen in ALS. Other conditions that produce similar findings include multiple sclerosis and masses such as a metastatic tumor. The primary diagnostic test is MRI of the cervical spine. Plain films are of little use as an initial diagnostic procedure. Electromyography is usually not helpful, although it is occasionally needed to exclude peripheral neuropathy. Nonsurgical treatment such as cervical bracing may be used in mild cases of CSM, but once a frank myelopathy occurs surgical intervention is the only option. Studies on bracing show variable results, although it is reported that symptomatic patients may deteriorate neurologically during bracing.

A 75-year-old male complains of loose stools and pain with defecation. This problem seems to have developed gradually over the last several months. His past medical history includes prostate cancer treated with radiation therapy 5 years ago, hypertension, and osteoarthritis. His medications include hydrochlorothiazide, a -blocker, and acetaminophen. Colonoscopy is negative for polyps and cancer, but the rectal and sigmoid areas show pallor with friability and telangiectasia. The most likely diagnosis is A) familial angiodysplasia B) Osler-Weber-Rendu syndrome C) radiation proctitis D) late-onset ulcerative colitis E) sensitivity to acetaminophen breakdown products

ANSWER: C Chronic radiation proctitis develops months to years after radiation exposure and is characterized by pain with defecation, along with diarrhea and sometimes rectal bleeding. On colonoscopy, the mucosa is pale and friable with telangiectases which are sometimes large, multiple, and serpiginous.

Which one of the following features in an elderly patient with cognitive problems is more suggestive of depression than of Alzheimer's disease? A) A long duration of cognitive problems B) A slow progression of cognitive problems C) Delusions that are congruent with mood D) No past history of psychiatric problems

ANSWER: C Cognitive problems are often a feature of depression in older patients, which can make it difficult to distinguish depression from dementia. Congruence of mood with delusions is more typical of depression. A longer duration of cognitive problems, slow progression of cognitive problems, and no past history of psychiatric problems are more typical of dementia.

A 32-year-old secretary complains of wrist pain at rest and when holding a pen. She says it feels like her thumb locks up at times. On examination you note tenderness on the radial side of her right wrist. A radiograph shows multiple sesamoid bones around her first metacarpophalangeal joint. You suspect A) rheumatoid arthritis B) carpal joint arthritis C) de Quervain's tenosynovitis D) cervical radiculopathy E) carpal tunnel syndrome

ANSWER: C De Quervain's tenosynovitis is a tenosynovitis of the first dorsal compartment of the wrist, specifically a chronic inflammation of the extensor pollicis brevis and abductor pollicis longus tendons. It is a fairly common cause of pain in the distal forearm. The diagnosis is made by physical examination. Pain, tenderness, and occasionally swelling are present on the radial side of the wrist. The pain is exacerbated by passive wrist ulnar deviation while the thumb is flexed and the fingers curled around it. These symptoms are not typical for rheumatoid disease, and cervical radiculopathy would not cause radial tenderness. Carpal joint arthritis would be more distal. The sesamoid bones are common and of no clinical significance. Carpal tunnel syndrome is caused by an entrapment neuropathy of the median nerve as it traverses the carpal tunnel. Symptoms typically include pain and paresthesias in the hand. Numbness occurs in the first two fingers, in the distribution of the median nerve. Thenar muscle weakness is a later sign.

For 2 weeks, a 62-year-old male with biopsy-documented cirrhosis and ascites has had diffuse abdominal discomfort, fever, and night sweats. His current medications are furosemide (Lasix) and spironolactone (Aldactone). On examination his temperature is 38.0°C (100.4°F), his blood pressure is 100/60 mm Hg, and his heart rate is 92 beats/min and regular. Examination of the heart and lungs is normal. The abdomen is soft with vague tenderness in all quadrants. There is no rebound or guarding. The presence of ascites is easily verified. Bowel sounds are quiet. The rectal examination is normal, and the stool is negative for occult blood. You perform diagnostic paracentesis and send a sample of the fluid for analysis. Which one of the following findings would best support the suspected diagnosis of spontaneous bacterial peritonitis? A) pH <7.2 B) Bloody appearance C) Neutrophil count >250/mL D) Positive cytology E) Total protein >1 g/dL

ANSWER: C Diagnostic paracentesis is recommended for patients with ascites of recent onset, as well as for those with chronic ascites who present with new clinical findings such as fever or abdominal pain. A neutrophil count >250/mL is diagnostic for peritonitis. Once peritonitis is diagnosed, antibiotic therapy should be started immediately without waiting for culture results. Bloody ascites with abnormal cytology may be seen with hepatoma but is not typical of peritonitis. The ascitic fluid pH does not become abnormal until well after the neutrophil count has risen, so it is a less reliable finding for treatment purposes. A protein level >1 g/dL is actually evidence against spontaneous bacterial peritonitis.

A 7-year-old white male is brought to your office about 5 hours after a dog bit him on the forearm. You examine the wound and decide not to suture it. His last tetanus immunization brought him up to date at 4 years of age. Which one of the following is most appropriate? A) Culture the wound B) Scrub the wound with povidone-iodine (Betadine) surgical scrub C) Irrigate the wound D) Administer tetanus immune globulin and DTaP immunization

ANSWER: C Dog and cat bite wounds may appear trivial, but if they are not managed appropriately they can become infected and may result in functional impairment. Cultures are recommended for wounds that are clinically infected. Because it can be toxic to tissue, povidone-iodine surgical scrub should not be used. Irrigation with either normal saline or Ringer's lactate solution may reduce the rate of infection by up to twentyfold. Tetanus immune globulin is not needed, and DTaP is not given to children 7 years of age or older.

Which one of the following interventions for bed-wetting in children should be recommended as initial therapy? A) Waking a child during the night and carrying him or her to the toilet B) Restriction of fluids during the day C) An enuresis alarm D) Imipramine (Tofranil) E) Oxybutynin (Ditropan)

ANSWER: C Enuresis alarms should be offered as initial treatment for bed-wetting, based on randomized, controlled trials and cost-effectiveness evidence. Desmopressin can also be considered if the child or parents do not want to try an alarm. Restriction of fluids during the day should not be recommended, as it is important that children have enough to drink. Waking a child and carrying him or her to the toilet has not been shown to have a long-term effect on bed-wetting. Oxybutynin and imipramine should only be considered in cases where bed-wetting does not respond to initial treatment.

An unconscious 22-year-old male is brought into the emergency department. His respiratory rate is 8/min, his pulse rate is 60 beats/min and regular, and his pupils are miotic. The most likely cause of his condition is A) organophosphate poisoning B) scopolamine overdose C) narcotics overdose D) benzodiazepine overdose

ANSWER: C In cases of drug overdose, several critical physical findings must be evaluated. The most important is the size of the pupils. Tolerance rarely reduces the miotic effects of narcotic medications. A patient who is comatose, with decreased breathing, a slow pulse, and small pupils should be strongly suspected of having overdosed on a narcotic. Naloxone should be administered to reverse these effects. The response to treatment with naloxone is irregular. Cerebral infarction in the pontine angle, organophosphate poisoning, phenothiazine overdose, and treatment for glaucoma can also cause constricted pupils, but these associations are seen much less frequently than narcotics overdose.

A 4-year-old female is brought to your office with a 5-day history of a cough and low-grade fever. She appears mildly anxious but in no respiratory distress, and the physical examination is notable for a temperature of 38.1°C (100.6°F), a respiratory rate of 44/min, and a spot O2 saturation of 94% with decreased breath sounds and fine crackles in the left lower lobe. You decide to prescribe amoxicillin. When you give the prescription to the mother she mentions that her 12-year-old son was given azithromycin (Zithromax) for pneumonia last year when he had similar symptoms and findings, and she asks why the children were given different antibiotics. What explanation would you give the mother for choosing amoxicillin? A) Younger children are more likely to have gram-negative pathogens B) Younger children usually have more virulent bacteria C) Younger children are less likely to have infections caused by atypical bacteria such as Mycoplasma D) The half-life of azithromycin is shortened in children younger than 5 years

ANSWER: C In preschool-age children, lower respiratory infections such as pneumonia are most commonly viral illnesses. Antibiotics may be withheld in young children who are mildly ill and are suspected of having a viral disease, but antibiotic therapy should be started if their clinical status worsens. In the preschool-age child with pneumonia, amoxicillin remains the first-line antibiotic of choice, as it provides coverage for Streptococcus pneumoniae and Haemophilus influenzae, which are the predominant bacterial causes of pneumonia in this age group. The pharmacokinetics of azithromycin do not preclude its use in children, but it is not the first-line choice for this patient. Viruses are also the most frequent cause of pneumonia in the older child, although after the age of 5 years atypical pneumonia becomes more common. This requires antibiotic coverage for organisms such as Mycoplasma. For these patients, empiric treatment with a macrolide antibiotic such as azithromycin is appropriate.

A 13-year-old male with a history of mild intermittent asthma reports escalating use of his short-acting -agonist inhaler. He routinely uses it at least 5-10 times per week for symptom relief. He has been experiencing wheezing and chest tightness with only minimal exertion, and sometimes at rest, which is a new problem for him. You recommend that he add which one of the following to his asthma medication regimen? A) Intermittent use of an inhaled long-acting -agonist B) Daily use of an inhaled long-acting -agonist C) Daily use of an inhaled corticosteroid D) Daily use of an oral corticosteroid E) Daily use of an oral immunomodulator

ANSWER: C Inhaled corticosteroids are the most potent and consistently effective long-term daily controller medications for monotherapy of mild persistent asthma (SOR A). They can be successfully used in combination with intermittent short-acting -agonists. Oral systemic corticosteroids are recommended for moderate to severe asthma exacerbations and usually for a very limited time period (SOR A). Daily long-acting -agonists are often used in combination with inhaled corticosteroids; however, long-acting -agonists are not recommended for use as daily monotherapy for long-term control of persistent asthma, or for intermittent use (SOR A). Immunomodulators such as omalizumab prevent binding of immunoglobulin E to the high-affinity receptors on basophils and mast cells. These are used as an additive therapy for patients age 12 years and older with severe persistent asthma, and are not recommended for routine use as monotherapeutic agents.

You see a 72-year-old male for follow-up after his third hospital admission for heart failure within the past 4 months. He is a widower and lives alone, but he wants to talk about options for in-home nursing care. He is accompanied today by his sister and his neighbor. This is the first time you have seen the patient. Which one of the following is the best choice regarding your interactions with the three of them? A) Construct a family genogram to determine how to proceed with the patient's care B) Speak privately with the sister and neighbor to determine possible ulterior motives C) Determine the reason each person is present today D) Discuss the patient's health information freely, as he willingly brought the neighbor and sister along

ANSWER: C Interactions between the physician and patient involve the patient's family and friends, as well as others who may be part of the patient's social support system. These interactions, as well as the physician-patient relationship itself, are also strongly influenced by ethnic, cultural, and spiritual values and by beliefs about illness and approaches to treatment and ongoing care. Involvement of family members in a patient's care is advantageous to good communication and helpful for both accurate diagnosis and appropriate treatment (SOR C). Patient confidentiality should be protected, and the mere presence of other people accompanying the patient does not automatically constitute permission to discuss private health information with them. The physician should determine not only who is present with a patient, but also each person's reason for being present (SOR C). Speaking to relatives or friends without the patient present may be appropriate under certain circumstances but would not be the best option in this scenario. Constructing a genogram can also be helpful, but that would not be the first priority in this case.

A 50-year-old female is hospitalized with severe, diffuse abdominal pain without nausea or vomiting. Vital signs on admission include a pulse rate of 110 beats/min, a respiratory rate of 35/min, and a temperature of 38.2°C (100.8°F). Laboratory findings include a WBC count of 21,000/mm3 (N 4300-10,800) with 80% segmented neutrophils, and a serum amylase level of 4000 U/L (N 53-123). CT of the abdomen is consistent with cholelithiasis and necrotizing pancreatitis without an abscess. Which one of the following measures is best supported by evidence? A) Corticosteroids B) Placement of a nasogastric tube C) Intravenous antibiotics D) Surgery if repeat CT shows development of a pseudocyst

ANSWER: C Intravenous antibiotics, especially imipenem, have been shown to be beneficial in patients with pancreatitis. Patients with pancreatitis who are not vomiting do not require nasogastric tube placement. Corticosteroids are not indicated in the management of acute pancreatitis, and pseudocysts can be managed initially with percutaneous aspiration.

Physicians are more likely than the general population to suffer from which one of the following? A) Depression B) Alcoholism C) Prescription drug abuse D) Illicit drug abuse

ANSWER: C It is thought that easier access to prescription medications leads to a higher incidence of misuse by physicians. The drugs most commonly abused are benzodiazepines and opioids. Most studies suggest that alcoholism rates among physicians approximate those of the general population when adjusted for socioeconomic status. The lifetime prevalence of depression also is similar for physicians and the general population (12.8% for men and 19.5% for women). Physicians are less likely to abuse illicit drugs, probably because of their access to prescription drugs.

A 72-year-old white male develops a rapidly growing epithelial tumor just in front of his right ear. He states that it began as a firm red papule about 6 weeks ago. It is now 1.5 cm in diameter and has a horny plug in the center. The most likely diagnosis is A) Bowen's disease B) basal cell carcinoma C) keratoacanthoma D) Kaposi's sarcoma E) seborrheic keratosis

ANSWER: C Keratoacanthoma is a relatively common lesion in the elderly, but is difficult to distinguish from squamous cell carcinoma. However, it is easily distinguished from Bowen's disease, basal cell carcinoma, Kaposi's sarcoma, and seborrheic keratosis. Most keratoacanthomas undergo a benign self-healing course but may leave a large, unsightly scar. Treatment is almost always preferred, both for cosmetic reasons and to prevent the rare case of malignant transformation. Proper treatment for a lesion with this appearance is excisional biopsy in order to distinguish between keratoacanthoma and squamous cell carcinoma.

A previously normotensive 20-year-old primigravida is admitted to the hospital at 39 weeks gestation with a headache, a blood pressure of 170/110 mm Hg, and proteinuria. Which one of the following is the most effective agent for preventing eclamptic seizure while preparing for expedited delivery? A) Benzodiazepines B) Fosphenytoin (Cerebyx) C) Magnesium sulfate D) Nimodipine

ANSWER: C Magnesium sulfate has a long history of use for preventing seizures in preeclampsia and eclampsia, and a recent Cochrane review confirmed that it is the preferred agent. Benzodiazepines and fosphenytoin are secondary agents that can be used if magnesium sulfate fails, but they are not as effective. Nimodipine was also shown to be less effective than magnesium sulfate. Delivery is indicated, but magnesium sulfate must also be administered (SOR A).

A 48-year-old female has had no menses for the past 12 months. She complains of hot flashes, especially at night, which significantly interfere with sleep. She also complains of fatigue, decreased appetite, unrefreshed sleep, and feeling "down" 4 or 5 days per week. A physical examination and laboratory findings are unremarkable. The patient prefers not to take estrogen replacement therapy. Which one of the following medications would be appropriate for this patient? A) Bupropion (Wellbutrin) B) Nortriptyline (Pamelor) C) Escitalopram (Lexapro) D) Imipramine (Tofranil)

ANSWER: C Many patients are concerned about the risks associated with estrogen replacement therapy, and alternative options should be addressed. Escitalopram has been shown to be effective for hot flashes in postmenopausal women. This would be a reasonable choice for this patient, who also likely has depression.

A 56-year-old male with type 2 diabetes mellitus has normal cardiac and renal function but has failed to achieve adequate control of his diabetes with diet and multiple oral agents. His BMI is 30.1 kg/m2 and his hemoglobin A1c level is 9.1%. Which one of the following is most likely to be beneficial in combination with insulin and diet therapy in this patient? A) Acarbose (Precose) B) Glimepiride (Amaryl) C) Metformin (Glucophage) D) Pioglitazone (Actos) E) Repaglinide (Prandin)

ANSWER: C Metformin has been found to reduce cardiovascular risk in patients with type 2 diabetes mellitus. It also decreases the risk of weight gain, and unlike some oral agents it does not significantly increase the risk of hypoglycemia. It should be continued when insulin is initiated in patients with no renal impairment (SOR B).

A full-term infant weighing 6 lb 8 oz at birth will typically weigh 20 lb at what age? A) 6 months B) 9 months C) 12 months D) 15 months E) 18 months

ANSWER: C Normal newborns may lose up to 10% of their weight following birth, and should return to their birth weight by the end of the first week of life. The steady addition of 4-7 oz of weight per week should result in a doubling of birth weight by 4-6 months of age. During the second half of the first year of life an addition of 3-5 oz/week is more the norm, resulting in a tripling of the birth weight by 1 year of age. Breastfed infants tend to gain weight more quickly during the first 6 months of life, while formula-fed infants do so from 6-12 months, with both groups having virtually equal weight gains by the end of the first year.

A 68-year-old male with type 2 diabetes mellitus has failed to control his diabetes with diet and exercise. His most recent serum creatinine level was 1.9 mg/dL (N 0.6-1.5). Which one of the following agents is most likely to cause symptomatic hypoglycemia in this patient? A) Glimepiride (Amaryl) B) Glipizide (Glucotrol) C) Glyburide (DiaBeta) D) Metformin (Glucophage) E) Repaglinide (Prandin)

ANSWER: C Older patients are at higher risk for hypoglycemia caused by oral antidiabetic agents. Glyburide is associated with a significantly greater risk of symptomatic hypoglycemia than other second-generation sulfonylurea hypoglycemic agents. Metformin decreases liver production of glucose and is not associated with hypoglycemia. Even so, this patient's creatinine elevation is a contraindication to metformin use, as it increases the risk of lactic acidosis. Glimepiride, glipizide, and repaglinide stimulate insulin release and increase the risk of hypoglycemia. However, the risk of symptomatic hypoglycemia is substantially lower compared to the risk associated with glyburide in patients with similar hemoglobin A1c values (SOR B).

A 26-year-old gravida 3 at 29 weeks gestation presents with painless vaginal bleeding. A sterile speculum examination reveals a small amount of blood in the cervix and dilation estimated at 1 cm. A sonogram shows a complete placenta previa. Her blood pressure is normal, she is not orthostatic, and her hemoglobin level is 10.7 mg/dL (N 12.0-16.0). The fetal heart rate is around 130 beats/min with good variability and no decelerations. No contractions are shown on the tocometer. In addition to hospital admission for monitoring, which one of the following would be most appropriate? A) Magnesium B) Calcium channel blockers C) Corticosteroids D) Antibiotics E) Urgent cesarean section

ANSWER: C Placenta previa is incidentally found on approximately 4% of sonograms performed between 20 and 24 weeks gestation. It often will resolve, and the incidence at term is approximately 0.4%. Symptomatic placenta previa usually manifests as painless bleeding in the late second or third trimester. It can be painful bleeding if it is associated with labor or abruption. Most patients with symptomatic placenta previa will be admitted to the hospital for evaluation. Most neonatal morbidity and mortality associated with placenta previa is due to the risks associated with preterm birth. Corticosteroids should be given to women who present with bleeding from a placenta previa between 24 and 34 weeks gestation (SOR A). Tocolytic agents such as magnesium or calcium channel blockers would be appropriate in patients who have vaginal bleeding associated with preterm contractions. The goal with tocolytic treatment would be to prolong the pregnancy until fetal lung maturity is achieved. This patient is not having preterm contractions so tocolytics would not be appropriate. The fetal heart rate is stable and the mother is hemodynamically stable, so there is no indication for an urgent cesarean section. Antibiotics do not have a role in the management of symptomatic placenta previa.

Most of the gait disturbances identified in geriatric patients in the outpatient primary care setting are related to which one of the following? A) Sensory ataxia B) Parkinson's disease C) Osteoarthritis D) Multiple strokes E) Myelopathy

ANSWER: C Problems with gait and balance increase in frequency with advancing age and are the result of a variety of individual or combined disease processes. Findings may be subtle initially, making it difficult to make an accurate diagnosis, and knowing the relative frequencies of primary causes may be useful for management. A cautious gait (broadened base, slight forward leaning of the trunk, and reduced arm swing) may be the first manifestation of many diseases, or it may just be somewhat physiologic if not excessive. In the past, a problematic gait abnormality in an elderly person was generally termed a senile gait if there was no clear diagnosis; it is more accurate, however, to describe this as an undifferentiated gait problem secondary to subclinical disease. From the long list of potential causes, arthritic joint disease is by far the most likely to be seen in the family physician's office, accounting for more than 40% of total cases. It most frequently causes an antalgic gait characterized by a reduced range of motion. The patient favors affected joints by limping or taking short, slow steps.

Which one of the following would disqualify a patient from being considered homebound, using Medicare's definition for the purpose of conducting a home visit? A) Participating in a state-licensed adult day care program B) Regularly attending religious services C) Being able to leave home with help from another person D) Being able to leave home without help, but requiring occasional use of a cane

ANSWER: D Medicare has an established definition of what constitutes a homebound patient. The definition includes patients who require the use of a cane or other supportive device in order to leave the home (not just occasional use) or require the help of another person to leave the home. Participation in a state-licensed adult day care program or regularly attending religious services does not disqualify a person from being considered confined to the home.

A 42-year-old male was admitted to the hospital yesterday for an extensive deep-vein thrombosis and treated with unfractionated heparin anticoagulation. The nurse alerts you that the patient has had multiple episodes of passing moderate amounts of maroon blood per rectum. He has a previous history of diverticulosis. He is hemodynamically stable. The initial treatment of choice for this patient is A) cryoprecipitate B) dabigatran (Pradaxa) C) protamine sulfate D) vasopressin (Pitressin) E) vitamin K

ANSWER: C Protamine sulfate is the treatment of choice for heparin overdose or significant bleeding secondary to heparin therapy. Vitamin K is used for reversal of anticoagulation from warfarin. Vasopressin is a pressor agent used to treat hypotensive episodes. Dabigatran is an anticoagulant used in nonvalvular atrial fibrillation. Cryoprecipitate is a blood product used for replacement of von Willebrand's factor, factor XIII, fibrinogen, and fibronectin.

A 24-year-old female presents to your office for a health maintenance evaluation. She mentions that she has had several episodes of indigestion after meals and started taking an over-the-counter proton pump inhibitor, which she feels has been helpful. She asks if it would be harmful to take this medicine on a long-term basis. You tell her that evidence has shown that continuing to take this medication will increase her risk for which one of the following? A) Hypomagnesemia B) Vitamin B12 deficiency C) Clostridium difficile colitis D) Having a child with birth defects (if taken in the first trimester) E) Colon cancer

ANSWER: C Proton pump inhibitors (PPIs) have a powerful effect on inhibiting the production of acid in the stomach. This dramatically reduces symptoms of acid-mediated gastritis, peptic ulcer disease, and gastroesophageal reflux. However, a significant reduction in stomach acidity may cause unintended consequences involving processes that are physiologically dependent on low pH in the gastrointestinal tract. These theoretical risks include decreased levels of vitamin B12, iron, and/or magnesium; decreased bone density; an increase in gut infections or pneumonia; an increase in gastrointestinal neoplasms; and changes in absorption of other medications. The evidence has been conflicting on some of these risks. Currently, consensus is emerging that chronic use of PPIs increases the risk for pneumonia and gut infections, primarily Clostridium difficile colitis (SOR B). PPIs may also decrease bone density in subsets of patients. These risks need to be weighed against the benefits that these medicines provide before prescribing them on a long-term basis.

Which one of the following is a proven strategy to reduce anterior cruciate ligament tears in high-school athletes? A) The use of neoprene knee sleeves by all athletes competing in high-risk sports B) Consistent inclusion of long-distance running in practice sessions C) Structured exercises stressing balance, muscle strength, and proprioception D) Prohibiting girls from playing on boys' sports teams E) Increased enforcement of penalties involving dangerous plays

ANSWER: C Several prospective trials have shown significant benefits from the use of sports-specific training of the hips and legs in preventing anterior cruciate ligament (ACL) injuries. These programs focus on plyometrics (repetitive actions that rapidly load and contract a targeted muscle group), strength training, and balance exercises, along with consistent feedback about proper landing technique. One such study was able to reduce the frequency of ACL injuries in female high-school soccer players by 88%. Although the risk of ACL injuries in female athletes is up to 10 times that of males, there is no data to show that restricting their participation in male-dominated sports is a successful strategy to prevent injuries.

Which one of the following jaundiced infants can be treated expectantly without a full workup for pathologic causes? A) A 12-hour-old term infant with a total bilirubin of 10 mg/dL B) A 1-day-old term infant with a total bilirubin of 20 mg/dL C) A 2-day-old term infant with a total bilirubin of 10 mg/dL D) A 1-week-old term infant with a total bilirubin of 25 mg/dL

ANSWER: C Term infants with an onset of jaundice before 24 hours of age, jaundice persisting beyond 3 weeks of age, or a bilirubin level requiring intensive phototherapy should not be considered healthy, and require further evaluation. A 2-day-old term infant with a total bilirubin of 10 mg/dL may be followed expectantly.

A 70-year-old male sees you because his left leg feels tender and swollen. Questioning reveals that a few days ago he returned from a long road trip with his wife, and that they had spent several days driving to visit relatives. On examination there is marked asymmetry between his left calf and his right calf; there is also a slight discoloration around the area of his left calf where it is most tender. You suspect the edema may be due to a deep-vein thrombosis (DVT). The patient has no personal or family history of blood clots. Further investigation reveals a high pretest probability score on the Wells Clinical Prediction Rule test for DVT. Which one of the following would be the most appropriate diagnostic test at this point? A) D-dimer B) Contrast venography C) Compression ultrasonography D) Helical CT E) MRI

ANSWER: C The first step in diagnosing deep-vein thrombosis (DVT) is to complete a validated clinical prediction inquiry such as the Wells Clinical Prediction Rule in order to estimate the pretest probability of DVT. The Wells criteria include such factors as active cancer, calf swelling, pitting edema, prolonged inactivity, or major surgery within the previous 12 weeks. The next step for patients with a low pretest probability of DVT is a high-sensitivity D-dimer assay, with a negative result indicating a low likelihood of DVT (SOR A). D-dimer is a degradation product of cross-linked fibrin blood clots and is usually elevated in patients with DVT, although it can also be elevated with other conditions such as recent surgery, hemorrhage, trauma, pregnancy, or cancer. If the assay is negative, the likelihood of DVT is very small. In this case, the patient has several factors listed in the Wells criteria, indicating a high pretest probability of DVT. D-dimer testing would not be useful, as the next step in this patient's evaluation should be imaging. Ultrasonography is the best test for symptomatic proximal-vein thrombosis, with a sensitivity ranging between 89% and 96% (SOR A). Although ultrasonography is the most appropriate first imaging test, contrast venography is considered the definitive test to rule out the diagnosis of DVT if there is still a high degree of suspicion after negative ultrasonography. Helical CT is commonly used to detect pulmonary embolism but is not routinely recommended to diagnose DVT. Similarly, MRI is not routinely recommended for detecting DVT.

Which one of the following has the highest sensitivity and specificity for carpal tunnel syndrome? A) A positive Phalen maneuver B) Distal tingling with percussion (Tinel's sign) C) A history of shaking the hand or flicking the wrist to alleviate nighttime pain (the flick sign) D) Thenar atrophy

ANSWER: C The flick sign has the highest sensitivity (93%) and specificity (96%) among the clinical findings of carpal tunnel syndrome. This sign is defined as a history of shaking the hand or flicking the wrist in an attempt to alleviate discomfort after being awakened with nighttime pain. Tinel's sign and the Phalen maneuver have a sensitivity of 36% and 57%, and a specificity of 75% and 58%, respectively. Thenar atrophy is usually seen in severe and chronic cases of carpal tunnel syndrome and has a sensitivity of 16% and a specificity of 90% (SOR B).

A 46-year-old African-American male presents to your office with a 1-day history of the sudden onset of severe dizziness. His symptoms include a sensation of abnormal rotation of his environment, as well as occasional headaches. He has felt nauseated but has not vomited. On examination he has resting nystagmus. There is no hearing loss, and a thorough neurologic examination is otherwise normal. He is vertiginous in all positions. Which one of the following is the most likely diagnosis? A) Basilar artery migraine with vertigo B) Benign positional vertigo C) Vestibular neuronitis D) Meniere's disease E) Eustachian tube dysfunction

ANSWER: C This patient's presentation is characteristic of vestibular neuronitis, a common condition affecting the vestibular apparatus. The exact location and cause of the derangement is uncertain, although a viral or post-viral cause has been postulated. Benign positional vertigo is characterized by brief attacks of vertigo. Meniere's disease is associated with tinnitus and hearing loss. Migraines have a more gradual onset, and the symptoms of eustachian tube dysfunction would be milder.

A 65-year-old male has complaints of insomnia and fatigue. His past medical history is significant only for hypertension and osteoarthritis. His physical examination is normal except for a blood pressure of 145/90 mm Hg and a heart rate of 105 beats/min. He has a normal BMI, does not smoke or drink alcohol, and denies any pain or chest pressure. He has not changed his daily exercise routine or diet, and has not traveled recently. Routine blood work is normal except for a TSH level of 0.3 U/mL (N 0.5-5.0). Which one of the following would be most appropriate at this point? A) Order a repeat TSH level and instruct the patient to fast beforehand B) Order a thyroglobulin level C) Order free T3 and free T4 levels D) Order a 24-hour radioactive iodine uptake test E) Begin treatment with levothyroxine (Synthroid)

ANSWER: C The low TSH level suggests hyperthyroidism. If TSH is <0.5 U/mL, the immediate next step is to measure free T3 and free T4 levels (SOR C), which are elevated in hyperthyroidism and normal in subclinical hyperthyroidism. If levels of free T3 and free T4 are elevated and the underlying cause of hyperthyroidism is unknown, then it is advisable to order a 24-hour radioactive iodine (RAI) uptake test. With Graves' disease, RAI uptake is increased and diffuse, whereas with toxic multinodular goiter it is increased and nodular (SOR A). If RAI uptake is low, subacute thyroiditis should be suspected and could be confirmed by measuring levels of thyroglobulin (SOR A). Levothyroxine is not indicated, as it is used to treat hypothyroidism. Fasting does not significantly affect TSH levels.

A 50-year-old male presents to your office with recurrent irritative voiding symptoms that are accompanied by testicular, perineal, and low back discomfort. He also reports occasional distal penile pain. Four months ago he visited another physician because of a similar episode. He was told then that he had a urinary tract infection, based on a positive urine culture that grew Escherichia coli, and was given a prescription for an antibiotic to take for 2 weeks. His symptoms improved but never completely resolved. On examination the patient is afebrile. His prostate is slightly enlarged, boggy, and moderately tender. Which one of the following is the most appropriate management step at this time? A) A prostate-specific antigen level prior to initiating treatment B) Transrectal ultrasonography of the prostate prior to initiating treatment C) Ciprofloxacin (Cipro) D) Tamsulosin (Flomax) E) High-dose oral ampicillin

ANSWER: C The patient has symptoms consistent with chronic bacterial prostatitis: irritative voiding symptoms; testicular, perineal, and low back pain; recurrent urinary tract infections; and distal penile pain. His symptoms have also been present for more than 3 months. Because chronic bacterial prostatitis is a bacterial infection, an appropriate antibiotic with good tissue penetration in the prostate should be selected. Fluoroquinolones have the best tissue concentration and are recommended as first-line agents. Penicillin derivatives, commonly used to treat acute prostatitis, have not been shown to provide good symptom relief for chronic bacterial prostatitis. -Blockers are second-line agents for treating chronic pelvic pain. Transrectal ultrasonography is indicated in patients whose acute prostatitis fails to resolve and who have a persistent fever or whose maximal temperature is not trending downward after 36 hours. In this case a prostatic abscess should be suspected, and transrectal ultrasonography can facilitate the diagnosis. Prostate-specific antigen is a screening test for prostatic malignancy and would contribute nothing to the management of this patient's problem.

You are following a 60-year-old female for osteopenia. Which one of the following tests is best for assessing her vitamin D status? A) 1,25-Dihydroxyvitamin D B) 24,25-Dihydroxyvitamin D C) 25-Hydroxyvitamin D D) Parathyroid hormone E) Phosphate

ANSWER: C The serum 25-hydroxyvitamin D level is the best indicator of overall vitamin D status because it reflects total vitamin D from dietary intake and sunlight, as well as conversion from adipose stores in the liver. Measurement of 1,25-dihydroxyvitamin D, the active form of vitamin D formed in the kidney, may be necessary in advanced chronic kidney disease. 24,25-Dihydroxyvitamin D is not biologically active. Phosphate and parathyroid hormone are involved in the regulation of vitamin D levels, but are not helpful in determining overall vitamin D status.

A 34-year-old female has a history of many years of diffuse pain, debilitating fatigue, and disrupted sleep. You suspect she may have fibromyalgia. Laboratory tests and imaging studies have been negative. She is confused about why she is in so much pain even though "everything is normal." What do you tell her is currently thought to be the etiology of fibromyalgia? A) Diffuse inflammation of soft tissues B) A chronic viral infection C) An exaggerated response to tactile stimuli by the central nervous system D) A malfunction of pain receptors in the dermis, causing excess sensitivity E) A psychological disorder in which the patient imagines the pain

ANSWER: C The understanding of fibromyalgia has been rapidly expanding in recent years, primarily due to the use of functional magnetic resonance imaging (fMRI). The hallmark of fibromyalgia is an exaggerated response to painful stimuli, or an attribution of pain to a stimulus that is normally not painful. Early research focused on peripheral tissues as the source of this condition. However, the cause has now been recognized as an abnormality in the central nervous system, which can be seen on fMRI. This physical brain abnormality differentiates fibromyalgia from psychogenic conditions such as conversion disorder and malingering. Any evidence of an actual abnormality of the peripheral tissue on physical examination, blood tests, or an imaging study in a patient suspected to have fibromyalgia should raise the suspicion that another diagnosis is also present. It is estimated that up to 25% of patients who have a definable rheumatologic condition such as lupus or rheumatoid arthritis also suffer from fibromyalgia. Many patients who develop fibromyalgia started having symptoms in the wake of a viral infection (especially Epstein-Barr virus). However, these viral illnesses are believed to trigger a genetic predisposition rather than being a necessary cause of this condition.

A 2-year-old female visiting with her parents from Mexico presents with a 1-week history of repeated episodes of forceful coughing followed by emesis. Her immunization status is unknown. Her mother reports that a runny nose and "cold" preceded the onset of the cough. The child is currently afebrile and appears mildly ill; her lungs are clear. Your management would include which one of the following? A) Hospitalization for ribavirin aerosol therapy B) Reassurance that the cough will abate over the next week C) Oral erythromycin therapy for 2 weeks D) Administration of immune serum globulin intramuscularly

ANSWER: C This child's presentation is highly suspicious for pertussis, given the severe coughing paroxysms and the possibility of inadequate immunization. Two weeks of oral erythromycin is recommended for children with mild to moderate illness, principally to halt the spread of the infection. Ribavirin is used for respiratory syncytial virus infection, which is generally seen in much younger children with more respiratory distress. The cough of pertussis often lasts several weeks. Immune globulin is not recommended.

A hospitalized 75-year-old white male with well-controlled type 2 diabetes mellitus is scheduled for abdominal CT with oral and intravenous iodinated contrast. Which one of the following medications should be withheld for at least 48 hours after the procedure? A) Acarbose (Precose) B) Glipizide (Glucotrol) C) Glyburide (Micronase, DiaBeta) D) Metformin (Glucophage) E) Rosiglitazone (Avandia)

ANSWER: D Contrast-induced nephropathy is a concern in patients undergoing contrast studies, and can lead to decreased renal function. Theoretically, this can cause an increased risk of lactic acidosis in patients taking metformin. Current guidelines recommend stopping metformin use before imaging procedures that use contrast, and restarting it 48 hours after the procedure if renal function is unchanged. The other drugs listed do not carry this risk, although they can cause other problems in hospitalized patients, such as hypoglycemia, depending on the situation.

A 53-year-old female is concerned about a skin lesion that has recently been changing in size and shape. On examination she is found to have a 7-mm, asymmetric, darkly pigmented lesion with some color variegation and irregular borders. Which one of the following skin biopsy techniques is most appropriate for confirming the diagnosis? A) A shave biopsy B) Electrodesiccation and curettage C) Elliptical excision D) Mohs surgery

ANSWER: C This lesion is suspicious for melanoma, based on the asymmetry, irregular border, color variegation, and size larger than 6 mm. In addition, a history of evolution of the lesion, with changes in size, shape, or color, has been shown in some studies to be the most specific clinical finding for melanoma. The preferred method of biopsy for any lesion suspicious for melanoma is complete elliptical excision with a small margin of normal-appearing skin. The depth of the lesion is crucial to staging and prognosis, so shave biopsies are inadequate. A punch biopsy of the most suspicious-appearing area is appropriate if the location or size of the lesion makes full excision inappropriate or impractical, but a single punch biopsy is unlikely to capture the entire malignant portion in larger lesions. Electrodesiccation and curettage is not an appropriate treatment for melanoma. Mohs surgery is sometimes used to treat melanomas, but is not used for the initial diagnosis.

A 67-year-old female hospitalized with pneumonia develops the rapid onset of dyspnea, pleuritic chest pain, tachypnea, and hypoxemia not responding to oxygen and requiring intubation. A physical examination is notable for rales throughout both lung fields with no peripheral edema noted. A chest radiograph shows bilateral pulmonary infiltrates. Her BNP level is 90 ng/L. Which one of the following is the most likely reason for her worsening clinical situation? A) Heart failure B) Hypersensitivity pneumonitis C) Acute respiratory distress syndrome D) Pulmonary embolus E) Pneumothorax

ANSWER: C This patient demonstrates classic findings for acute respiratory distress syndrome (ARDS). In many cases ARDS must be differentiated from heart failure. Heart failure is characterized by fluid overload (edema), jugular venous distention, a third heart sound, an elevated BNP level, and a salutary response to diuretics. A BNP level <100 pg/mL can help rule out heart failure (SOR A). In addition, a patient with ARDS would not have signs of left atrial hypertension and overt volume overload. Hypersensitivity pneumonitis is usually preceded by exposure to an inciting organic antigen such as bird feathers, mold, or dust. Pulmonary embolus, while certainly in the differential, is unlikely to cause such dramatic radiographic findings. Pneumothorax would be seen on the chest radiograph.

A 40-year-old male with diabetes mellitus has the following fasting lipid profile: Total cholesterol...204 mg/dL Triglycerides... 223 mg/dL Low-density lipoprotein (LDL)... 112 mg/dL High-density lipoprotein (HDL)...42 mg/dL The patient is currently on simvastatin (Zocor), 40 mg, for management of his dyslipidemia. Which one of the following would be most appropriate? A) Continuing the current medication regimen B) Increasing the dosage of simvastatin C) Switching to atorvastatin (Lipitor) D) Adding gemfibrozil (Lopid)

ANSWER: C This patient does not meet the LDL-cholesterol goals for a diabetic patient and therefore needs adjustment of his antihyperlipidemic regimen. In June 2011, the Food and Drug Administration recommended limiting the use of the highest dosage of simvastatin (80 mg/day) because of concerns about an increased risk of muscle damage. This dosage should only be used in patients who have already been taking 80 mg/day for 12 months or more without evidence of muscle injury, and it should not be started in new patients. This patient should be switched to an alternative medication that provides a greater reduction of LDL-cholesterol, such as atorvastatin. Gemfibrozil is contraindicated for use with simvastatin because it can raise simvastatin drug levels and increase the risk of myopathy.

A 20-year-old white male states that he was physically abused by his natural parents, and as a result of running away from home on several occasions he was placed in a series of foster homes. His schooling was sporadic, and he was frequently in trouble for truancy, vandalism, initiating fights, and stealing. He dropped out of school at the age of 16, and during that year he was arrested for car theft and driving while intoxicated. He has not worked at any job for more than 6 months, and has had frequent changes of address due to failure to pay rent and other financial obligations. He brags that he has fathered three children by three different women, but has not provided any support or made any contact with any of them since their pregnancies. He has used several aliases, one of which he had printed on a business card listing his occupation as "Barroom Brawler and Superstud." IQ testing is normal and there is no history of a psychotic break. The most accurate diagnosis of this patient's condition is A) borderline personality disorder B) unipolar manic disorder C) antisocial personality disorder D) abused child reaction formation E) schizotypal personality disorder with psychoactive substance abuse

ANSWER: C This patient meets the criteria for antisocial personality disorder, including age over 18, evidence of conduct disorder in childhood, a pattern of irresponsible and antisocial behavior since age 15, and absence of schizophrenia or manic episodes. Although the patient has some features of borderline personality disorder, such as unstable relationships, the persistently aggressive nature and lack of remorse are much more typical of antisocial personality. While the boasting quality of the patient might appear somewhat grandiose, there are no other features to suggest mania. Abused child reaction formation is not a recognized diagnosis in the Diagnostic and Statistical Manual of Mental Disorders. Schizotypal personality disorder is not usually associated with such pervasive antisocial behavior and violence.

A 40-year-old male presents with a 3-month history of persistent low back pain and stiffness. He cannot recall any specific episode associated with the onset of the pain, and intermittent ibuprofen has provided little benefit. The pain does not radiate into his legs. He has experienced similar back pain before, but it had always resolved within 2 weeks with rest, cutting back on his activities, and taking ibuprofen. During his third episode about a year ago MRI of his lumbosacral spine did not show any significant pathology. When you examine the patient he describes mild, generalized discomfort with palpation throughout his lumbosacral region, but has full range of motion of his back, normal deep tendon reflexes, and good muscle strength in his legs. The straight leg-raising test produces mild low back discomfort but does not result in any leg pain. Which one of the following treatment options has the best evidence for restoring function in this situation? A) Acupuncture B) Back school C) Back exercises D) Spinal manipulation E) Epidural corticosteroid injection

ANSWER: C This patient suffers from chronic low back pain, defined as pain, muscle tension, or stiffness localized below the costal margin and above the inferior gluteal folds, with or without sciatica, that has persisted for 12 weeks or more and is not attributed to a recognizable pathology. Among all the listed treatment options for chronic low back pain, only back exercises are given the "beneficial" recommendation in a systematic review. Acupuncture and spinal manipulation are in the "likely to be beneficial" category. Back school and epidural corticosteroid injections are of "unknown effectiveness."

A 26-year-old male presents to the emergency department with a fever, and he appears acutely ill. After a previously undocumented grade 3 murmur is detected on examination, a transthoracic echocardiogram is ordered and reveals a 1.5-cm vegetation on the tricuspid valve. Which one of the following is the most likely causative organism? A) Cardiobacterium hominis B) Enterococcus faecalis C) Pseudomonas aeruginosa D) Staphylococcus aureus E) Streptococcus viridans

ANSWER: D Staphylococcus aureus is the most common cause of acute infectious endocarditis worldwide. Additionally, the most common cause of tricuspid valve endocarditis is intravenous drug abuse, and Staphylococcus aureus is the infecting organism in 80% of tricuspid valve infections. Streptococcus viridans is also a frequent cause of infectious endocarditis, with Enterococcus, Pseudomonas, and Cardiobacterium being less likely causes.

To decrease stroke risk in patients undergoing coronary artery bypass grafting (CABG) who have concomitant carotid stenosis, current evidence supports which one of the following? A) Carotid endarterectomy at the same time as CABG B) Postoperative beta-blockers C) Postoperative aspirin D) Postoperative statins

ANSWER: C To address and minimize perioperative neurologic morbidity in patients undergoing coronary artery bypass grafting (CABG), individualized surgical management strategies are now recommended. These address patient risk factors for postoperative stroke, such as carotid stenosis, hypertension, older age, a past history of stroke, small-vessel disease in the brain, and diabetes mellitus. Because concomitant carotid disease is often associated with CAD, pre-CABG carotid Doppler ultrasonography is routinely recommended. Several approaches have been evaluated for decreasing the risk associated with carotid stenosis. Performing both carotid and CABG surgery at the same time increases stroke risk, and no studies have compared doing one before the other. While the use of statins has increased in patients with systemic atherosclerosis, the roles of both postoperative statins and beta-blockers are still controversial. The only treatment that has been shown to reduce postoperative cerebrovascular events is the use of aspirin in the first 48 hours after surgery.

A 52-year-old female presents with vulvovaginal dryness and pain with intercourse. She has not menstruated for 6 months and denies hot flashes, insomnia, or other vasomotor symptoms. She has no past history of cancer or surgery. Her examination is consistent with vaginal atrophy. Which one of the following is the recommended first-line treatment for this patient? A) Oral estrogen B) Oral estrogen and progestogen C) Vaginal estrogen D) Vaginal estrogen and oral progestogen

ANSWER: C Vaginal atrophy is a common symptom accompanying menopause. Local application of estrogen is the most effective treatment (SOR A) and is FDA approved for this indication. Efficacy ranges from 80% to 100%. All formulations are equally effective, so patient preference should drive the choice. Potential adverse effects of vaginal estrogen include candidal infections, bleeding, burning with application, and breast pain. It is not necessary to add an oral progestogen to the local estrogen treatment. Vaginal estrogen does not lead to endometrial proliferation, so endometrial protection in a patient who still has her uterus is not necessary.

A 16-year-old male comes to your office with a 1-month history of increasingly severe lower back pain. He plays on his school's soccer team, but denies any history of injury. Hyperextension is particularly painful, and now the pain occurs during normal daily activities. Examination reveals a hyperlordotic posture, limited range of motion, and tight hamstrings. The remainder of the examination is unremarkable. Which one of the following should be done initially? A) A complete blood profile B) Rheumatoid factor and HLA-B27 testing C) Plain radiography D) MRI E) A radionuclide bone scan

ANSWER: C While spondylolysis occurs in 6% of the general population, it may be the cause of 50% of back pain in young adults. This unilateral or bilateral vertebral defect of the pars interarticularis is likely due to repetitive hyperextension of the posterior spine that results in a fracture or stress injury. This usually occurs at L4-L5. Sports that put increased demands on the spine include football, gymnastics, weightlifting, soccer, volleyball, and ballet. 21 The recommended initial study for athletes with back pain of more than 3 weeks' duration is lumbar spine radiographs, including anterior/posterior, lateral, and oblique views bilaterally. The "Scotty dog with a collar" sign can be noted on the oblique view. This may not be present in early spondylolysis, so a SPECT scan may be appropriate. Treatment for spondylolysis consists of discontinuing the offending activity, medication for pain, physical therapy, and possibly bracing. Healing may take 9-12 months.

A 45-year-old male with type 1 diabetes mellitus receives a corticosteroid injection for osteoarthritis of the left knee. Which one of the following is true regarding monitoring of his blood glucose levels? A) Glucose levels should be closely monitored for 48 hours B) Glucose levels should be closely monitored for 7 days C) Glucose levels should be closely monitored for 14 days D) No additional monitoring is necessary

ANSWER: D A single intra-articular injection has little or no effect on glycemic control (SOR A). Soft-tissue or peritendinous injections can affect blood glucose levels for 5-21 days, however, and diabetic patients should closely monitor blood glucose levels for 2 weeks following these injections.

A 67-year-old female comes to your office to establish care after recently moving to your community. Her medical history includes hypertension, nonobstructive coronary artery disease, and heart failure, with an ejection fraction of 35% on an echocardiogram done last year. She does not have diabetes mellitus or lung disease and she has never had a myocardial infarction. She tolerates her medications well and is active, walking about 2 miles daily. Her medications include aspirin, 81 mg daily; lisinopril (Prinivil, Zestril), 40 mg daily; and simvastatin (Zocor), 40 mg daily. Which one of the following possible additions to her medication regimen has the best evidence for reducing mortality? A) Clopidogrel (Plavix) B) Ezetimibe (Zetia) C) Losartan (Cozaar) D) Metoprolol succinate (Toprol-XL) E) Spironolactone (Aldactone)

ANSWER: D According to the 2011 update of the American Heart Association/American College of Cardiology Foundation guidelines on secondary prevention of coronary artery disease, metoprolol succinate has the best evidence for mortality reduction when compared to the other medications listed. The other medications have utility, but in more specialized circumstances: losartan for those intolerant of ACE inhibitors, clopidogrel for those intolerant of aspirin, and ezetimibe for those intolerant of statins. Spironolactone has evidence of benefit post myocardial infarction when added to a regimen that includes an ACE inhibitor and a beta-blocker.

A 58-year-old female presents with significant nausea and abdominal cramping 1 month after beginning lithium carbonate. Her symptoms have not been relieved by multiple doses of antacids. A physical examination is remarkable only for the appearance of discomfort and a resting pulse rate of 92 beats/min. You suspect lithium toxicity and advise her to discontinue the lithium immediately and submit blood samples for a lithium level and complete metabolic panel. When you review her results that afternoon you find that her lithium level is in the mid-therapeutic range and the only abnormal laboratory findings are a calcium level of 13.0 mg/dL (N 8.8-10.0), an albumin level of 3.0 g/dL (N 3.8-5.0), a BUN level of 35 mg/dL (N 7-18), and a creatinine level of 1.6 mg/dL (N 0.6-1.2). Of the options listed below, the best immediate intervention is A) cinacalcet (Sensipar) B) furosemide (Lasix) C) glucocorticoids D) intravenous saline infusion E) intravenous zoledronic acid (Zometa)

ANSWER: D Drugs such as lithium, thiazide diuretics, sex hormones, and vitamins A and D can increase the serum ionized calcium level. The gastrointestinal symptoms associated with lithium toxicity are also the most common presenting symptoms of hypercalcemia. Hypercalcemic patients may also complain of constipation, fatigue, lethargy, polyuria, and weakness, all the result of an increased serum level of ionized calcium (roughly calculated to be 40% of the total serum calcium level plus 0.8 g/dL for each 1 g/dL decrease in serum albumin below 4 g/dL). The most common causes of hypercalcemia are malignancy and hyperparathyroidism, together accounting for over 80% of all cases. Excessive ingestion of antacids can result in milk-alkali syndrome, another cause of hypercalcemia. Granulomatous disease and renal diseases are other possible causes. Although additional details are required in this case to determine the cause, primary hyperparathyroidism augmented by medications is highly likely given the patient's age and sex, and measurement of her parathyroid hormone level must be included in the workup. No matter the cause, the treatment of symptomatic hypercalcemia should be immediate and directed at lowering the serum calcium level. The safest and most effective way to accomplish this is with intravenous normal saline volume replacement, reducing the need for reabsorption of salt, water, and, coincidentally, calcium in the proximal tubules. Because hypercalcemia often results in volume depletion, aggressive fluid replacement is often ideal, provided there is no contraindication to doing so. Once the volume depletion is corrected the addition of loop diuretics such as furosemide can facilitate excretion of calcium. Each of the other options has a place in the longer term treatment of hypercalcemia in appropriate situations: bisphosphonates for malignancy, glucocorticoids for granulomatous disease, and cinacalcet for hyperparathyroidism.

A 55-year-old nonsmoking male presents with a 2-month history of a nonproductive cough. He has no other respiratory tract symptoms, including dyspnea, and no history of fever. He takes hydrochlorothiazide and metoprolol succinate (Toprol-XL) for hypertension, and is otherwise healthy. A physical examination, including vital signs with pulse oximetry, is unremarkable. Which one of the following should you do next? A) Discontinue metoprolol B) Begin empiric antibiotic treatment for atypical pathogens C) Perform a complete spirometry evaluation D) Order a chest radiograph E) Order CT of the chest with contrast

ANSWER: D Adults with a chronic cough lasting 2 months or longer who are nonsmokers and are not taking ACE inhibitors should have plain radiographs to rule out specific causes prior to initiating empiric therapy (SOR C). Any treatment should be targeted to the most likely cause. The three most common causes of chronic cough in adults, other than ACE inhibitors, are gastroesophageal reflux disease, asthma, and upper airway cough syndrome. 26 Patients who are taking an ACE inhibitor should be switched to another class of antihypertensive drugs. Metoprolol does not cause a cough per se, although it may unmask preexisting asthma or COPD, resulting in a cough. -Blockers should not be discontinued abruptly, however. Formal spirometry and advanced radiographic imaging have eventual roles in the evaluation of chronic cough but are expensive tests and are not the best initial steps for evaluating a chronic cough.

Which one of the following cardiac arrhythmias is associated with antipsychotic use? A) Third degree heart block B) Paroxysmal atrial tachycardia C) Atrial fibrillation D) Ventricular fibrillation (torsades de pointes) E) Wolff-Parkinson-White syndrome

ANSWER: D All antipsychotic agents can prolong ventricular repolarization, leading to a prolonged QT interval, which can in turn lead to torsades de pointes and sudden cardiac death. Although all antipsychotics can affect EKG intervals, the agents with the greatest propensity to prolong QTc are thioridazine, pimozide, droperidol, and ziprasidone. The incidence of sudden cardiac death among patients taking antipsychotics is about twice that of the general population.

A 55-year-old male with type 1 diabetes mellitus is being treated in the wound care clinic for a skin ulcer on his lower right leg. The ulcer is slow to improve. He comes to see you in the office because over the past 2 days his right knee has become swollen, red, warm, painful, and difficult to flex. When you examine him, his right knee is swollen, erythematous, indurated, and held in full extension. Active and passive ranges of motion are limited. Areas of erythema and induration continue to surround his leg ulcer. His WBC count is mildly elevated, but his erythrocyte sedimentation rate and C-reactive protein level are normal. Which one of the following would be most appropriate at this point? A) Plain radiographs B) Ultrasonography C) MRI D) Arthrocentesis E) Antinuclear antibody studies

ANSWER: D Any patient with risk factors for infection who presents with acute joint swelling, pain, erythema, warmth, and joint immobility should be evaluated for septic arthritis. Risk factors for septic arthritis in this patient include a cutaneous ulcer and diabetes mellitus. Serum markers such as the WBC count, erythrocyte sedimentation rate, and C-reactive protein levels are often used to determine the presence of infection or inflammatory response. However, patients with confirmed septic arthritis may have normal erythrocyte sedimentation rates and C-reactive protein levels. Because the clinical presentation of septic arthritis may overlap with that of other causes of acute arthritis, arthrocentesis is needed to differentiate between the various causes and, in the case of septic arthritis, to identify the causative agent and determine appropriate therapy. No findings on imaging studies are pathognomonic for septic arthritis. Antinuclear antibody studies may be indicated later in the course of management if synovial fluid analysis is not consistent with infection, and if synovial fluid cultures are negative.

With regard to screening mammography, which one of the following is lower in women 40-49 years of age compared to women age 50 and older? A) Radiation risk B) The false-positive rate C) The false-negative rate D) The absolute risk reduction for breast cancer mortality

ANSWER: D Based on a review of randomized, controlled trials, screening mammography reduces breast cancer mortality for women between 40 and 49 years of age (SOR A). The absolute risk reduction is less for women in this age group than for older women, however. For technical reasons, and because of greater breast density, the false-positive and false-negative rates may be higher in this group than in older women. Radiation risk is also greater in younger women.

A 67-year-old female presents with the inability to smell. She is in good health, and her only medical problem is osteoporosis, treated with alendronate (Fosamax). She says she has no sinus or nasal symptoms. A physical examination is normal including an ear, nose, and throat examination. Which one of the following would be most appropriate at this point? A) Discontinuing the alendronate B) An anti-tissue transglutaminase antibody test C) A serum vitamin D level D) MRI of the brain

ANSWER: D Certain drugs can affect taste more than smell, but this does not include the bisphosphonates. Olfactory disorders may be associated with deficiencies of vitamins A, B6, B12, and trace metals, but not with vitamin D deficiency. Celiac disease is not known to cause a decreased ability to smell. Rare tumors involving the olfactory region of the brain can affect smell, and are best detected by MRI.

In neonatal resuscitation, chest compressions should begin if the heart rate drops below a threshold of A) 120 beats/min B) 100 beats/min C) 80 beats/min D) 60 beats/min E) 40 beats/min

ANSWER: D Chest compressions are recommended for a heart rate below 60 beats/min in a neonate.

In your role as team physician, you are attending a high-school basketball game. A player suffers an inversion injury of her ankle while coming down after reaching for a rebound. You examine her immediately and diagnose a grade 2 ankle sprain. Which one of the following is the most successful treatment for this injury? A) Use of a weight-bearing short leg cast for 2 weeks, then progressively increased activity B) Non-weight bearing, with crutches and an elastic wrap for 2 weeks C) Ankle taping, an elastic wrap, and partial weight bearing for 6 weeks D) Icing, a gel or air splint, and mobilization within 48 hours

ANSWER: D Conservative treatment of grade 1 and 2 ankle sprains in athletes, consisting of the use of leg casts for 2 weeks followed by progressive increases in activity, has been found to lead to a loss of playing time of 4-6 weeks. Treatment consisting of an elastic wrap and use of a crutch until pain resolves produces similar results. Early mobilization after aggressive control of inflammation is recommended. Typical treatment includes extensive icing, compression, and elevation, followed by the application of air or gel splints. In the first 48 hours, physical therapy begins with early mobilization, strengthening, and proprioception retraining. In one study utilizing this more aggressive approach, athletes were able to return to functional status in 9 days after grade 1 sprains and in 12 days after grade 2 injuries.

A 38-year-old female has spontaneous rupture of the membranes with thick meconium just prior to delivery of a male infant at 40 weeks gestation. Oropharyngeal suctioning of the infant is performed prior to delivery of the shoulders. Upon delivery the infant is noted to have spontaneous respirations, a heart rate of 120 beats/min, cyanosis of the hands and feet, and good tone. Which one of the following would be most appropriate in the immediate management of the newborn? A) Intubation and tracheal suctioning below the vocal cords B) Suctioning of the stomach to remove any swallowed meconium C) Positive pressure ventilation D) Expectant management only

ANSWER: D Endotracheal suctioning of vigorous infants born through meconium-stained amniotic fluid is not recommended (SOR C). Although infants born through thick meconium are more likely to develop aspiration syndrome, endotracheal suctioning does not provide any benefit over expectant management in preventing this condition or other respiratory problems. Endotracheal suctioning may be useful if the infant is not vigorous or shows signs of respiratory depression. Suctioning of the infant's stomach can be done electively but is not required for immediate management. Positive pressure ventilation is indicated for ventilatory support of newborns with respiratory depression who are not born through meconium-stained amniotic fluid.

A 40-year-old male presents with a new rash. On examination you note multiple erythematous-to-yellow dome-shaped papules on the extensor surfaces of his extremities, on his buttocks, and on his hands. The papules are tender and pruritic. A biopsy reveals foamy macrophages and dermal extracellular lipids. This patient's rash is associated with A) endocarditis B) systemic vasculitis C) a viral infection of the skin D) hypertriglyceridemia E) urticaria

ANSWER: D Eruptive xanthomas like these are associated with elevated triglycerides, obesity, alcohol abuse, diabetes mellitus, and estrogen or retinoid therapies. The lesions of molluscum contagiosum can be distinguished from these xanthomas by the characteristic central umbilication of molluscum.

A 45-year-old patient develops acute erythema and pain in the first metatarsophalangeal joint, the second such episode in 4 months. There is no apparent joint effusion. Results of a standard laboratory profile are normal, including an erythrocyte sedimentation rate, CBC, liver enzymes, BUN/creatinine, electrolytes, calcium, and uric acid. A radiograph is read as normal. The most likely diagnosis is A) hydroxyapatite crystal disease B) Morton's neuroma C) systemic lupus erythematosus D) acute gouty arthritis

ANSWER: D Erythema, redness, and pain in the first metatarsal-phalangeal joint are typical symptoms of gout. The uric acid level can be normal at various times in gout. Acute synovitis is occasionally caused by apatite deposition disease, but it is usually associated with long-standing osteoarthritis, and the joints involved are most commonly the shoulder, hip, and knee. Morton's neuroma is an entrapment neuropathy of the interdigital nerve, usually occurring between the third and fourth toes, not associated with erythema and redness. Acute arthritis in systemic lupus erythematosus typically involves the wrists, the small joints of the hands, and the knees.

Traveler's diarrhea can be effectively treated in the great majority of cases with which one of the following? A) Erythromycin B) Penicillin V C) Sulfacetamide D) Ciprofloxacin (Cipro)

ANSWER: D Fluoroquinolones such as ciprofloxacin have been shown to significantly reduce the duration and severity of traveler's diarrhea when given for 1-3 days. Sulfacetamide is available only in a topical form for use in the eye. Penicillin and erythromycin are not effective against the most common cause of traveler's diarrhea, enterotoxigenic Escherichia coli.

A 36-year-old male requests further testing for infertility. His female partner has undergone all testing, and her results are normal. He has recently undergone a semen analysis, which revealed azoospermia. Suspecting hypogonadism, you evaluate morning levels of FSH and total serum testosterone levels to help distinguish between primary and secondary causes. Which one of the following would you expect with primary hypogonadism? A) Normal levels of both FSH and testosterone B) Low levels of both FSH and testosterone C) Low FSH and increased testosterone D) High FSH and low testosterone E) High levels of both FSH and testosterone

ANSWER: D If semen analysis suggests hypogonadism (e.g., severe oligospermia or azoospermia), it is important to distinguish between primary and secondary causes (SOR C). Evaluation of morning FSH and total serum testosterone levels can help make this determination. Low testosterone levels correlate with hypogonadism. High levels of FSH in the presence of low testosterone levels correlate with primary hypogonadism (SOR B). Low levels of both hormones suggest secondary hypogonadism (SOR B). High testosterone levels are unlikely to be associated with hypogonadism.

An 80-year-old female who lives independently at home is admitted to the hospital with acute pyelonephritis. When she is taken to her hospital room she is incontinent, unsteady when walking, and somewhat disoriented. Her past medical history includes hypertension with evidence of diastolic dysfunction on echocardiography and asymptomatic glucose intolerance. Which one of the following orders would be appropriate for this patient? A) Telemetry B) Continuous pulse oximetry C) Foley catheter placement D) A regular diet E) Bed rest

ANSWER: D Interventions recommended for hospitalized older patients to reduce the risk of hospital-induced disability include minimizing restricted diets. Bed rest orders should be avoided, with recommendations that the patient ambulate 3-4 times/day and be out of bed and in a chair for all meals. This patient is disoriented and probably has delirium. Restraints should be avoided if possible, and should be limited if they become necessary. This would also apply to functional restraints, such as indwelling urinary catheters, IV poles, nasal cannulas, continuous pulse oximetry, and telemetry, which all increase the risk of delirium.

In patients with mild to moderate allergic rhinitis, which one of the following is the most effective treatment? A) First-generation antihistamines such as diphenhydramine (Benadryl) B) Second-generation antihistamines such as fexofenadine (Allegra) C) Montelukast (Singulair) D) Intranasal corticosteroids E) Prednisone

ANSWER: D Intranasal corticosteroids are the most effective treatment for mild to moderate allergic rhinitis and should be first-line therapy. Second-line therapies that can be used for symptoms that do not respond to initial treatment include antihistamines, decongestants, cromolyn, and leukotriene receptor antagonists. Nonpharmacologic measures that may be helpful include nasal irrigation and avoiding irritants.

A 57-year-old male with a BMI of 37.1 kg/m2 was found to have a fasting blood glucose level of 115 mg/dL on a screening test. His mother and three siblings have type 2 diabetes mellitus. A follow-up hemoglobin A1c (HbA1c) level is 6.2%. Six months later, after lifestyle interventions, the patient's BMI is 35.5 kg/m2 and his HbA1c is 6.1%. On a lipid panel, his triglyceride level is 457 mg/dL and his HDL-cholesterol level is 32 mg/dL. His serum creatinine level is 1.0 mg/dL (N 0.6-1.2). You consider the use of pharmacologic therapy. Which one of the following would be the best initial medication? A) Acarbose (Precose) B) Exenatide (Byetta) C) Glipizide (Glucotrol) D) Metformin (Glucophage) E) Pioglitazone (Actos)

ANSWER: D Metformin is well tolerated and there is good data to show it helps prevent type 2 diabetes mellitus in high-risk patients. Pioglitazone has been shown to slow the progression from prediabetes to diabetes, but it has more side effects and is more expensive than metformin. Neither glipizide nor exenatide is currently recommended as a treatment for prediabetes. Acarbose has a high discontinuation rate due to side effects.

Which one of the following outcomes is associated with hospital palliative care programs? A) Shortened hospital stays B) Reduced use of nonphysician personnel C) Increased ICU utilization D) Lower overall hospital costs

ANSWER: D More than 50% of U.S. hospitals have palliative care programs, which focus on pain and symptom management. These programs decrease both overall hospital costs and ICU use. Because palliative care requires a team approach, the number of nonphysician personnel is not decreased. The length of patient hospital stays is also not decreased (level of evidence 2, SOR A).

In addition to fluid resuscitation, which one of the following is the recommended first-line agent for the management of hypotension in a patient with sepsis? A) Albumin B) Dopamine C) Epinephrine D) Norepinephrine (Levophed) E) Phenylephrine (Neo-Synephrine)

ANSWER: D Norepinephrine is the recommended first-line vasopressor agent to correct hypotension in patients with sepsis (SOR A). Vasopressor therapy is required to sustain life and maintain perfusion in the face of life-threatening hypotension, even when hypovolemia has not yet been resolved. Maintaining a mean arterial pressure of at least 65 mm Hg is critical for tissue perfusion. Dopamine is recommended as an alternate first-line agent to elevate arterial pressure, but it is less potent compared to norepinephrine. Dobutamine is recommended as the first-line agent for managing hypotension in cardiogenic shock. Phenylephrine is recommended as the second-line agent for managing hypotension in patients with septic shock who also have tachycardia or dysrhythmias. Albumin and epinephrine are not recommended as first-line agents for managing hypotension in patients with sepsis.

A 46-year-old male admits to consuming at least 4 alcoholic drinks a day. He states that he doesn't think his alcohol use is a problem, and that he has read that drinking helps keep the heart healthy. The cardiovascular effects of this level of alcohol use include A) a decrease in blood pressure while drinking B) a decreased risk for acute coronary events C) an increased risk of valvular disease D) an increased risk of heart failure E) no apparent effect on stroke risk

ANSWER: D Observational studies have consistently shown that alcohol use has a J-shaped curve for several health effects. Small amounts of alcohol on a daily basis (less than ½-1 drink/day for women, and 1-2 drinks/day for men) are associated with an 18% lower risk for all-cause mortality and a 30% decreased risk for coronary heart disease. As the use of alcohol increases these benefits disappear and even reverse, showing a dose-dependent increase in all-cause mortality when women consume more than 2 drinks/day or men over 4 drinks/day. At this level of alcohol use, rates increase for hypertension, cancer, stroke, heart failure, dementia, and diabetes mellitus.

A 60-year-old male is started on simvastatin (Zocor) for elevated cholesterol. He mentions that he takes several herbal and dietary supplements. Which one of the following should this patient be advised to avoid? A) Ginkgo biloba B) Ginseng C) Garlic D) St. John's wort

ANSWER: D Of the common herbal supplements, St. John's wort interacts with the most drugs, including statins, warfarin, and antidepressants. The other herbal supplements listed do not interact with statins. Ginkgo biloba and ginseng may interact with warfarin.

In otherwise healthy, nonsmoking, previously unimmunized 65-year-olds, current Centers for Disease Control and Prevention recommendations for immunization with pneumococcal vaccine (Pneumovax) recommend administration of the vaccine A) yearly B) every 3 years C) every 5 years D) once E) only for patients who are immunocompromised

ANSWER: D Pneumococcal vaccine is usually given only once to individuals 65 years of age. A repeat dose may be given 5 years later for those at higher risk. Immunization is also recommended for younger persons with chronic medical problems, such as heart disease, diabetes mellitus, renal failure, and sickle cell anemia, as well as those who have undergone splenectomy or who work or live with high-risk persons.

An obese 32-year-old female has not conceived after more than 4 years of unprotected intercourse. You perform an appropriate workup and diagnose polycystic ovary syndrome. Of the following, the most effective management for her infertility would be A) spironolactone (Aldactone) B) luteinizing hormone C) basal body temperature monitoring D) clomiphene (Clomid) E) bromocriptine (Parlodel)

ANSWER: D Polycystic ovary syndrome may be the most common cause of female infertility, affecting 6%-8% of women. Some patients with polycystic ovary syndrome have hyperandrogenism, elevated levels of luteinizing hormone, and hyperinsulinemia. While early studies supported the use of metformin to increase fertility, a more recent study has shown that only 7% of women treated with metformin were able to conceive, whereas 22% of women treated with clomiphene citrate had a live birth. Spironolactone is useful for treating hirsutism, but not infertility. Since levels of other hormones are already elevated, the other measures listed would not be of benefit.

A 52-year-old African-American male sees you for a routine visit. His only medical problem is hypercholesterolemia. Because you wish to initiate a statin, you order a liver profile with the following results: Total bilirubin... 2.0 mg/dL (N 0.0-1.0) Direct bilirubin... 0.2 mg/dL (N 0.0-0.4) Albumin... 4.0 g/dL (N 3.5-5.0) LDH...250 U/L (N 45-90) AST (SGOT)... 25 U/L (N 7-27) ALT (SGPT)...15 U/L (N 1-21) Alkaline phosphatase... 25 U/L (N 13-39) Which one of the following would best explain these results? A) Alcoholic hepatitis B) Steatohepatitis C) Chronic hepatitis C D) Hemolysis E) Gilbert's syndrome

ANSWER: D The combination of elevated LDH and elevated unconjugated bilirubin with otherwise normal liver enzyme levels suggests hemolysis. Gilbert's syndrome would not explain the LDH elevation. Hepatitis is unlikely with normal transaminase levels.

A 32-year-old female presents with complaints of moderate irritability and anxiety during the week before nearly all of her menstrual periods. During this time she also has problems with weight gain and breast tenderness. She says she is her "usual happy self" at other times during the month. You diagnose premenstrual syndrome (PMS). Which one of the following complementary and alternative therapies has been shown to be helpful in reducing the symptoms of this problem? A) Saffron B) St. John's wort C) Soy D) Pyridoxine (vitamin B6) E) Vitamin E

ANSWER: D Premenstrual syndrome (PMS) may be diagnosed when recurrent psychological and physical symptoms occur only during the week prior to menses. The presence of more severe affective and somatic symptoms that cause significant dysfunction in a patient's social and work life is more consistent with premenstrual dysphoric disorder. Both pyridoxine (vitamin B6), 50-100 mg/day, and chasteberry, 20 mg/day, have been shown in randomized, controlled trials to reduce the symptoms of PMS compared with placebo. No good evidence supports the use of vitamin E, saffron, St. John's wort, or soy.

You are managing a patient who is critically ill and is being placed on a ventilator. Management should include which one of the following to reduce the occurrence of ventilator-associated pneumonia? A) Continuous sedation B) Nasotracheal intubation C) Metoclopramide (Reglan) prophylaxis D) Elevation of the head of the bed E) Initiation of tube feedings within 24 hours

ANSWER: D Recommendations for reducing ventilator-associated pneumonia (VAP) include elevation of the head to an angle of at least 30°, noninvasive mechanical ventilation rather than intubation when appropriate, oral intubation when an endotracheal tube is necessary, orogastric rather than nasogastric tubes, minimization of sedation, administration of a proton pump inhibitor when prophylaxis is indicated, changing ventilator tubing every 7 days or when it becomes soiled, avoidance or elimination of endotracheal tube leaks, good technique in removal of condensate, and excellent hand hygiene. One study favored waiting more than 5 days before initiating tube feedings, as this reduced the incidence of VAP, although further data is needed to confirm this.

Which one of the following is most likely to cause hypoglycemia when used as monotherapy? A) Acarbose (Precose) B) Exenatide (Byetta) C) Pioglitazone (Actos) D) Repaglinide (Prandin) E) Sitagliptin (Januvia)

ANSWER: D Repaglinide is a non-sulfonylurea agent that interacts with a different portion of the sulfonylurea receptor to stimulate insulin secretion. It has a relatively short duration of action, and while it may cause hypoglycemia this is less likely than with a sulfonylurea agent. Pioglitazone reduces insulin resistance and has no hypoglycemic effect. Acarbose delays absorption of carbohydrates such as starch, sucrose, and maltose, but does not affect the absorption of glucose and other monosaccharides. Sitagliptin inhibits the enzyme responsible for the breakdown of the naturally occurring incretins, and its major advantage is the absence of side effects. Exenatide stimulates insulin secretion in a glucose-dependent fashion, inhibits glucagon secretion, slows gastric emptying, and may have a central satiety effect. It does not cause hypoglycemia when used as monotherapy, but may increase the risk when used with an insulin secretagogue such as glyburide or glipizide.

A 2-year-old male is brought to your office for a well child check. He was born with pectus excavatum, which has progressed somewhat as he has grown. You and the parents are concerned about the potential for abnormal cardiopulmonary function and body image issues as the child grows. Repair of mild to moderately symptomatic pectus excavatum ideally should be considered when the patient is A) a toddler B) preschool age C) in elementary school D) an adolescent

ANSWER: D Repair of symptomatic pectus excavatum should be postponed until adolescence, if possible, as this approach allows for completion of growth and reduces the chance of recurrence. Younger children with severe cardiopulmonary problems may also be candidates for surgery, but repair at too early an age can result in improper growth of the chest wall and increases the risk of recurrence of the deformity. Adult repair is also feasible.

An 8-year-old male is brought to the emergency department with an acute asthma attack that began 48 hours earlier. His mother initiated his asthma action plan when the attack began, starting oral prednisolone plus albuterol (Proventil, Ventolin) by metered-dose inhaler with a spacer every 3-4 hours. In the emergency department the child is alert, with a respiratory rate of 30 beats/min and an oxygen saturation of 94% on room air. He is audibly wheezing. Peak flow is 40% of the predicted value. Which one of the following should you do next? A) Continue the current albuterol treatment but switch to a nebulizer B) Administer high-dose albuterol via nebulizer every 20 minutes for 1 hour C) Administer intravenous corticosteroids within the first hour D) Administer magnesium sulfate intravenously E) Prescribe high-dose mucolytics and chest physiotherapy

ANSWER: D Repeated doses of a short-acting 2-agonist and correction of hypoxia are the main elements of initial emergency department treatment for acute asthma exacerbations in children. Nebulizer treatments are no better than a metered-dose inhaler with a spacer (SOR A). High-dose nebulized albuterol every 20 minutes for 1 hour has not been shown to be beneficial. In children already receiving standard treatment with albuterol and corticosteroids the addition of intravenous magnesium sulfate has been shown to improve lung function and reduce the need for hospitalization (SOR A). Oral administration of corticosteroids is as effective as the intravenous route for reducing the need for hospital admission (SOR A). Mucolytics and chest physiotherapy have not been shown to be effective in children with acute asthma attacks. Ref: Pollart SM, Compton RM, Elward KS: Management of acute asthma exacerbations. Am Fam Physician 2011;84(

Which one of the following is most consistent with the rash of infantile seborrheic dermatitis? A) Sparing of the diaper area B) An onset after 6 months of age C) Pruritus at the time of onset D) Resolution within weeks to months E) Progression to atopic dermatitis

ANSWER: D Seborrheic dermatitis is one of the more common skin conditions affecting infants within the first few months of life. The characteristic reddish, waxy rash most commonly involves the scalp (cradle cap), but can also appear on the face, ears, neck, skin folds, and diaper area. While the rash is similar to that of atopic dermatitis, seborrheic dermatitis is not associated with pruritus. Although the rash can appear alarming to parents, reassurance that the condition can be expected to resolve within a few months is the most appropriate management.

While vacationing, a 27-year-old white male was exposed to poison ivy. Between 48 and 72 hours after exposure he developed a pruritic, erythematous, papulovesicular eruption on his arms and neck. He began treating himself with an over-the-counter topical hydrocortisone cream, and when the eruption did not improve after 24 hours of treatment he sought help from the local emergency department. He was given oral methylprednisolone (Medrol Dosepak), starting with 24 mg/day and tapered by 4 mg/day over 6 days. His condition began to improve, but on day 6 he noted a dramatic exacerbation of the eruption with intense pruritus, erythema, and vesiculation, involving extensive areas of his arms, neck, and face. The most appropriate management at this time would be to A) prescribe a superpotent topical corticosteroid B) repeat the oral methylprednisolone treatment C) begin diphenhydramine (Benadryl), 4 times a day D) begin high-dose oral prednisone and taper over 2 weeks E) discontinue all medications and recommend cool compresses

ANSWER: D Systemic corticosteroids are recognized for their dramatic impact on both the subjective and objective course of poison ivy dermatitis. Oral prednisone at an initial dosage of 1 mg/kg/day tapered over 14-21 days is the standard regimen. Complications can result from the use of shorter prepackaged courses of corticosteroid therapy, resulting in significant rebound flares. These products usually begin with an initial dosage approximately half that of the recommended dosage, with the course tapering too rapidly. Over-the-counter topical hydrocortisone is ineffective for all but the mildest cases, and once the disease is established, superpotent topical corticosteroids do little to alter the overall course and natural history. Antihistamines and compresses provide some symptomatic relief, but do little to alter the course of established disease.

While on vacation you get up for an early morning swim and find a young man face down in the hotel lap pool. He is flaccid and unresponsive when you pull him from the water. You are alone, there is no automated external defibrillator (AED) available, and the telephone is at the opposite side of the room. Which one of the following actions is most consistent with American Heart Association 2010 guidelines for resuscitation? A) Turning the victim on his side to drain upper airway fluid before starting CPR B) Performing the Heimlich maneuver before starting CPR C) Performing 10 minutes of CPR before activating emergency medical services D) Attempting ventilation before chest compression E) Maintaining cervical spine immobilization with whatever is available

ANSWER: D The 2010 American Heart Association guidelines for resuscitation emphasize the importance of chest compression in CPR. Compression-Airway-Breathing (C-A-B) is now recommended over Airway- Breathing-Compression (A-B-C). Individualization of this sequence is recommended, however, and in drowning victims the A-B-C approach is preferred because of the hypoxic nature of the cardiac arrest. Emergency medical services (EMS) should be activated when the victim is found flaccid and unresponsive. In certain situations CPR may be performed for up to 2 minutes before calling 911, but a 10-minute interval is excessive. The Heimlich maneuver and attempts to positionally drain the airway may be harmful and delay effective CPR. The reported incidence of cervical spine injury in drowning victims is 0.009%. Attempts at cervical spine immobilization are not necessary and may impede airway maintenance (SOR C).

A patient has fatigue and joint pain and is concerned about the possibility of systemic lupus erythematosus (SLE) after reading about this condition on the Internet. After taking a brief history you decide that further evaluation is appropriate. In addition to the history and physical findings, which one of the following laboratory findings would most support the diagnosis of SLE? A) An abnormal C-reactive protein level B) An erythrocyte sedimentation rate of 48 mm/hr C) A positive antimicrosomal antibody test D) A positive test for antiphospholipid antibodies E) A positive test for rheumatoid factor

ANSWER: D The American College of Rheumatology criteria for the diagnosis of systemic lupus erythematosus (SLE) includes the presence of 4 of 11 criteria. One criterion is evidence of the presence of an immunologic disorder. Evidence of an immunologic disorder includes a positive finding of antiphospholipid antibodies, based upon one of the following: an abnormal serum level of immunoglobulin G or M anticardiolipin antibodies, a positive lupus anticoagulant test, or a false-positive serologic test for syphilis. The other test results listed are not criteria for the diagnosis of SLE.

Which one of the following is a contraindication to the use of combined hormonal contraceptives? A) A family history of breast cancer in a first degree relative B) Rheumatoid arthritis treated with immunosuppression C) Morbid obesity D) Migraine headaches with aura E) Ovarian cancer

ANSWER: D The U.S. Medical Eligibility Criteria for Contraceptive Use were created to guide health care providers in assessing the safety of contraceptive use for patients with specific conditions. Category 1 includes conditions for which no restrictions exist for use of the contraceptive method. Category 2 indicates that the method generally can be used, but careful follow-up may be required. Category 3 is used to classify conditions for which the method usually is not recommended unless more-preferred methods are not available or acceptable. Category 4 comprises conditions that represent an unacceptable health risk if the method is used. For combined hormonal contraceptives, migraine headaches with aura at any age are classified as category 4 because of the increased risk of ischemic stroke. A family history of breast cancer is category 1, rheumatoid arthritis treated by immunosuppression is category 2, a BMI 30 kg/m2 is category 2, and ovarian cancer is category 1.

Which one of the following statements is consistent with current U.S. Preventive Services Task Force recommendations for skin cancer screening for the adult general population with no history of premalignant or malignant lesions? A) Whole-body examination should be conducted by a primary care provider every 3 years B) Whole-body patient self-examination should be performed every 6 months C) Benefits from screening have been established only for high-risk patients D) The evidence is currently insufficient to determine whether early detection reduces mortality and morbidity from skin cancer E) The harms of detection and early treatment outweigh the benefits

ANSWER: D The U.S. Preventive Services Task Force has concluded that current evidence is insufficient to assess the balance of benefits and harms of whole-body skin examination by a primary care physician or by patient skin self-examination for the early detection of cutaneous melanoma, basal cell cancer, or squamous cell skin cancer in the adult general population. Due to the lack of studies, the evidence is insufficient to determine whether early detection of skin cancer reduces mortality or morbidity from skin cancer. The same is true regarding the magnitude of harms from screening for skin cancer. Benefits from screening are uncertain, even in high-risk patients.

A 17-year-old female comes to your office with an 8-month history of amenorrhea. Menarche occurred at age 12 and her menses were regular until the past year. The patient's vital signs are in the normal range for her age except for a BMI of 16.1 kg/m2 (below the third percentile for age). She is a high-school senior who dances with the local ballet company. She practices dance several hours a day and works out regularly. She admits that she follows a strict 800-calorie/day diet to keep in shape for ballet. You order a CBC, a complete metabolic profile, a urine beta-hCG level, FSH and LH levels, and a TSH level. Which one of the following is also recommended as part of the workup? A) An EKG B) Pelvic ultrasonography C) Abdominal/pelvic CT D) A DXA scan E) A nuclear bone scan

ANSWER: D The female athlete triad is a relatively common condition in athletes, and is characterized by amenorrhea, disordered eating, and osteoporosis. It is more common in sports that promote lean body mass. Female athletes should be screened for the disorder during their preparticipation evaluations. Individuals who present with one or more components of the triad should be evaluated for the other components. This patient evidences disordered eating (low BMI for age) and secondary amenorrhea, and should be screened for osteoporosis using a DXA scan. The International Society for Clinical Densitometry recommends using the Z-score, rather than the T-score, when screening children or premenopausal women. The T-score is based on a comparison to a young adult at peak bone density, whereas the Z-score uses a comparison to persons of the same age as the patient. A Z-score less than -2.0 indicates osteoporosis. The American College of Sports Medicine defines low bone density as a Z score of -1.0 to -2.0. An EKG is not required in this patient since she has normal vital signs. Pelvic ultrasonography is not necessary unless an abnormal finding is identified on a pelvic examination. Abdominopelvic CT would be inappropriate given the patient's age and lack of abdominopelvic symptoms such as pain or a mass. A nuclear bone scan likewise is not recommended, as it is not used to diagnose osteoporosis (SOR C).

A 22-year-old female has a 4-month history of suprapubic pain, urinary frequency, urinary urgency, dysuria, and dyspareunia. She has been empirically treated with antibiotics for a urinary tract infection despite the fact that multiple urine tests have been negative for infection or other abnormalities. You suspect the patient has interstitial cystitis. Which one of the following would be most appropriate at this point? A) Fluoxetine (Prozac) B) Ibuprofen C) Nitrofurantoin (Macrobid) D) Pentosan polysulfate sodium (Elmiron) E) Trimethoprim/sulfamethoxazole (Bactrim, Septra)

ANSWER: D The only FDA-approved oral medication for the treatment of interstitial cystitis is pentosan polysulfate sodium, which is thought to repair the urothelium (SOR B). Trimethoprim/sulfamethoxazole and nitrofurantoin are indicated for urinary tract infections (UTIs), but usually not in cases of cystitis with no infection. In addition, this patient has already received empiric treatment for a UTI despite having multiple negative urine cultures. Ibuprofen is an anti-inflammatory medication commonly used to treat pain but is not specifically indicated for interstitial cystitis. While tricyclic antidepressants such as amitriptyline have been used to treat interstitial cystitis, fluoxetine is not generally recommended.

A 25-year-old previously healthy female presents to the urgent care clinic with swelling of her lips and tongue, wheezing, dyspnea, and urticaria that developed after she was stung by a wasp. Her only medication is atenolol (Tenormin), which she takes for migraine prophylaxis. You immediately administer epinephrine 1:1000 dilution subcutaneously, but the patient does not improve even after two more injections 10 minutes apart. She continues to be hypotensive despite administration of an intravenous normal saline bolus, intramuscular diphenhydramine, and nebulized albuterol (Proventil, Ventolin). Which one of the following intravenous medications is most appropriate for treating this patient's hypotension? A) Aminophylline B) Diphenhydramine C) Epinephrine D) Glucagon (GlucaGen) E) Hydrocortisone

ANSWER: D The response to epinephrine may be limited in patients with anaphylaxis who have been taking -blockers. Such individuals may have persistent hypotension, bradycardia, and prolonged symptoms. Since glucagon exerts positive inotropic and chronotropic effects on the heart without depending on catecholamines, an intravenous bolus followed by an infusion would be a good choice to treat the refractory hypotension. The use of corticosteroids in this setting is common, but their effectiveness has not been established. Their benefit is not realized for at least 6 hours, however, so they may aid in the prevention of recurrent anaphylaxis. Diphenhydramine sometimes provides dramatic symptom relief, but it would not improve the hypotension. beta-Agonists such as albuterol and aminophylline can be used for bronchospasm, but are not helpful for hypotension.

A 67-year-old female has a bone density study that indicates a T score of -3.5. You prescribe alendronate (Fosamax) but at her next visit she says she cannot tolerate the side effects and asks about other therapies. Which one of the following has the best evidence for prevention of both vertebral fractures and hip fractures? A) Calcitonin-salmon (Miacalcin) B) Raloxifene (Evista) C) Teriparatide (Forteo) D) Zoledronic acid (Reclast)

ANSWER: D There are a number of alternatives to the bisphosphonates. Unfortunately, efficacy data is not encouraging for most of them. Intravenous zoledronic acid has been shown to reduce both hip fracture risk and vertebral fracture risk. Teriparatide reduces vertebral fracture risk but not hip fracture risk. The same is true for raloxifene and calcitonin salmon.

Which one of the following has good evidence of effectively improving borderline personality disorder? A) SSRIs B) Second-generation antipsychotics C) Omega-3 fatty acids D) No currently available pharmacotherapy

ANSWER: D There are no proven therapies to reduce the severity of borderline personality disorder (SOR A). The most promising psychological therapy is dialectic behavioral therapy (DBT). DBT is a multi-faceted program specifically designed to treat borderline personality disorder. The few, small studies of DBT found improvement in many symptoms of borderline personality disorder, but long-term data is lacking. Another promising therapy is psychoanalytic-oriented day hospital therapy. Again, study sizes have been small and data cannot be extrapolated to the population as a whole. Omega-3 fatty acids, second-generation antipsychotics, and mood stabilizers have been shown to be helpful for some symptoms of borderline personality disorder but not for overall severity. Their benefits are based on single-study results and side effects were not addressed in the studies. SSRIs are not recommended for borderline personality disorder unless there is a concomitant mood disorder.

For most patients, which one of the following is the most effective treatment for anemia of chronic disease? A) Elemental iron B) Erythropoietin C) Prednisone D) Optimal management of the underlying disorder E) Combined therapy with oral iron, vitamin B12, folic acid, and erythropoietin

ANSWER: D There is no specific therapy for anemia of chronic disease except to manage or treat the underlying disorder. Iron therapy is of no benefit, but erythropoietin may be helpful in some patients. There is no available data to suggest that combination therapy or prednisone is beneficial for this disorder.

A 68-year-old female with a previous history of multiple medical problems presents to your office with dizziness. She describes this dizziness as an "off-balance" or "wobbly" feeling. She has not had a sensation of spinning or motion, or loss of consciousness. She is not anxious or depressed. She takes the following medications: Lovastatin (Mevacor), 20 mg daily for hypercholesterolemia Metoprolol succinate (Toprol-XL), 25 mg daily Chlorthalidone, 12.5 mg daily Lisinopril (Prinivil, Zestril), 20 mg daily for hypertension Sertraline (Zoloft), 25 mg daily for menopausal symptoms Alendronate (Fosamax), 70 mg weekly Calcium, 600 mg 2 times daily Vitamin D, 1000 units daily for osteoporosis Oxybutynin (Ditropan XL), 10 mg daily for overactive bladder Acetaminophen, 1000 mg 2 times daily for osteoarthritis Meclizine (Antivert, Bonine), 25 mg 3 times daily as needed for dizziness Cyclobenzaprine (Flexeril), 5 mg 3 times daily as needed for muscle spasm Zolpidem (Ambien), 5 mg at bedtime as needed A physical examination is normal, including a neurologic examination, and the patient has a normal gait. There is no evidence of peripheral neuropathy, and Romberg testing is normal. There is no orthostatic decrease in blood pressure. The Dix-Hallpike maneuver is negative. A CBC, chemistry profile (CMP), TSH level, and vitamin B12 level are normal. Which one of the following would be most appropriate at this point? A) A 24-hour heart monitor B) A tilt table test C) Carotid ultrasonography D) Medication reduction E) Increasing the dosage of sertraline to 50 mg daily

ANSWER: D This patient has a disequilibrium type of dizziness. Causes of this include medication side effects, Parkinson's disease, and peripheral neuropathy. In this patient the history and examination do not indicate a specific cause. She is on several medications, and one or more could be contributing to her symptoms. A trial of medication reduction should be considered before ordering additional studies.

A concerned father brings his 20-month-old daughter to see you because of the overnight onset of a "barky" cough along with hoarseness, a runny nose, and a fever to 100.8°F. She is an otherwise healthy child who is up to date on her vaccinations. During the course of the visit you observe her to be coughing intermittently, and on examination you note clear lungs with occasional stridor and no retractions. She is not tachypneic or tachycardic. Her oxygen saturation is 95% on room air. Appropriate medical management of this patient includes which one of the following? A) Inhaled albuterol (Proventil, Ventolin) B) Inhaled epinephrine C) Oral azithromycin (Zithromax) D) Oral dexamethasone E) Oral oseltamivir (Tamiflu)

ANSWER: D This patient has mild acute croup, also known as laryngotracheobronchitis. She has no signs of pneumonia, epiglottitis, or bacterial tracheitis. Acute croup is almost always viral in nature, with parainfluenza viruses being the most common etiologic agents. A single dose of oral dexamethasone has been shown to benefit children with even mild croup, presumably by decreasing edema of the laryngeal mucosa. Inhaled epinephrine is helpful in severe croup with signs of respiratory distress. Inhaled albuterol is used to treat asthma. Oseltamivir would be appropriate treatment for influenza, and azithromycin for bacterial pneumonia.

A 72-year-old female presents with a 2-month history of constipation. She says she has to strain to evacuate at least half the time and reports that her stools have become clay-like in consistency and narrower in caliber. At least half the time she has the sensation that evacuation is not complete, and she has occasionally used manual maneuvers to complete evacuation. She had a normal colonoscopy 8 years ago. An abdominal examination is normal, and stool with a clay-like consistency is palpated during a rectal examination. No prolapse is seen with straining, and the anal wink is present. Screening laboratory tests indicate a mild microcytic, hypochromic anemia. Which one of the following would be most appropriate at this time? A) A trial of lactulose B) Lifestyle modifications C) Phosphosoda enemas D) Colonoscopy E) Pelvic floor muscle exercises

ANSWER: D This patient has several red flags that require complete colon evaluation with endoscopy: age >50, a change in stool caliber, and obstructive symptoms. Other red flags include heme-positive stools, anemia consistent with iron deficiency, and rectal bleeding. Malignancy should be eliminated as a possible diagnosis prior to initiating any treatment. Biofeedback training is used to manage pelvic floor dysfunction caused by incoordination of pelvic floor muscles during attempted evacuation. Common symptoms include prolonged or excessive straining, soft stools that are difficult to pass, and rectal discomfort. The other options are appropriate management strategies once malignancy has been eliminated as a possibility.

A 49-year-old female with type 1 diabetes mellitus presents to your office with a 1-week history of a red, hot, swollen foot. She recalls twisting her ankle when stepping off a curb the day before the swelling began. She denies fever or significant pain. She has difficulty walking due to stiffness in the foot. On examination you find the patient is in no distress. Her temperature is 36.7°C (98.1°F), blood pressure 144/82 mm Hg, and heart rate 80 beats/min. Her right foot is edematous, erythematous, and excessively warm. Monofilament testing reveals significant impairment of sensation of both feet. She has restricted range of motion of the right ankle and foot compared to the left. No skin lesions are present. Dorsalis pedis pulses are brisk and symmetric. A radiograph of the right foot is normal. Laboratory Findings Hemoglobin A1c ... 8.2% Capillary blood glucose...213 mg/dL Hematocrit... 37.2% (N 36.0-46.0) WBCs... 11,000/mm3 (N 4300-10,800) Platelets...350,000/mm3 (N 150,000-350,000) Erythrocyte sedimentation rate... 30 mm/hr (N 1-25) Which one of the following is the most likely diagnosis? A) Osteomyelitis B) Osteoarthritis C) Acute gout D) Charcot foot

ANSWER: D This patient most likely has Charcot foot (neuropathic arthropathy). This is an uncommon condition, most often found in patients with at least a 10-year history of type 1 or 2 diabetes. Patients have peripheral neuropathy and typically present with painless swelling of a foot. About 50% of patients can recall minor trauma preceding the onset of symptoms. Early radiologic findings may be normal, but as the condition progresses plain films may reveal bony fractures, fragmentation, and bone destruction. On examination the foot is either stiff or hypermobile, warm, erythematous, and edematous. Crepitus can be felt in later stages of the disease. Laboratory results can be normal or show increases in the WBC count. Any patient with long-standing diabetes mellitus and a warm, red, swollen foot should be presumed to have Charcot foot and have the foot immobilized immediately to protect it from stress and to prevent further destruction and disability (SOR C). 57 The differential diagnosis of Charcot foot includes osteomyelitis, which can be difficult to distinguish. Patients with osteomyelitis can have skin ulcers that may be probed to the bone. They will often be febrile, with higher erythrocyte sedimentation rates and WBC counts than patients with Charcot foot. Aspiration of the joint fluid is helpful in distinguishing between the two conditions. Aspiration can also help distinguish between acute gout and Charcot foot. Patients may have pain and stiffness in their foot with either condition. Patients with gout may have an elevated uric acid level, and plain films may reveal tophi. Patients with osteoarthritis are unlikely to have warm, erythematous skin overlying the involved joint. Symptoms often have an insidious onset rather than the more acute onset of Charcot foot.

A 58-year-old postmenopausal female sees you for an initial health maintenance visit. Her examination is normal and she has no complaints. You perform a Papanicolaou (Pap) test, which she has not had done in 15 years. The smear is read as "negative for intraepithelial lesion and malignancy, benign endometrial cells present." What would be the most appropriate follow-up for this finding? A) A repeat Pap test in 4-6 months B) A repeat Pap test in 1 year C) HPV testing D) An endometrial biopsy E) Colposcopy and endocervical curettage

ANSWER: D This patient should have an endometrial biopsy (SOR C). Approximately 7% of postmenopausal women with benign endometrial cells on a Papanicolaou smear will have significant endometrial pathology. None of the other options listed evaluate the endometrium for pathology. An asymptomatic premenopausal woman with benign endometrial cells would not need an endometrial evaluation because underlying endometrial pathology is rare in this group.

Patients with which one of the following platelet disorders should be hospitalized and treated emergently? A) Drug-induced thrombocytopenia B) Congenital thrombocytopenia C) Gestational thrombocytopenia D) Thrombotic thrombocytopenic purpura E) Thrombocytopenia associated with Lyme disease

ANSWER: D Thrombotic thrombocytopenic purpura (TTP) is an emergent condition that can result in up to 30% mortality. Prompt hospitalization with plasma exchange is the preferred treatment. Patients with TTP present with nonspecific symptoms such as fever, abdominal pain, nausea, and weakness. Patients may also exhibit neurologic deficits. Microangiopathic anemia is also likely to be present, as evidenced by schistocytes on a peripheral smear and elevated levels of LDH and nucleated RBCs. Congenital thrombocytopenia is a benign condition in which patients have long-standing low platelet counts and/or a family history of thrombocytopenia. It is usually asymptomatic but a concomitant bleeding diathesis may occur. Gestational thrombocytopenia is also benign and asymptomatic. It is often confused with mild immune thrombocytopenic purpura. Platelet counts rarely drop below 70,000/mm3. There is no associated fetal thrombocytopenia. Preeclampsia and HELLP syndrome should also be ruled out. Platelet counts return to normal after delivery (SOR C). Drug-induced thrombocytopenia can be severe, but platelet counts do not usually drop below 20,000/mm3. It is characterized by an abrupt drop in the platelet count within a week of starting the offending medication and resolves within 2 weeks after the medication is stopped. Lyme disease can be associated with a transient thrombocytopenia. Patients present with common symptoms of Lyme disease, such as fever, myalgias, and rash. The thrombocytopenia resolves with treatment of the underlying infection.

Tourette's syndrome is associated with which one of the following comorbidities? A) Cardiac arrhythmias B) Partial or complex seizures C) Hypertension D) Attention-deficit disorder E) Hypothyroidism

ANSWER: D Tourette's syndrome is often associated with psychiatric comorbidities, mainly attention-deficit/hyperactivity disorder and obsessive-compulsive disorder. The other conditions listed are not associated with Tourette's syndrome.

You suspect mild Pneumocystis jiroveci pneumonia in a patient whose past medical history is significant for HIV infection. He has not been compliant with his disease management and is not on any medications. What is the recommended treatment for this patient's pneumonia? A) Azithromycin (Zithromax) B) Clindamycin (Cleocin) C) Rifampin (Rifadin) D) Trimethoprim/sulfamethoxazole (Bactrim, Septra)

ANSWER: D Trimethoprim/sulfamethoxazole is the drug of choice for treating Pneumocystis jiroveci pneumonia. Atovaquone has been shown to be very effective for treating mild to moderate Pneumocystis jiroveci pneumonia and is also very well tolerated, and would be the first choice for a patient with a sulfa allergy. Clindamycin plus primaquine is also effective therapy, but clindamycin is not effective as monotherapy. Azithromycin and rifampin are not effective against this organism. Other treatment options include pentamidine, dapsone plus trimethoprim, and clindamycin plus primaquine.

You see a 78-year-old male in the hospital the day after his hip-replacement surgery. He has not voided in the past 12 hours. A urethral catheter is placed and 500 mL of urine is removed from his bladder. Which one of the following is most likely to improve the success rate of a voiding trial? A) Using a specialized catheter coudé instead of a standard catheter B) Leaving the catheter in place for at least 2 weeks C) Immediately removing the catheter to prevent a urinary tract infection D) Starting tamsulosin (Flomax), 0.4 mg daily, at the time of catheter insertion E) Starting antibiotic prophylaxis at the time of catheter insertion

ANSWER: D Urinary retention is a common problem in hospitalized patients, especially following certain types of surgery. Starting an -blocker at the time of insertion of the urethral catheter has been shown to increase the success of a voiding trial (SOR A). Voiding trial success rates have not been shown to be improved by leaving the catheter in for 2 weeks, immediate removal of the catheter, using a specialized catheter, or antibiotic prophylaxis.

A previously healthy 24-year-old female presents with a 10-day history of facial pain and fever. On examination she has tenderness over the maxillary sinus on the left. Which one of the following would be most appropriate for treatment of this patient's condition? A) Intranasal saline flushes B) Intranasal antihistamines C) Oral antihistamines D) Oral antibiotics E) Reassurance only

ANSWER: D While there are several guidelines for the clinical diagnosis of acute bacterial sinusitis (ABS), there is general agreement that patients with a duration of symptoms of at least 10 days without improvement should be treated with antibiotics, including both children and adults (SOR C). Signs and symptoms may include nasal drainage and congestion, facial pressure and/or pain, sinus tenderness, and headache. Recommendations for the duration of treatment vary. One set of guidelines calls for empiric treatment with amoxicillin alone; another recommends going directly to amoxicillin/clavulanate. Suggested alternatives include a "respiratory" quinolone or the combination of a third-generation cephalosporin and clindamycin, particularly in patients with penicillin allergy. Due to the increasing emergence of resistant Streptococcus and Haemophilus species, neither trimethoprim/sulfamethoxazole nor macrolides are now recommended for empiric treatment of ABS. Data regarding the efficacy of other measures such as nasal irrigation and the use of decongestants is limited and variable. The most recent guidelines do not recommend the use of decongestants, whether oral or topical.

A neurologically intact 77-year-old female presents with severe low back pain following a fall 2 days ago that caused her to land on her buttocks. A radiograph of her lower spine shows a compression fracture of L3 with a loss of about 50% of the vertebral body height. Which one of the following is most appropriate at this point? A) Referral for kyphoplasty B) Referral for vertebroplasty C) Back bracing D) Bed rest E) Calcitonin-salmon (Miacalcin)

ANSWER: E A number of measures for managing spinal compression fractures have been evaluated. The evidence for recommending kyphoplasty is weak, and the evidence for recommending against vertebroplasty is strong. The data on bracing is inconclusive, as is the recommendation for bed rest. Calcitonin has been shown to reduce the incidence of recurrent fractures and may be useful in the relief of pain.

Which one of the following is a risk factor for depression during pregnancy? A) High socioeconomic status B) Nonsmoking C) Age over 25 D) A family history of hyperthyroidism E) Childhood abuse

ANSWER: E A previous history of depression is the strongest risk factor for depression during pregnancy. Other risk factors include childhood abuse, smoking, age under 20, and low socioeconomic status, especially without social support. A family history of hyperthyroidism is not a risk factor.

You are preparing to evaluate a patient in the emergency department. A BNP level was ordered by the physician from the previous shift who handed the patient over to you. The level is reported as 459 pg/mL. You have not yet interviewed or examined the patient. Based upon the information you have at this point, which one of the following is true regarding this patient? A) The patient has diastolic heart failure B) The patient has systolic heart failure C) The patient has acute heart failure D) The patient does not have heart failure E) The patient's diagnosis is uncertain

ANSWER: E According to the 2010 American Heart Association scientific statement regarding acute heart failure syndrome, levels of natriuretic peptides such as BNP lack the specificity necessary to function as absolute indicators of acute heart failure syndrome even when they exceed established thresholds for the diagnosis. BNP levels vary with age, sex, body habitus, renal function, and abruptness of symptom onset. Elevated BNP levels also have been associated with renal failure (because of reduced clearance), pulmonary embolism, pulmonary hypertension, and chronic hypoxia. BNP measures are not a substitute for a comprehensive assessment for signs and symptoms of heart failure, and a laboratory test by itself cannot be used to determine the diagnosis or management of heart failure. Clinical evaluation and follow-up are essential to assure proper care for patients with heart failure or any other cardiac problem.

For a 1-year-old patient, pneumococcal 13-valent conjugate vaccine (Prevnar) is preferred rather than polyvalent pneumococcal vaccine (Pneumovax) because of which one of the following advantages? A) It is available in an oral form B) It is less expensive C) It requires only one dose D) It can be combined with MMR in a single injection E) It is more immunogenic

ANSWER: E Pneumococcal 13-valent vaccine produces a satisfactory immune response in 1-year-old children, while polyvalent vaccine does not cause a good antibody response in children under the age of 2 years. Neither vaccine is available orally, and cost is not a factor. The 13-valent vaccine requires multiple doses. The vaccine cannot be combined with MMR in a single injection, but can be administered concurrently with routine childhood immunizations at a separate site using a separate syringe.

A healthy 47-year-old female presents with a 3-day history of moderately severe low back pain after attempting to lift a heavy container of potting soil in her garden. She has no history of back problems. Her pain is in the right lower back with radiation to the buttock. She denies urinary or bowel incontinence, urinary retention, and numbness or tingling. A physical examination confirms low back muscular strain. Which one of the following interventions has been shown to be beneficial in this situation? A) Bed rest B) Massage therapy C) Lumbar traction D) Prednisone E) Cyclobenzaprine (Flexeril)

ANSWER: E Acute low back pain is one of the most common presenting symptoms in family medicine practices. In the absence of red flags such as fever, a history of cancer, or neurologic deficits, patients can be successfully treated with conservative therapy. Interventions that have been shown to be beneficial include non-benzodiazepine muscle relaxers (SOR A). They are most effective in the first 1-2 weeks but can be used for up to 4 weeks. Additional beneficial treatments include physical therapy, acetaminophen, and NSAIDs. Bed rest is inadvisable for patients with low back pain (SOR A). Patients who stay active have better outcomes than those who stay at rest. There is no good evidence that oral corticosteroids are beneficial for acute back pain, and insufficient evidence that massage therapy is effective. Lumbar traction provides no benefit in acute low back pain (SOR B).

A 66-year-old white female presents to your office for a routine physical examination. Her medical problems include hypertension, diabetes mellitus, hypercholesterolemia, and gastroesophageal reflux, all controlled with medications. A bone density study is consistent with osteopenia. She is taking a multivitamin and calcium carbonate, 1200 mg daily. Which one of her medications would reduce her calcium carbonate absorption? A) Atorvastatin (Lipitor) B) Hydrochlorothiazide C) Lisinopril (Prinivil, Zestril) D) Metformin (Glucophage) E) Omeprazole (Prilosec)

ANSWER: E Because of the high prevalence of reduced gastric acidity related to either endogenous causes or medications such as proton pump inhibitors, calcium carbonate is best taken with meals to optimize absorption. Calcium citrate, which is well absorbed regardless of gastric acidity, may be taken with or without food. The other medications listed do not impair calcium absorption.

A 45-year-old male presents to your office for a health maintenance visit. Other than mild fatigue, which he attributes to long hours at work and lack of exercise, he has no complaints. He is married, and says he takes no routine medications and does not smoke or drink. His examination is unremarkable except for a BMI of 32.3 kg/m2. A CBC is unremarkable, but a fasting metabolic profile shows a glucose level of 115 mg/dL, an AST (SGOT) level of 100 U/L (N 5-40), and an ALT (SGPT) level of 112 U/L (N 7-56). The remainder of the profile is normal. Which one of the following is the most likely cause of the abnormal laboratory findings? A) Acetaminophen toxicity B) Hepatitis B or C C) Herbal preparations containing kava D) Alcohol use E) Nonalcoholic fatty liver disease

ANSWER: E Currently, nonalcoholic fatty liver disease is the leading cause of transaminase elevations, and is becoming increasingly common as obesity becomes more prevalent. It is estimated that some 30% of adults in the United States have this disease. Patients with metabolic syndrome, diabetes mellitus, or elevated triglycerides are at the highest risk. If the AST/ALT ratio is >2, especially if -glutamyl transpeptidase is elevated, alcoholic liver disease should be suspected. It is well known that severe hepatotoxicity can occur with acetaminophen overdoses, and dosages of even 4 g/day for 5-10 days will cause enzyme elevations in more than half of healthy nondrinkers. Herbal preparations associated with elevated liver enzymes include kava and germander. Hepatitis C can cause transient enzyme elevations, typically of ALT. If liver enzymes remain elevated on a repeat test 2-4 weeks later, the patient should be tested for hepatitis B and C, and iron, iron binding capacity, and ferritin levels should be ordered to check for hemochromatosis. A lipid profile and glucose level should be ordered as well, and abdominal ultrasonography considered to look for evidence of fatty infiltration of the liver.

A 50-year-old male presents to your office with erythroderma and fever. He has not had a sore throat, rhinorrhea, cough, or urinary tract symptoms. His current medications include lisinopril (Prinivil, Zestril), atenolol (Tenormin), and allopurinol (Zyloprim). On examination he has a blood pressure of 110/90 mm Hg, a pulse rate of 90 beats/min, and a temperature of 38.6°C (101.5°F). The skin is remarkable for marked erythema over 90% of the body, with tenderness to touch. His mental status is clear and his neck is supple. Mildly tender adenopathy is noted in the neck, axillae, and groin. He has no oral ulcerations or ocular symptoms. A CBC shows a WBC count of 15,000/mm3 (N 4300-10,800) with 20% eosinophils. A metabolic profile shows an AST (SGOT) level of 100 U/L (N 10-40) and an ALT (SGPT) level of 110 U/L (N 10-55), but is otherwise normal. Which one of the following is the most likely diagnosis? A) Stevens-Johnson syndrome B) Erysipelas C) Red man syndrome D) Toxic shock syndrome E) Drug reaction with eosinophilia and systemic symptoms (DRESS syndrome)

ANSWER: E DRESS is an acronym for Drug Reaction with Eosinophilia and Systemic Symptoms. The hallmark of DRESS syndrome is erythroderma accompanied by fever, lymphadenopathy, elevation of liver enzymes, and eosinophilia. The offending medication should be discontinued immediately and treatment with corticosteroids should be initiated. Seizure medications such as carbamazepine, phenytoin, lamotrigine, and phenobarbital are responsible for approximately one-third of cases. Allopurinol-associated DRESS syndrome has the highest mortality rate. Toxic shock syndrome should be suspected in patients with erythroderma, hypotension, and laboratory evidence of end-organ involvement (elevated liver enzymes or kidney function studies, anemia, thrombocytopenia, or elevation of creatine kinase). Treatment with intravenous clindamycin, which inhibits toxin synthesis, should be undertaken immediately. Stevens-Johnson syndrome is characterized by a vesiculobullous rash with mucocutaneous involvement, and erysipelas is a painful localized rash with well-demarcated borders. Red man syndrome is associated with vancomycin.

A 40-year-old male complains of a cough that has persisted for more than 3 months. He is otherwise asymptomatic. A chest radiograph and pulmonary function tests are normal. Which one of the following is the most likely cause? A) Bronchiectasis B) Tuberculosis C) Sarcoidosis D) Asthma E) Gastroesophageal reflux disease

ANSWER: E Gastroesophageal reflux disease is one of the most common causes of chronic cough. Patients with "silent" gastroesophageal reflux may not have the classic symptoms of heartburn and regurgitation. The diagnosis is based on resolution of the cough with an empiric trial of a proton pump inhibitor, although a chest radiograph should be obtained in all patients with a chronic cough to exclude bronchiectasis, tuberculosis, and sarcoidosis. Asthma is another frequent cause of chronic cough, but it can be ruled out with normal pulmonary function tests.

A 70-year-old male sees you for a preoperative evaluation 3 days prior to iliofemoral bypass surgery. He has a 54-pack-year history of cigarette smoking, and has a long-term history of hypertension and peripheral vascular disease. His current medications include lisinopril (Prinivil, Zestril), hydrochlorothiazide, and low-dose aspirin. He has no past history of myocardial infarction, diabetes mellitus, or hyperlipidemia. His blood pressure is 156/84 mm Hg and his pulse rate is 80 beats/min. The cardiopulmonary examination is normal. Foot pulses are diminished but present bilaterally. In order to reduce this patient's risk of perioperative cardiac complications, which one of the following is recommended prior to his surgery? A) A pharmacologic cardiac stress test B) Discontinuation of aspirin C) Starting a beta-blocker D) Starting enoxaparin (Lovenox) E) Starting a statin

ANSWER: E In addition to their lipid-lowering effects, statins have been shown to have plaque-stabilizing and vascular anti-inflammatory effects. There is strong clinical evidence that perioperative statin therapy, even when initiated within days of the procedure and without regard to lipid levels, significantly reduces cardiovascular risk for patients undergoing vascular surgery (SOR A). There is strong clinical evidence of benefit in perioperative cardiovascular risk reduction for continuation of -blockers before, during, and after vascular surgery in patients who have been on them for at least 4 weeks preoperatively (SOR A). However, in patients who have not been on a beta-blocker for at least 1-4 weeks preoperatively, initiation prior to surgery may be harmful (SOR B). Traditionally, aspirin has been discontinued prior to surgery for fear of increased surgical bleeding complications. However, studies have shown that in most cases it is safe to continue low-dose aspirin in the perioperative period, and doing so reduces cardiovascular complications. This is especially true for patients with a past history of myocardial infarction or with coronary stents (SOR B). Preoperative cardiac stress testing is of little value in patients with low or medium cardiovascular risk status, such as the patient described here. Enoxaparin would not be indicated preoperatively in this patient.

Of the following cardiovascular parameters, which one increases with normal aging? A) Maximum heart rate B) Heart rate variability C) Left ventricular ejection fraction D) Arterial wall elasticity E) Blood pressure

ANSWER: E It can be difficult to determine the point at which changes of normal aging are more appropriately considered disease processes. Although the direction of expected change is generally well understood, variables such as the level of fitness and overall health of an individual affect the degree of change. As the body ages, the measured left ventricular ejection fraction, heart rate variability, and maximum heart rate trend downward, the walls of the major aorta and major arteries stiffen, and the vasodilator capacity of most smaller vessels is reduced (SOR A). The arterial wall changes increase peripheral resistance and result in an increase in blood pressure. Positive adaptive changes have been shown in older adults who engage in regular aerobic exercise, however, and these changes can be measured after only 3 months of moderate-intensity exercise (SOR A).

A 23-year-old male comes to your office accompanied by his girlfriend to talk about attention-deficit disorder. He minimizes the concerns she raises, which include sleeping less (sometimes just 2-3 hours a night), rambling on tangentially during conversations, and being highly irritable. When you ask him about these observations, he agrees that they are true and reflect a change in his usual behavior. However, he explains that he is just becoming more social and that his girlfriend is probably jealous of his new popularity. The patient has no family history of attention-deficit disorder. His father died at a young age as a result of alcoholism. He denies stimulant use and a urine drug screen is negative. Which one of the following mental disorders is most likely in this patient? A) Attention-deficit disorder B) Attention-deficit/hyperactivity disorder C) Generalized anxiety disorder D) Major depressive disorder E) Bipolar disorder

ANSWER: E It is estimated that about one-third of patients with bipolar disorder seek medical care within a year of the onset of symptoms, but that nearly 70% do not receive an accurate diagnosis. The symptoms can often be subtle and may be attributed to other causes by patients or their loved ones. A diagnosis of attention-deficit disorder requires that a patient's symptoms be present since early childhood, although they are sometimes not recognized at the time. This patient and his girlfriend have both acknowledged that he is not his usual self. He presents with increased self-esteem, a decreased need for sleep, pressured/tangential speech, and irritability, which point to the possibility of a manic or hypomanic episode. Together these symptoms suggest bipolar disease (SOR C). Patients with full-blown mania are often out of touch with reality and easy to identify. However, patients with hypomania consider themselves to have increased well-being and productivity, and will not always seek attention or consider themselves to have a problem. Other symptoms that should alert the physician to this diagnosis include substance abuse (present in over 70% of cases) and involvement in other pleasurable but destructive activities such as overspending or hypersexuality. If substance abuse is present, however, it must be addressed before making a diagnosis of bipolar disorder. Bipolar disorder is highly genetic, and asking about affected first degree family members can often assist in making the diagnosis.

A 46-year-old African-American female sees you because of a history of excessive uterine bleeding and irregularity in her menstrual cycle. She has three children and had a tubal ligation after her last delivery. A pelvic examination does not reveal any pathology to explain her symptoms. Further laboratory evaluation indicates that she is mildly anemic. You perform an endometrial biopsy in the office that confirms your suspicion of endometrial hyperplasia without atypia. Which one of the following is the treatment of choice for this patient? A) Elective hysterectomy B) Hysteroscopic endometrial laser ablation C) High-dose oral estrogen supplementation D) Antifibrinolytic therapy E) Progestational drugs

ANSWER: E Medical therapy with progestational drugs is the treatment of choice for menorrhagia due to endometrial hyperplasia without atypia. Progestins convert the proliferative endometrium to a secretory one, causing withdrawal bleeding and the regression of hyperplasia. The most commonly used form is cyclic oral medroxyprogesterone, given 14 days per month, but implanted intrauterine levonorgestrel is the most effective (SOR A) and also provides contraception. High-dose estrogen supplementation would further stimulate the endometrium. Estrogen is useful in cases where minimal estrogen stimulation is associated with breakthrough bleeding. The anti-fibrinolytic agent tranexamic acid prevents the activation of plasminogen and is given at the beginning of the cycle to decrease bleeding. Side effects and cost limit this treatment option, however. It may be most useful in women with bleeding disorders or with contraindications to hormonal therapy. NSAIDs, which decrease prostaglandin levels, reduce menstrual bleeding but not as effectively as progestins. While mefenamic acid is marketed for menstrual cramps and bleeding, all NSAIDs have a similar effect in this regard. If medical management fails, hysteroscopic endometrial ablation is an option for reducing uterine bleeding but is considered permanent and obviously will impair fertility. Hysterectomy is reserved for severe and chronic bleeding that is not relieved by other measures.

A 63-year-old white male has been diagnosed with myasthenia gravis and is experiencing progressive muscle weakness despite maximum pharmacotherapy. Which one of the following surgical options would be most likely to improve his condition? A) Thyroidectomy B) Radioactive thyroid ablation C) Adrenalectomy D) Removal of a pituitary microadenoma E) Thymectomy

ANSWER: E Myasthenia gravis is a neuromuscular illness with an underlying immune-related cause. Corticosteroids and anticholinesterase medications such as oral pyridostigmine can be helpful, but thymectomy may be appropriate for patients with generalized disease not responding to medication. Thymectomy increases the remission rate and improves the clinical course.

A new first-time mother calls for advice on nipple pain with breastfeeding. She is 6 days post partum after an uncomplicated delivery. Which one of the following would be most effective? A) Lanolin cream B) Expressed breast milk C) Tea bag compresses D) Hydrogel dressing E) Education on positioning

ANSWER: E Nipple pain with breastfeeding is extremely common, with some studies reporting a prevalence of up to 96%. Preventing or alleviating nipple pain is important for comfort, but also for promoting breastfeeding in general. The best intervention for nipple pain is education on proper positioning and attachment of the infant. Topical remedies may also be effective, although no one topical agent has been shown to be clearly superior, and none is as effective as education on positioning and latch-on.

A clinical trial reports that a new therapy is non-inferior to your usual choice of treatment. You can assume which one of the following? A) The new treatment has proven efficacy B) A large placebo group was studied C) The study was both double blind and placebo controlled D) The new therapy is not superior to what you are currently using E) The new therapy is not less effective than what you are currently using

ANSWER: E Noninferiority trials compare an active control group with a new therapy. The use of a placebo group would be unethical, since the present therapy is either lifesaving or prevents serious injury. The new therapy may prove superior to or slightly less effective than the standard therapy.

A 45-year-old male recently recovered from a second episode of left lower extremity cellulitis. He has onychomycosis on his left foot but is otherwise in good health. Which one of the following treatments is best overall if eradication of the onychomycosis is necessary? A) Ciclopirox topical (Penlac Nail Lacquer) B) Oral fluconazole (Diflucan) C) Oral griseofulvin (Grifulvin V) D) Oral itraconazole (Sporanox) E) Oral terbinafine (Lamisil)

ANSWER: E Onychomycosis is a difficult condition to treat successfully. If symptoms are minimal, treatment is often deferred. Cellulitis of the involved extremity may be related to the onychomycosis and is an accepted reason to consider eradication treatment. Oral terbinafine is the best treatment in terms of cure rate and tolerability (SOR A). Significant liver disease is a contraindication. Itraconazole is less effective and more toxic, and griseofulvin is significantly less effective. Topical ciclopirox lacquer is also less effective than terbinafine, although it eliminates the risk for systemic toxicity. Fluconazole is not indicated for onychomycosis.

Four weeks after successful initial treatment of unilateral otitis media in a 2-year-old white male enrolled in a local day-care center, you reevaluate the child. He is asymptomatic, but you detect a middle ear effusion in the affected ear. The tympanic membrane is otherwise normal. The best management at this time would be A) inflation of the eustachian tube by the Valsalva maneuver B) an antihistamine daily for 30 days C) low-dose corticosteroids for 30 days D) referral to an ENT specialist E) no further treatment, with reevaluation in 2 months

ANSWER: E Otitis media is a major health problem in the United States; it is the number one reason children visit doctors and accounts for one-fourth of all antibiotic prescriptions. With appropriate antibiotics most patients will improve in 2-3 days. Persistence or worsening of symptoms requires immediate reevaluation, since complications such as bacterial resistance or meningitis may be developing. Occasionally a persistent middle ear effusion will be found on reexamination 10-14 days after initial treatment. Inflation of the eustachian tube using the method of Politzer or employing the Valsalva maneuver has been shown to be ineffective, as have antihistamines and systemic steroids. Most asymptomatic effusions with mild hearing loss will clear in 90 days if left alone.

A 7-year-old male is brought to your office by his mother because she is concerned about his ability to focus and stay still in school all day. She has paperwork from school and home, including his report card, Connor Rating Scales, behavioral screening, IQ tests, and performance testing. Your evaluation leads to a diagnosis of attention-deficit/hyperactivity disorder (ADHD) with no apparent comorbidities. As you discuss management options the mother expresses concern because her parents tell her that medications for ADHD are overprescribed and addictive. She asks you for further guidance. After providing the mother with comprehensive educational material, which one of the following would you recommend as first-line treatment? A) Cognitive-behavioral therapy B) Atomoxetine (Strattera) C) Bupropion (Wellbutrin) D) Clonidine (Catapres) E) Methylphenidate (Ritalin LA, Concerta)

ANSWER: E Research has consistently confirmed that stimulant medications are the most efficacious first-line treatment for children with attention-deficit/hyperactivity disorder (ADHD) (SOR A). No research supports the notion that the use of a stimulant in ADHD patients will promote addiction. To the contrary, some evidence suggests that ADHD patients who take stimulant medication have lower rates of drug abuse than those who do not. Diversion and misuse of prescription stimulants is a growing concern, however, and the use of a long-acting stimulant can decrease the chances for diversion. There are a number of well-supported behavioral interventions for ADHD. Most behavioral approaches focus on rewarding desired behavior and applying consequences for unwanted behavior to gradually reshape the child's thinking and actions. Interventions that help reinforce parental involvement include support groups, which connect parents who have children with similar problems, and parenting skills training, which gives parents techniques and tools for managing their child's behavior. Psychotherapy and cognitive-behavioral therapy have little or no documented effectiveness for the treatment of ADHD. 65 A multicenter, randomized study comparing the effectiveness of multimodal treatment (combined behavioral interventions and pharmacotherapy) with either treatment alone showed that combination treatment and pharmacotherapy alone yielded similar results and each was more effective than behavioral treatment alone or standard care in reducing core ADHD symptoms. A tool kit has been developed by the American Academy of Pediatrics and the National Initiative for Children's Healthcare Quality to help physicians improve the management of ADHD. While the second edition is only available in print form, the first edition can be downloaded free at http://www.nichq.org/adhd_tools.html.

A 44-year-old female has recently lost her best friend to ovarian cancer. She has no family history of cancer in her siblings, parents, or grandparents. She requests screening for ovarian cancer. Her physical examination, including a pelvic examination, is normal. According to current guidelines, which one of the following would be best for this patient? A) CA-125 testing B) CA-125 testing and ovarian ultrasonography C) Ovarian ultrasonography alone D) CT of the pelvis E) No screening

ANSWER: E Routine screening of the general population for ovarian cancer is not recommended by any professional society.

A 32-year-old female presents with a history of recurring headaches. They are usually unilateral, last for 24-48 hours, have a pulsatile quality, and are associated with nausea. She sometimes experiences photophobia as well. The patient describes the headaches as intense, usually requiring her to limit her activities. She has tried several over-the-counter migraine medications that have been minimally effective in aborting these headaches, and requests a prescription for an abortive therapy. Which one of the following would be the best choice for first-line therapy? A) Acetaminophen B) Acetaminophen/oxycodone (Percocet) C) Butalbital/aspirin/caffeine (Fiorinal) D) Prednisone E) Sumatriptan (Imitrex)

ANSWER: E Several medications from different classes are recommended as first-line abortive therapies to treat acute migraine. Because relatively few trials have directly compared the different medication classes, there are no definitive algorithms as to which class works best. NSAIDs and acetaminophen/aspirin/caffeine are recommended as first-line therapies and can be obtained over the counter (SOR A). Triptans are effective and safe for treatment of acute migraine and are recommended as first-line therapy (SOR A) but require a prescription. Opiates and barbiturates are not recommended because of their potential for abuse (SOR C). Acetaminophen alone is not effective, and the same is true of oral corticosteroids.

A 12-year-old African-American male is brought to your office by his parents because he has been limping for the past month. He says he has pain in the groin and knee, but the pain is poorly localized. On examination he is noted to be obese, with normal findings on examination of the knee. There is some decrease in internal rotation of the hip on the involved side. His gait is antalgic. The most likely cause of this problem is A) unreported trauma B) aseptic necrosis of the femoral head C) reactive arthritis D) juvenile rheumatoid arthritis E) slipped capital femoral epiphysis

ANSWER: E Slipped capital femoral epiphysis is often misdiagnosed, as the symptoms are frequently vague. It is the most common hip disorder in adolescents, with the age range being 9 to 15 years. It occurs when the proximal femoral epiphysis slips posteriorly and inferiorly on the femoral neck through the growth plate. The typical presentation is a limping child who may have pain in the groin, hip, thigh, or knee. Very often the pain is vague and poorly localized. It occurs more often in boys, with African-Americans and Pacific Islanders having a higher rate of involvement, possibly due to increased levels of obesity in these population groups. Physical findings vary, depending on the severity of the slippage. A child with a severe slip may not be able to bear weight. Obligatory external rotation of the involved hip is noted when the hip is passively flexed to 90°. Radiographs are needed to diagnose unstable slipped capital epiphysis, and should include frog-leg lateral views and anteroposterior views of both hips. Another cause of hip pain in adolescent patients is apophyseal avulsion fractures. Clinical features include pain after a sudden, forceful movement. Hip apophysitis presents as activity-related hip pain with a history of overuse and negative radiographs. In children under the age of 10 years, transient synovitis is also a common cause of hip pain. It occurs after a viral illness and is associated with negative radiographs but positive laboratory tests. Fractures may be seen in children on occasion, but there will be a history of trauma. Septic arthritis is an infrequent cause of hip pain in children, but patients have a history of fever with elevation of the WBC count and inflammatory joints. The diagnosis would be confirmed by joint aspiration. Legg-Calvé-Perthes disease is also infrequent, and features include vague hip pain with decreased internal rotation of the hip. The diagnosis is based on findings from radiographs or MRI.

A 38-year-old white female who complains of abdominal pain insists that she be referred for surgical evaluation. She has a history of multiple unexplained physical symptoms that began in her late teenage years. She is vague concerning past medical evaluations, but a review of her thick medical chart reveals multiple normal blood and imaging tests, several surgical procedures that have failed to alleviate her symptoms, and frequent requests for refills of narcotic analgesics. This history is most compatible with which one of the following? A) Hypochondriasis B) Malingering C) Panic disorder D) Generalized anxiety disorder E) Somatization disorder

ANSWER: E Somatization disorder usually begins in the teens or twenties and is characterized by multiple unexplained physical symptoms, insistence on surgical procedures, and an imprecise or inaccurate medical history. Abuse of alcohol, narcotics, or other drugs is also common in these patients. Hypochondriacs are overly concerned with bodily functions, and can often provide accurate, extensive, and detailed medical histories. Malingering is an intentional pretense of illness to obtain personal gain. Patients with panic disorder have episodes of intense, short-lived attacks of cardiovascular, neurologic, or gastrointestinal symptoms. Generalized anxiety disorder is characterized by unrealistic worry about life circumstances accompanied by symptoms of motor tension, autonomic hyperactivity, or vigilance and scanning.

A 16-year-old female has had foot pain for the past 3 weeks. She has no known history of trauma, but is participating in cross-country running events for her high school and has significantly increased her training schedule over the past 2 months. An examination reveals tenderness at the base of the fifth metatarsal, but no swelling. Radiographs are negative, but a radionuclide bone scan shows increased uptake in the proximal portion of the fifth metatarsal. Which one of the following would be most appropriate? A) A reduced training schedule for 1 month B) A DXA scan to evaluate bone mineral density C) A wooden cast shoe D) Ice and NSAID therapy only E) Referral to an orthopedist

ANSWER: E Stress fractures are caused by repetitive loading that exceeds the bone's ability to heal. They occur more commonly in female athletes. Patients should be evaluated for risk factors such as eating disorders, menstrual irregularities, and chronic medical conditions. Lower-extremity alignment, gait, and strength should also be evaluated. High-risk fractures such as those of the femoral neck, anterior cortex of the tibia, or proximal fifth metatarsal should be referred to an orthopedist, as there is a high likelihood of fracture-related complications.

Which one of the following is associated with testosterone replacement for men with hypogonadism? A) Osteoporosis B) Depression C) Reduced cognitive function D) Increased fat deposition E) Infertility

ANSWER: E Testosterone replacement therapy can improve many of the effects of hypogonadism. Beneficial effects include improvements in mood, energy level, sexual functioning, sense of well-being, lean body mass and muscle strength, erythropoiesis, bone mineral density, and cognition. However, there are also some risks associated with testosterone use, including an increased risk for prostate cancer, worsening of symptoms of benign prostatic hyperplasia, liver toxicity and tumor, worsening of sleep apnea and heart failure, gynecomastia, infertility, and skin diseases. Testosterone replacement therapy is not appropriate in men who are interested in maintaining fertility, as exogenous testosterone will suppress the hypothalamic-pituitary-thyroid axis.

A 45-year-old male asks you to evaluate his cardiovascular health status. He is currently asymptomatic, but wants to do everything he can to prevent heart disease and to understand his potential cardiovascular risk. His Framingham score indicates that he is at low risk (10-year risk <6%), and his physical examination is normal. He asks which laboratory and imaging tests he should have, and you recommend a lipid profile. According to the American College of Cardiology Foundation and the American Heart Association, which one of the following should also be recommended for this patient at this time? A) Lipoprotein and apolipoprotein levels B) A C-reactive protein level C) Measurement of cardiac calcium D) An ankle-brachial index E) No further testing

ANSWER: E The American College of Cardiology Foundation/American Heart Association guidelines for early cardiovascular assessment do not recommend lipoprotein and apolipoprotein levels. A C-reactive protein level can help to determine the need for statin therapy in men 50 and older and women 60 and older whose LDL-cholesterol levels are <130 mg/dL and who are not on lipid-lowering medication, hormone therapy, or immunosuppressive therapy, and who do not have clinical coronary heart disease, diabetes mellitus, chronic kidney disease, severe inflammatory disease, or contraindications to statins. A C-reactive protein level may also be reasonable in younger patients with intermediate, but not low, cardiovascular risk. Measurement of cardiac calcium levels is reasonable in patients whose cardiovascular risk is intermediate (10-year risk 10%-20%) or low-to-intermediate (10-year risk 6%-10%). An ankle-brachial index is reasonable for intermediate-risk, but not low-risk, patients. At this point in time, the patient described here does not meet any recommended criteria for further testing.

The American Heart Association recommends a goal blood pressure of 130/80 mm Hg for patients with A) heart failure B) pulmonary hypertension C) atrial fibrillation D) angina pectoris E) chronic kidney disease

ANSWER: E The American Heart Association recommends a goal blood pressure of 130/80 mm Hg or less for the treatment of hypertension in patients with diabetes mellitus, chronic kidney disease, or coronary artery disease.

An 85-year-old male is brought to your office by his family for a follow-up visit for Alzheimer's dementia. His dementia has been present for 4 years. He has been experiencing increasing agitation and delusions over the past several weeks, and the family requests a medication to "calm him down." Which one of the following is indicated in this situation according to FDA guidelines? A) Aripiprazole (Abilify) B) Haloperidol C) Olanzapine (Zyprexa) D) Risperidone (Risperdal) E) No antipsychotic drugs

ANSWER: E The FDA states that antipsychotics are not indicated for treating dementia-related psychosis. The reason for this is that the efficacy for antipsychotics has not been consistently shown in clinical trials and, in fact, patients treated with olanzapine functioned worse after treatment than did those who received a placebo. There is also evidence that these drugs may increase mortality from infection or heart-related conditions. Practice guidelines recommend the use of antipsychotics only after other options have been exhausted and symptoms are severe, persistent, and not responsive to nonpharmacologic interventions (SOR B).

A previously healthy 26-year-old white male carpenter reports episodes of chest tightness and dyspnea. He states that he feels better on weekends. He most likely has A) hypersensitivity pneumonitis B) toxic pneumonitis C) byssinosis D) benign pleural effusion E) occupational asthma

ANSWER: E The diagnosis of occupational asthma can be made when both bronchospasm and its relationship to the work environment can be demonstrated. A history of cough, wheezing, chest tightness, or episodic dyspnea in varying combinations or singly should lead one to suspect bronchospasm. Relating bronchospasm to the work environment can be done in several ways. A history of exposure to a known sensitizer is helpful, as is a pattern of symptoms occurring after exposure. With many agents the onset of symptoms may be delayed up to several hours. A 10% decrease in FEV1 measured before and after a work shift supports the diagnosis. Improvement of bronchospasm with removal from exposure also suggests the diagnosis. Treatment includes both standard pharmacologic therapy and removal from exposure as soon as possible. Hypersensitivity pneumonitis is an immune-mediated syndrome that is not as common as occupational asthma. It begins with malaise, fever, and myalgias 4-6 hours after exposure to an antigen to which the person has become sensitized. Byssinosis is due to exposure to the dust of hemp, flax, or cotton. Symptoms vary from reversible chest tightness on one or more days early in the work week to chronic bronchitis and permanent obstructive lung disease. Toxic pneumonitis or pulmonary edema is the result of very high exposure to irritant gases, metal dust, or metal fumes, usually associated with unusual circumstances such as a fire, explosion, or spill. Benign pleural effusions are the most common sequela during the first 20 years after asbestos exposure. The diagnosis is one of exclusion, made by ruling out other causes of exudative effusions in workers with known asbestos exposure.

A 45-year-old female with type 1 diabetes mellitus currently takes NPH insulin (Humulin, Novolin) twice a day. She expresses a desire to change to insulin glargine (Lantus). Her diabetes has always been well controlled, and her current hemoglobin A1c of 7.4% is typical for her. Which one of the following is most likely to be reduced if this change is made? A) Quality of life B) Hemoglobin A1c C) Morbidity from all causes D) Treatment costs E) Her risk for hypoglycemia

ANSWER: E The extended flat pharmacokinetic curve of long-acting insulin analogues makes once-daily administration of larger doses of insulin possible. Such treatment should, in theory, provide increased flexibility with regard to the timing of injections and improve compliance. This should improve control of the patient's diabetes, reduce the risk of hypoglycemia, and improve overall patient satisfaction. To date, however, the only proven benefit of treatment with insulin analogues is a reduction in the low rate of symptomatic, nocturnal, and overt hypoglycemia experienced by patients treated with isophane insulin. Although the total cost of treatment with insulin analogues is higher, a Cochrane review of the limited number of studies comparing insulin treatments showed no statistically significant differences in the hemoglobin A1c levels measured at the end of the studies in any treatment group (SOR C). Significant changes in morbidity, mortality, or quality of life have not been demonstrated (SOR C).

A 48-year-old white male has experienced five episodes of right upper quadrant pain during the past year. The most recent episode occurred 2 weeks ago. The episodes last 2-4 hours and are associated with nausea and vomiting. Which one of the following is most likely to provide an explanation for the patient's symptoms? A) A serum bilirubin level B) An AST (SGOT) level C) A plain film of the abdomen D) A HIDA scan E) Abdominal ultrasonography

ANSWER: E The patient described has a history compatible with gallbladder disease. In a patient with such a typical history, abdominal ultrasonography is likely to show gallstones and thus provide support for the diagnosis. Serum bilirubin and AST levels are usually normal except at the time of an attack. A HIDA scan may be useful if performed during an attack, since the scan assesses the patency of the cystic duct. A plain abdominal film will detect only 10%-15% of cases of cholelithiasis.

A 35-year-old primigravid schoolteacher awakens with a rash clinically consistent with varicella early in the 38th week of her pregnancy. She had a negative varicella titer early in her pregnancy. The clinical course is mild and all vesicles have either crusted over or healed 1 week later. She has an uncomplicated labor and vaginal delivery at 40 weeks gestation, and delivers a healthy-appearing male. Of the following options, which one is the most appropriate initial management for the newborn? A) Intravenously administered varicella immune globulin B) A weight-appropriate dose of intravenous acyclovir (Zovirax) C) Varicella vaccine D) Combination treatment with varicella vaccine, intravenous acyclovir, and varicella immune globulin E) Close observation only

ANSWER: E The result of neonatal varicella infection can be catastrophic, with a fatality rate approaching 30%. Maternal immunity is ideal, but since varicella vaccination is contraindicated during pregnancy the best alternative is advising the patient to avoid contact with infected individuals until safe postpartum immunization is possible. Maternal varicella infection is particularly problematic during weeks 13-20 of pregnancy (resulting in a 2% risk of congenital varicella in the newborn) and when the onset of maternal symptoms occurs from 5 days before until 2 days after delivery. Administration of varicella immune globulin to the expectant mother has not been shown to benefit the fetus or infant, but because pregnancy can increase the risk of serious complications in the mother the Advisory Committee on Immunization Practices (ACIP) recommends that administration to pregnant women be considered following known exposure. The ACIP also recommends that term infants born within the 7-day window described above, as well as all preterm infants, receive varicella immune globulin, and that those who develop any signs of varicella infection also be given intravenous acyclovir. Term infants delivered more than 5 days after the onset of maternal varicella are thought to have adequate passive immunity for protection and the expected benign course generally requires only observation.

When discussing end-of-life issues with patients, physicians may inadvertently send the wrong message. Which one of the following has the greatest potential to be misinterpreted by a patient? A) "We can offer many options to control your symptoms" B) "If you become extremely ill, would you like to be put on artificial life support?" C) "The cancer has not responded to the treatment as we had hoped" D) "We want to provide coordinated care with a team of professionals to help you remain comfortable" E) "It is time to consider withdrawal of care"

ANSWER: E The statement, "It is time to consider withdrawal of care," can make patients think that the physician no longer wants to care for them. It would be better to ask, "Do you think it is time for us to consider a different type of treatment that focuses on your symptoms?" Telling the individual that you want to provide intense coordinated care with a team that will treat symptoms and maintain comfort is a way of involving hospice without making the patient feel hopeless.

An asymptomatic 30-year-old female has developed hypertension that has been difficult to control despite the use of hydrochlorothiazide, lisinopril (Prinivil, Zestril), atenolol (Tenormin), and hydralazine. She sees you for a follow-up visit, and her blood pressure is 165/98 mm Hg. The examination is otherwise unremarkable, including cardiac auscultation and distal pulses. Her CBC, TSH level, complete metabolic panel, and urinalysis are all normal. Which one of the following tests would be best to confirm the most likely diagnosis? A) An aldosterone/renin ratio B) A renal biopsy C) 24-hour urinary free cortisol D) 24-hour urinary total metanephrines E) CT angiography

ANSWER: E There are several possible causes of secondary hypertension in young adults age 19-39, including coarctation of the aorta, thyroid dysfunction, renal parenchymal disease, and fibromuscular dysplasia. Fibromuscular dysplasia is more common in females, and has a predilection for causing stenosis of the renal arteries. The diagnosis can be made using MRI with gadolinium contrast media, or with CT angiography. Middle-aged adults (age 40-64) are more likely to have primary aldosteronism (evaluated with an aldosterone/renin ratio), sleep apnea, pheochromocytoma (associated with elevated metanephrines), or Cushing's syndrome (elevated 24-hour urinary cortisol). The patient described has no signs or symptoms of any of these problems.

A 24-year-old female presents with a 10-day history of cough productive of green sputum. Her past medical history is unremarkable and she is up to date on all immunizations. The patient's temperature is 37.2°C (98.9°F), blood pressure 127/76 mm Hg, pulse rate 89 beats/min, respiratory rate 24/min, and O2 saturation 95% on room air. Her physical examination is unremarkable except for a loose cough. Which one of the following is best supported by evidence for management of this patient's condition? A) A macrolide antibiotic such as azithromycin (Zithromax) B) An oral corticosteroid such as prednisone C) An inhaled -agonist such as albuterol (Proventil, Ventolin) D) An expectorant such as guaifenesin E) Reassurance that symptoms will likely resolve on their own within 3 weeks

ANSWER: E This patient has acute bronchitis. The most appropriate management option is to provide reassurance that symptoms will likely resolve on their own within 3 weeks. Approximately 90% of cases are caused by viruses, and antibiotics do not significantly change the course of the condition. For this reason, and because of concerns about antibiotic resistance and side effects from antibiotic use, antibiotics should not be used routinely for the treatment of acute bronchitis (SOR B). Despite this, approximately two-thirds of patients in the United States diagnosed with bronchitis are still treated with antibiotics. Corticosteroids and -agonists are not indicated in the absence of asthma or wheezing on examination. Expectorants have not been shown to be effective in the treatment of bronchitis (SOR B).

A 25-year-old medical assistant presents with a 3-month history of a tremor that began shortly after the death of her husband. The tremor starts abruptly and then spontaneously remits. It is not an action tremor and has no association with posture. On examination you notice the tremor severity increases with questions calling attention to her tremor symptoms and lessens when she is distracted with questions about her hobbies and summer plans. Her neurologic examination is completely normal, including no signs of dystonia, and she has no laboratory or radiologic evidence of disease. She denies taking any medications or using any substances that might cause a tremor. The patient also reports that the tremor does not improve with moderate alcohol consumption, and it did not respond to a trial of anti-tremor medications prescribed by another physician. Based on your findings, you suspect this patient most likely has which type of tremor? A) Cerebellar B) Dystonic C) Essential D) Parkinsonian E) Psychogenic

ANSWER: E This patient most likely has a psychogenic tremor, given its abrupt onset, spontaneous remission, changing tremor characteristics, and extinction with distraction (SOR C). Other characteristics of this case that suggest psychogenic tremor are the associated stressful life event, the patient's employment in a health care setting, and no evidence of disease by laboratory or radiologic investigations. In addition, the tremor increases with attention and has been unresponsive to anti-tremor medications (SOR C). Dystonic tremor is a rare tremor found in less than 1% of the population, and other signs of dystonia, such as abnormal flexion of the wrists, are usually present. Essential tremor is an action tremor and is usually postural; however, persons with essential tremor typically have no other neurologic findings. Essential tremor typically improves with alcohol consumption (2 drinks/day). A cerebellar tremor is usually associated with other neurologic signs, such as dysmetria (overshoot on finger-to-nose testing), dyssynergia (abnormal heel-to-shin testing and/or ataxia), and hypotonia. A parkinsonian tremor is most often a resting tremor, and although it may become less prominent with voluntary movement, it usually does not spontaneously remit.

A 45-year-old female presents to your office with a complaint of hair loss. Examination reveals thinning of the hair over the central superior portion of the scalp. Her frontal hairline is preserved. There are no oval patches of baldness and no scarring. She has a history of hirsutism, infertility, irregular menses, and cystic acne. Her testosterone, dehydroepiandrosterone sulfate, and prolactin levels are normal, as are thyroid and iron levels. Of the following, which one would be most appropriate for treatment of this patient's hair loss? A) Oral estrogen B) Oral corticosteroids C) Topical corticosteroids D) Finasteride (Propecia) E) Topical minoxidil (Women's Rogaine)

ANSWER: E This patient's examination is consistent with female pattern hair loss. Women with female pattern hair loss who also have a history of abnormal menses, infertility, cystic acne, and hirsutism should have an evaluation for hyperandrogenism. Minoxidil 2% topically is the only treatment approved by the FDA for treating female pattern hair loss in women over 18, but a hyperandrogenic state may limit the response to minoxidil. If the hyperandrogenism evaluation is normal, spironolactone, 100-200 mg daily, may slow the rate of hair loss. Approximately 90% of such women report a modest decrease in hair loss with this treatment.

A 65-year-old male who has been in good health presents to your office with a 2-day history of a sensation of pressure and hearing loss in his left ear. A physical examination and a thorough neurologic examination are both unremarkable. Both tympanic membranes are normal. An audiogram shows a 30-decibel hearing loss at three consecutive frequencies in the left ear, with normal hearing on the right. Placing a vibrating tuning fork in the midline of the forehead reveals sound lateralizing to the right ear. Which one of the following would be most appropriate at this point? A) CT B) A CBC, metabolic profile, and thyroid studies C) Nifedipine (Procardia) D) Acyclovir (Zovirax) E) Oral corticosteroids

ANSWER: E When a patient presents with sudden hearing loss it is important to distinguish between sensorineural and conductive hearing loss. Patients should be asked about previous episodes, and the workup should include both an assessment for bilateral hearing loss and a neurologic examination. Sudden sensorineural hearing loss is diagnosed by audiometry demonstrating a 30-decibel hearing loss at three consecutive frequencies, with no other cause indicated from the physical examination. Evaluation for retrocochlear pathology may include auditory brainstem response, MRI, or follow-up audiometry. Routinely prescribing antiviral agents, thrombolytics, vasodilators, vasoactive substances, or antioxidants is not recommended. Oral corticosteroids may be offered as initial therapy, and hyperbaric oxygen therapy may be helpful within 3 months of diagnosis. The guidelines also strongly recommend against routine laboratory tests or CT of the head as part of the initial evaluation.

An adult male has obvious gynecomastia, without galactorrhea, that has been present for the past 10 years. A careful drug history, physical examination, and endocrine and malignancy workups are negative. He wants the problem resolved. Which one of the following is the treatment of choice? A) Clomiphene (Clomid, Serophene) B) Danazol C) Tamoxifen (Soltamox) D) Topical testosterone (AndroGel) E) Surgery

ANSWER: E When gynecomastia persists for a prolonged period, the initial glandular hyperplasia is transformed to a progressive fibrosis and hyalinization. Surgery remains the mainstay of therapy. Medical management is most useful when the onset is recent or to prevent the initial development of the problem. All the drugs listed have been tried with varying success in this context, but their clinical usefulness is not established.

A 77-year-old white female complains of fatigue and mild dyspnea with exertion. After a thorough evaluation you conclude that she has early heart failure. She has no edema or evidence of volume overload, and echocardiography reveals an ejection fraction of 34%. Which one of the following would be most appropriate as INITIAL treatment? A) Digoxin B) Furosemide (Lasix) C) Hydrochlorothiazide D) Isosorbide dinitrate E) Lisinopril (Prinivil, Zestril)

ANSWER: E While most patients with heart failure should be treated with an ACE inhibitor and a diuretic, a subset of patients with heart failure present with only fatigue or mild dyspnea on exertion, and no evidence of volume overload. Since ACE inhibitors alone appear to prevent or slow the development of heart failure in patients with asymptomatic left ventricular dysfunction, it is reasonable to start an ACE inhibitor such as lisinopril in patients with very mild symptoms and observe to see if the symptoms resolve. The other medications listed are useful in the treatment of heart failure but would not be appropriate as initial treatment in the patient described.

A 77-year-old male presents for a periodic health evaluation. Your practice is organized as a patient-centered medical home, and this is the patient's initial visit. His records indicate that he received all recommended screening tests and immunizations 4 years ago, and he asks what screening tests are necessary at his age. The U.S. Preventive Services Task Force recommends that this patient be screened for which one of the following? A) Prostate cancer B) Colorectal cancer C) Abdominal aneurysm D) Dementia E) Depression

ANSWER: E While the U.S. Preventive Services Task Force (USPSTF) recommends against screening for prostate cancer using prostate-specific antigen testing, other screening methods have not been evaluated in controlled studies. For men who have smoked, one-time ultrasonography is recommended as a screen for aortic aneurysm between the ages of 65 and 75. The USPSTF has no recommendation for men who have never smoked. The USPSTF states that no evidence supports routine colorectal cancer screening in patients age 76-85, but that there may be some individuals with specific considerations for whom colorectal cancer screening would be recommended. At present, there is no evidence to support screening of older adults for dementia, but it is recommended that all adults be screened for depression when staff support is in place to ensure adequate diagnosis, treatment, and follow-up. In most instances, the elderly population will present to a primary care provider with somatic complaints (level of evidence 1b).

ANSWER: C Radiographs confirm the clinical diagnosis of small-bowel obstruction in most patients and more accurately define the site of obstruction. Small-bowel obstruction typically occupies the more central portions of the abdomen. Patients with mechanical small-bowel obstruction usually have minimal or no colonic gas. Films taken in the upright or lateral decubitus position in patients with small-bowel obstruction usually show multiple gas-filled levels, with the distended bowel resembling an inverted U. Patients with small-bowel obstruction are likely to be depleted of fluids and electrolytes, and will require intravenous fluids, electrolyte management, and surgical evaluation.

An 80-year-old male presents with a 4-hour history of generalized abdominal pain, vomiting, and fever to 101°F. On examination you note normal cardiovascular findings, generalized moderate abdominal tenderness, absent bowel sounds, and a normal rectal examination. Figure 1 shows a diagnostic abdominal film, which suggests A) a leaking abdominal aortic aneurysm B) toxic megacolon C) small bowel obstruction D) diverticulitis E) a perforated viscus

A 23-year-old female presents with recurrent unprovoked epistaxis. The patient's mother is known to have hereditary hemorrhagic telangiectasia. Contrast echocardiography is recommended to screen for which one of the following frequently associated conditions? A) Atrioseptal defect B) Ventricular septal defect C) Aortic root aneurysm D) Pulmonary arteriovenous malformation E) Myocardial perfusion defects

Answer D: Pulmonary arteriovenous malformations are found in 15%-30% of patients with hereditary hemorrhagic telangiectasia (HHT), also known as Osler-Weber-Rendu syndrome. All patients with possible or confirmed HHT should be screened for pulmonary arteriovenous malformations with contrast echocardiography (SOR C). While contrast echocardiography is used to detect atrioseptal and ventricular septal defects, neither of these conditions is particularly prevalent in HHT. Aortic aneurysms and myocardial perfusion defects are also not associated with HHT.

Which one of the following children should be referred immediately for evaluation of speech delay? A) A 12-month-old who babbles but speaks no words B) An 18-month-old who does not understand action words C) A 2-year-old who has a vocabulary of 25 words D) A 2-year-old who is unable to follow three-step directions E) A 3-year-old who has a vocabulary of 50 words

Answer E: Because speech-language therapy is effective for primary expressive language disorders, referral as early as possible is critical (SOR A). Red flags suggesting the need for immediate evaluation include no babbling in a 12-month-old, not saying "mama" or "dada" at 18 months, a vocabulary of less than 25 words at age 2, and using less than 200 words at age 3. Children should be able to follow two-step commands by 2 years of age (SOR A).

ANSWER: B Eczema craquelé, a common complication of aging, is due to dryness of the skin. It is best treated with wet compresses and antibiotics to remove crusts and suppress infection, followed by topical corticosteroids and lubricants. The primary lesions do not suggest scabies or necrobiosis lipoidica, and neurodermatitis and lichen sclerosis are secondary responses to itching.

During a follow-up visit for a 72-year-old white female with type 2 diabetes mellitus, her daughter asks you to treat the lesions shown in Figure 7. These lesions bother the patient and she complains of itching, especially at night. The most likely diagnosis is A) chronic, reactive scabies infestation B) eczema craquelé (xerotic eczema) C) neurodermatitis D) necrobiosis lipoidica E) lichen sclerosus

ANSWER: D The salient feature of atrial fibrillation is the absence of P waves, along with normal QRS complexes that are irregular in time (irregularly irregular) and sometimes vary in amplitude. Sinus tachycardia, paroxysmal tachycardia, multifocal atrial tachycardia, and atrial flutter are all associated with P waves that are constantly related to QRS complexes, although they may sometimes be abnormal and difficult to discern.

The EKG shown in Figure 2 reveals A) sinus tachycardia B) paroxysmal atrial tachycardia C) multifocal atrial tachycardia D) atrial fibrillation E) atrial flutter

ANSWER: C The condition shown is representative of a simple anterior dislocation of the lunate. The semilunar shape of the lunate bone is displaced anterior to the distal radial articular surface. Occasionally, a transnavicular fracture may occur along with this injury and is termed a trans-scaphoid perilunate fracture-dislocation. There is no evidence in the radiograph shown, however, of dislocations of the other areas mentioned.

The most likely diagnosis of the condition shown in the radiographs in Figure 4 is A) dislocation of the trapezium B) dislocation of the scaphoid C) dislocation of the lunate D) fracture of the distal radius E) fracture of the distal ulna


Conjuntos de estudio relacionados

Paediatrics - Disability Dynamic Online Independent Learning Module [ANSWERS]

View Set

Exam 1 - Emergency Nursing, Cardiac, Chronicity

View Set

Pharm Exam 2 NCLEX Prep Questions

View Set

Understanding Luxury Homes Features

View Set